Anda di halaman 1dari 83

KUMPULAN SOAL – SOAL UJIAN MASUK PERGURUAN TINGGI NEGERI

BAHASA INDONESIA
UMPTN 2001 – 2009

SOAL BAHASA INDONESIA TAHUN 2009

1. SNMPTN 2009
(1) Sekelompok anjing laut utara (Mirounga angustirostris) sedang berganti kulit di sebuah pantai di dekat San
Simeon, California. (2) Pantai ini sudah biasa dijadikan tempat berkumpulnya sekelompok anjing laut utara. (3) Di
tempat ini pada musim panas, anjing laut mengalami pergantian kulit besar-besaran yang berlangsung sekitar satu bulan.
(4) Selama sebulan, mereka kehilangan bulu dan kulit. (5) Anjing laut utara menghangatkan badan mereka sewaktu
musim berganti kulit dengan cara berkumpul di pantai sambil menunggu tumbuhnya kulit dan bulu yang baru.

Kalimat topik paragraf di atas terletak pada nomor ....


A. (1)
B. (2)
C. (3)
D. (4)
E. (5)

2. SNMPTN 2009
Departemen Agama seharusnya sanggup menyelenggarakan ibadah haji secara lebih baik. Sesuai dengan undang-
undang terbaru, departemen ini tetap menjadi regulator sekaligus operator, bahkan ikut menentukan anggota komisi
pengawas. Peningkatan kualitas pelayanan diharapkan terjadi lewat pembenahan manajemen. Keluhan mulai
muncul ketika belum ada perbaikan yang berarti. Lihatlah, baru-baru ini 89 anggota jemaah calon haji dari Jakarta
dilaporkan mengalami diare saat berada di Madinah. Penderitaan mereka diduga akibat makanan yang basi atau
tidak memenuhi standar kesehatan. Insiden seperti ini tidak perlu terjadi seandainya perusahaan katering menjaga
kualitas makanan yang disajikan. Kualitas layanan akan terjamin lagi jika tim kesehatan selalu mengecek dan
mengawasi makanan untuk jemaah calon haji. (Koran Tempo, 21 November 2008)

Pemyataan berikut ini merupakan opini dari teks di atas, KECUALI ...
A. jemaah calon haji Indonesia tidak memperoleh layanan maksimal dari Departemen Agama.
B. Departemen Agama tidak bekerja secara profesional dalam penyelenggaraan ibadah haji.
C. manajemen Departemen Agama lemah sehingga tidak mampu meningkatkan pelayanaan penyelenggaraan ibadah
haji.
D. tim kesehatan tidak mengecek dan mengawasi makanan sehingga jemaah calon haji makan nasi basi.
E. koordinasi tidak berjalan baik di antara komponen panitia penyelenggara ibadah haji di Departemen Agama.

3. SNMPTN 2009
Saat ini diperlukan realisasi janji dari para pemimpin negeri ini, yakni peningkatan kualitas pendidikan dengan
anggaran hingga 20% dari total APBN dan APBD, Sayangnya, baru sekitar 44 kabupaten di Indonesia yang
mengalokasikan anggaran pendidikan sesuai dengan ketentuan tersebut. Padahal, peningkatan kualitas pendidikan
ini telah masuk dalam delapan prioritas pembangunan dalam program Presiden Susilo Bambang Yudhoyono. Di dalam
salah satu dari delapan prioritas itu, disebutkan dengan jelas bahwa peningkatan akses dan kualitas pendidikan dan
kesehatan menjadi prioritas pem bangunan pem erintah In donesia. Nam un, realisasinya masih sangat
mengecewakan (Koran Sindo, 21 November 2008)

Pernyataan berikut yang tidak relevan dengan isi teks di atas adalah ...
A. pemerintah menjanjikan anggaran pendidikan sebesar 20% dari APBN dan APBD.
B. alokasi anggaran pendidikan 20% baru dilakukan oleh 44 kabupaten.
C. janji pemerintah menetapkan anggaran pendidikan 20% belum ditepati.
D. pendidikan dan kesehatan merupakan prioritas utama program pemerintah.
E. realisasi dari delapan prioritas pembangunan sangat mengecewakan.

4. SNMPTN 2009
Di bawah ini, manakah yang termasuk kalimat efektif?
A. Dalam musyawarah itu menghasilkan lima ketetapan yang harus dilaksanakan bersama.
B. Dalam pertemuan itu memutuskan bahwa Andi yang ditunjuk menjadi ketua Koperasi.

Bidang Studi BAHASA INDONESIA 1


KUMPULAN SOAL – SOAL UJIAN MASUK PERGURUAN TINGGI NEGERI

C. Kegagalan proyek itu karena perancangan yang tidak baik dan pengawasan yang kurang cermat.
D. Meskipun negara itu merupakan penghasil kapas nomor satu dunia, tetapi harga tekstilnya sangat tinggi.
E. Budiman ingin menjadi juara umum di sekolahnya, tetapi ia hanya berhasil menjadi juara ketiga.

5. SNMPTN 2009
Exxon Mobil memiliki komitmen untuk selalu menangani risiko-risiko yang berkaitan dengan keselamatan saat
bekerja. Langkah pertama untuk mencapai prestasi kerja tanpa cacat adalah dengan memastikan tingkat motivasi
dan partisipasi karyawan yang tinggi dalam mencegah terjadinya kecelakaan. Di samping itu, karyawan harus
mengecek dan membenahi adanya kerusakan. Exxon Mobil yakin bahwa penciptaan lingkungan kerja yang aman
dapat memberikan kinerja yang baik. Dengan bantuan teknologi baru, Exxon Mobil terus berupaya membuat
kegiatan operasionalnya lebih aman, sehat, dan ramah lingkungan. Di Exxon Mobil, kesempurnaan dalam masalah
keselamatan, kesehatan, dan perlindungan lingkungan merupakan unsur inti yang diutamakan. (Majalah Tempo,
17-23 November 2008)

Simpulan teks tersebut yang paling tepat adalah ...


A. Exxon Mobil sangat peduli terhadap keselamatan kerja karyawan.
B. Exxon Mobil menetapkan dua langkah untuk mencapai prestasi kerja.
C. lingkungan kerja yang aman akan menghasilkan kinerja yang baik.
D. teknologi baru dapat membuat kegiatan di Exxon Mobil lebih aman.
E. motivasi clan partisipasi karyawan Exxon Mobil harus tinggi.

6. SNMPTN 2009
Blog awalnya adalah catatan harian yang ditempelkan (posting) ke sebuah situs yang dapat diakses siapa saja. Blog
menjadi semakin populer ketika kemudian tersedia situs yang menyediakan diri sebagai rumah gratis bagi
blog, seperti blogspot dan wordpress. Apabila di dunia intemasional blog mulai dikenal tahun 1998, di Indonesia, kata
Wicaksono, muncul tahun 2004 dan meningkat tajam tahun 2007. "Uniknya, di Indonesia paling-paling hanya 5 persen
yang bicara politik. Mengherankan juga karena sekarang ruang untuk berpendapat jauh lebih bebas," kata
Wicaksono.
Lainnya berisi mulai dari kuliner, wisata, atau perjalanan, jual-beli, hingga prosa dan puisi. Beberapa blog begitu
populernya sehingga dapat membangkitkan nilai ekonomi. (Kompas, 23 November 2008)

Blog di tanah air menjadi salah satu cara berkomunikasi di dunia maya yang populer karena...
A. ada banyak blog dibuat orang Indonesia sehingga tidak harus berkomunikasi dalam bahasa Inggris.
B. ada situs yang menyediakan diri sebagai rumah gratis untuk blog sehingga pengguna tidak perlu mengeluarkan
biaya.
C. pengguna blog akan dipandang sebagai orang modern karena dapat berkomunikasi dengan peralatan komunikasi
modern.
D. pengguna blog dapat berbicara politik dan lain-lain secara bebas karena sulit diidentifikasi jati dirinya.
E. siapa saja dapat bergabung di sana dan apa saja dapat disampaikan meskipun ada berbagai masalah.

7. SNMPTN 2009
Seseorang akan mengembangkan karangan yang bertema upaya mengatasi pergaulan bebas pada kaum remaja di
perkotaan.
Berdasarkan terra tersebut, topik-topik untuk kerangka karangan yang runtut adalah ...
A. masalah yang timbul akibat pergaulan bebas pada kaum remaja di perkotaan, pentingnya mengatasi pergaulan
bebas, langkah mengatasi pergaulan bebas, saran yang dapat dilakukan untuk mengatasi pergaulan bebas.
B. pentingnya mengatasi pergaulan bebas pada kaum remaja di perkotaan, masalah yang timbul akibat pergaulan bebas,
langkah mengatasi pergaulan bebas, saran yang dapat dilakukan untuk mengatasi pergaulan bebas.
C. langkah mengatasi pergaulan bebas pada kaum remaja di perkotaan, pentingnya mengatasi pergaulan bebas, masalah
yang timbul akibat pergaulan bebas, saran yang dapat dilakukan untuk mengatasi pergaulan bebas.
D. pentingnya mengatasi pergaulan bebas pada kaum remaja di perkotaan, langkah mengatasi pergaulan bebas, masalah
yang timbul akibat pergaulan bebas, saran yang dapat dilakukan untuk mengatasi pergaulan bebas.
E. pentingnya saran yang dapat dilakukan untuk mengatasi pergaulan bebas di perkotaan, masalah yang timbul akibat
pergaulan bebas, langkah mengatasi pergaulan bebas, saran yang dapat dilakukan untuk mengatasi pergaulan bebas.

8. SNMPTN 2009
..... Salah satu penyakit kulit adalah acne vulgaris. Tidak ada seorang pun di dunia ini yang tidak pernah menderita
penyakit ini. Acne vulgaris adalah penyakit peradangan folikel sebasea yang umumnya terjadi pada mass remaja
dan dapat sembuh sendiri. Dalam masyarakat umum, acne vulgaris biasa dikenal dengan istilah jerawat.
Kalimat manakah yang tepat untuk mengisi titik-titik di atas.
A. Acne vulgaris adalah penyakit kulit biasa.
B. Acne vulgaris pernah dialami semua manusia.

Bidang Studi BAHASA INDONESIA 2


KUMPULAN SOAL – SOAL UJIAN MASUK PERGURUAN TINGGI NEGERI

C. Penyakit kulit banyak jenis dan macamnya.


D. Jerawat banyak jenis dan macamnya.
E. Penyakit yang diderita remaja bermacam-macam.

9. SNMPTN 2009
Pasar uang dan pasar modal Indonesia belum pulih. Rupiah masih terus mengalami tekanan mengikuti penurunan
sebagian besar mata uang Asia. Masalah minimnya pasokan dolar AS di pasar valas semakin mempersulit mata
uang lokal ini. pada penutupan perdagangan valas tanggal 24 November 2008, rupiah melemah hingga 320 poin ke
posisi 12.320 per dolar AS. Rupiah bahkan sempat menembus 12.325 per dolar AS. Pelaku pasar cemas karena belum
ada sentimen positif dari dalam negeri. Oleh karena itu, pasar lebih memilih memegang dolar AS dalam kondisi pasar
global yang sedang rentan ini.

Paragraf di atas dikembangkan dengan menggunakan pola sebab-akibat karena ...


A. kalimat utamanya menyatakan hubungan sebab dan akibat.
B. kalimat pertama menyatakan sebab dan kalimat lainnya menyatakan akibat.
C. kalimat ketiga menyatakan sebab, sedangkan kalimat lainnya menyatakan akibat.
D. kalimat pertama dan ketujuh menyatakan akibat, sedangkan lainnya menyatakan sebab.
E. kalimat ketujuh menyatakan akibat, sedangkan kalimat lainnya menyatakan sebab.

10. SNMPTN 2009


Tokoh-tokoh ternama, seperti Charlie Chaplin, Jawaharlal Nehru, Gamal Abdul Naser, dan Bung Karno, pernah
menginap di sini. Gaya seni Art Deco-nya yang unik seolah menjadi trademark hotel yang satu ini. Cobalah
menyusuri Jalan Asia Afrika di Bandung. Di sang ada gedung yang memiliki lengkung bangunan unik berwarna
keabuan. Keunikan tersebut seolah-olah tak terpisahkan dari sejarah Kota Kembang yang pernah kental sekali aura
Eropanya. Hotel yang kini disebut Savoy Homann Bidakara Hotel ini selalu menjadi kebanggaan `urang Bandung'.
Selanjutnya, di bawah pengelolaan Van Es, bangunan hotel Homann yang semula sempit dan sederhana diperluas
serta dimodernisasi menjadi salah satu hotel terkemuka di Asia Tenggara. Gaya seni Art Deco yang melanda daratan
Eropa pada tahun 1920-an ikut mewarnai bangunan Homann lama, dan hiasan interior, jendela kaca patri, ornamen
dinding, model meubel, sampai kap lampu, semuanya berbau Art Deco. (Koran Tempo, 21 November 2008)

Makna tersirat yang terdapat pada kutipan teks tersebut adalah ...
A. setiap kota memiliki bangunan hotel kebanggaan warganya sebagaimana warga Kota Bandung memiliki hotel
Savoy Homann.
B. Hotel Savoy Homann menjadi kebanggan warga Kota Bandung karena megah, serta pernah ditempati pemimpin
dan seniman besar dunia.
C. bangunan hotel menjadi indah jika mengadopsi model bangunan Eropa sebagaimana model bangunan Hotel
Savoy Homann di Kota Bandung.
D. hotel menjadi kebanggaan warga jika memiliki sejarah panjang sebagaimana Hotel Savoy Homann yang
ditempati tokoh terkenal.
E. Hotel Savoy Homann Bidakara yang berada di Kota Bandung bergaya Art Deco Eropa karena berbentuk
lengkung.

11. SNMPTN 2009


NASA selesai menguji coba sebuah sistem yang baru pertama kali diluncurkan, yaitu sebuah sistem 'internet
antarplanet'. Jaringan ini akan dapat dipergunakan secara otomatis untuk menyebarluaskan informasi ke bumi,
wahana antariksa, dan para astronot. Wahana antariksa biasanya melakukan komunikasi langsung dengan bumi
untuk memberikan instruksi kepada rover-rover Mars dalam mini eksplorasinya. Spirit dan opportunity
mentransmisikan data ke orbiter lalu diteruskan kembali ke bumi. (Media Indonesia, 23 November 2004)

Gagasan utama paragraf di atas adalah ...


A. NASA selesai menguji coba sistem komunikasi baru internet antarplanet.
B. jaringan internet antarplanet secara otomatis menyebarkan informasi.
C. jaringan internet antarplanet tidak memerlukan bantuan tenaga manusia.
D. wahana antariksa selalu melakukan komunikasi langsung dengan bumi.
E. sistem baru internet antarplanet NASA bernama Spirit dan Opportunity.

12. SNMPTN 2009


Baru-baru ini sebuah lembaga daripada survei pendidikan tinggi di seluruh dunia tidak berapa lama ini telah
mengeluarkan daftar peringkat tentang perguruan tinggi yang berkualitas di seluruh dunia.

Perbaikan terhadap kalimat di atas adalah ...


A. Tidak berapa lama ini sebuah lembaga survei pendidikan tinggi telah mengeluarkan daftar peringkat tentang

Bidang Studi BAHASA INDONESIA 3


KUMPULAN SOAL – SOAL UJIAN MASUK PERGURUAN TINGGI NEGERI

perguruan tinggi yang berkualitas di seluruh dunia.


B. Baru-baru ini sebuah lembaga dari survei pendidikan tinggi telah mengeluarkan daftar peringkat perguruan
tinggi berkualitas di seluruh dunia.
C. Tidak berapa lama ini sebuah lembaga daripada survei pendidikan tinggi baru saja mengeluarkan daftar
peringkat perguruan tinggi yang berkualitas di seluruh dunia.
D. Baru-baru ini sebuah lembaga survei pendidikan tinggi mengeluarkan daftar peringkat perguruan tinggi
berkualitas di seluruh dunia.
E. Baru-baru ini sebuah lembaga survei pendidikan tinggi telah mengeluarkan daftar peringkat tentang perguruan
tinggi berkualitas di seluruh dunia.

13. SNMPTN 2009


Keanekaragaman hayati dan pemandangan ... bawah laut di perairan Pulau Lemukutan, yang menjadi bagian dari
Kawasan Konservasi Laut Daerah (KKLD) Bengkayang, dalam setengah tahun terakhir makin ... wisatawan.
KKLD Bengkayang berada di sekitar 35 kilometer sebelah barat Pulau Kalimantan di pesisir pantai Bengkayang.
Di situ terdapat ... pulau seperti Pulau Lemukutan, Randayan, Penata Besar, Penata Kecil, Baru, dan Kabung.

Untuk mengisi titik-titik pada teks di atas, kata-kata yang tepat adalah ...
A. pesona, disenangi, kelompok.
B. indah, dikenal, bentangan.
C. elok, diketahui, banyak.
D. cantik, dikunjungi, deretan.
E. alam, diminati, gugusan.

14. SNMPTN 2009


Berikut ini adalah identitas buku yang digunakan sebagai sumber pustaka.
Judul Buku Penulis Penerbit Kota Tahun
1 . Horizon Yusuf Anas Al-Huda Jakarta 2006
Manusia
2. Kalimat Efektif Ida Bagus Rapika Aditama Bandung 2007
Putrayasa
3. Retorika Modern Jalaludin Rakhmat Rosdakarya Bandung 2006

Manakah penulisan daftar pustaka yang tepat untuk buku-buku di atas?


A. Anas, Yusuf, 2006, Horizon Manusia, Jakarta; Al-Huda.
Putrayasa, Ida Bagus, 2007, Kalimat Efektif,Bandung; Rapika Aditama.
Rakhmat, Jalaludin, 2006, Retorika Modern, Bandung; Rosdakarya.

B. Anas, Yusuf. 2006. Horizon Manusia. Jakarta, Al-Huda.


Putrayasa, Ida Bagus. 2007. Kalimat Efektif.Bandung, Rapika Aditama.
Rakhmat, Jalaludin. 2006. Retorika Modern.Bandung, Rosdakarya.

C. Anas, Yusuf. 2006. Horizon Manusia. Jakarta: Al-Huda.


Putrayasa, Ida Bagus. 2007. Kalimat Efektif.Bandung: Rapika Aditama.
Rakhmat, Jalaludin. 2006. Retorika Modern.Bandung: Rosdakarya.

D. Anas, Yusuf. 2006. Horizon Manusia. Jakarta: Al-Huda.


Putrayasa, Ida Bagus. 2007. Kalimat Efektif.Bandung: Rapika Aditama.
Rakhmat, Jalaludin. 2006. Retorika Modern.Bandung: Rosdakarya.

E. Anas, Yusuf. 2006. "Horizon Manusia ". Jakarta: Al-Huda.


Putrayasa, Ida Bagus. 2007. "Kalimat Efektif'.Bandung: Rapika Aditama
Rakhmat, Jalaludin. 2006. "Retorika Modern".Bandung: Rosdakarya.

15. SNMPTN 2009


Pembelajaran berbasis teknologi komputer perlu segera disosialisasikan sampai ke sekolah-sekolah di pelosok
tanah air.
Pernyataan tersebut terdapat dalam buku "Pembelajaran Berbasis TIK" halaman 32 karya Rahma Arif yang
diterbitkan oleh Penerbit Fantasi pada tahun 2009.

Bidang Studi BAHASA INDONESIA 4


KUMPULAN SOAL – SOAL UJIAN MASUK PERGURUAN TINGGI NEGERI

Teknik pengutipan teks di atas yang betul adalah ...


A. Menurut Arif (2009:32) adalah "Pembelajaran ber ba si s t e kn ol ogi kom put er perlu segera disosialisasikan
sampai dengan sekolah-sekolah pelosok tanah air."
B. "Pembelajaran berbasis teknologi komputer perlu segera disosialisasikan sampai dengan sekolah-sekolah
pelosok tanah air" (Arif, 2009:32).
C. Arif (2009:32) "pembelajaran berbasis teknologi komputer perlu segera disosialisasikan ke sekolah-sekolah di
pelosok tanah air."
D. Menurut Arif (2009:32) bahwa pembelajaran berbasis teknologi komputer perlu segera disosialisasikan ke
sekolah-sekolah di pelosok tanah air.
E. "Pembelajaran berbasis teknologi komputer perlu segera disosialisasikan ke sekolah-sekolah di pelosok tanah air"
(dalam Arif, 2009:32).

UM UNDIP 2009
Soal No.1 sampai dengan No.20 gunakan Petunjuk A

Bacaan

(1) Trend jumlah penduduk miskin di Indonesia dari waktu ke waktu lebih banyak tinggal di wilayah pedesaan.
Tahun 1970 sampai dengan 1978, penduduk miskin lebih banyak tinggal di pedesaan, antara tahun 1980 sampai dengan
1990 lebih banyak tinggal di kawasan perkotaan,dan 1993 hingga 2000 jumlah penduduk miskin kembali lebih banyak
di pedesaan. Bahkan tahun 1998,selisih jumlah penduduk miskin yang tinggal di pedesaan cukup menyolok lebih besar
daripada mereka yang tinggal di kota.

(2) Beberapa karakteristik penduduk miskin dapat dilihat dari jumlah anggota rumah tangga,pendidikan kepala
rumah tangga,dan sumber pendapatan. Menurut BPS,tahun 1993 rata-rata jumlah anggota keluarga penduduk miskin
sebesar 5,9 orang,sedangkan jumlah keluarga rumah tangga yang hidup di atas garis kemiskinan lebih kecil dari itu. Hal
ini menunjukkan bahwa kepala rumah tangga miskin mempunyai beban yang lebih berat daripada yang tidak miskin.

(3)Penduduk miskin di pedesaan 72,00 persen dipimpin oleh kepala rumah tangga yang tidak tamat SD dan 24,30
persen dipimpin oleh kepala rumah tangga berpendidikan tamat SD.Penduduk miskin di perkotaan 57,00 persen
dipimpin oleh kepala rumah tangga tidak tamat SD dan 31,80 persen dikepalai oleh seorang tamatan SD.

(4) Karakteristik lain yang berkaitan dengan pendidikan dan distribusi penduduk adalah sumber pendapatan.Sumber
pendapatan utama penduduk miskin dari sektor pertanian sebesar 62,00 persen,sektor perdagangan 10,40 persen,sektor
industri sebesar 7,40 persen, sektor jasa 6,50 persen dan selebihnya dari selain sektor yang disebutkan tersebut. Dengan
demikian,sektor pertanian merupakan kantong kemiskinan penduduk.

(sumber:Gunawan Sumodiningrat,2001).

Jawablah pertanyaan nomor 1-4 berdasarkan bacan di atas.

1. UM UNDIP 2009
Bacaan di atas lebih tepat jika diberi judul…
A. Trend jumlah penduduk miskin
B. Karakteristik penduduk miskin
C. Distribusi penduduk miskin
D. Trend dan karakteristik penduduk miskin
E. Trend dan distribusi penduduk miskin

2. UM UNDIP 2009
Bacaan diatas merupakan jenis karangan…
A. Eksposisi
B. Argumentasi
C. Narasi
Bidang Studi BAHASA INDONESIA 5
KUMPULAN SOAL – SOAL UJIAN MASUK PERGURUAN TINGGI NEGERI

D. Deskripsi
E. Persuasi

3. UM UNDIP 2009
Jika bacaan tersebut dilanjutkan paragraf kelima,maka gagasan utama yang tepat adalah
A. Sektor pertanian
B. Karakteristik dan ditribusi sektor perdagangan
C. Karakteristik dan distribusi sektor pertanian
D. Karakteristik sektor industri dan jasa
E. Sektor jasa

4. UM UNDIP 2009
Kata trend pada paragraf pertama dapat diganti kata:
A. Pada umumnya
B. Dominan
C. Faktor determinan
D. Terkonsentrasi
E. Kecenderungan

5. UM UNDIP 2009
Penulisan kata gabung dibawah ini benar ,KECUALI
A. Terminal bus antarkota di Semarang selalu dilanda banjir.
B. seorang residivis harus mempertanggungjawabkan perbuatan kriminalnya.
C. Rendy meneruskan studinya di program pascasarjana Undip
D. gerakan KB sebagai suatu gerakan untuk mewujudkan program caturwarga
E. Karena prestasi akademiknya yang gemilang,Anton ditawari bekerja di kantor asuransi Bumiputera.

6. UM UNDIP 2009
Kalimat di bawah ini yang menggunakan peyorasi adalah
A. Bupati itu sering menerima amplop dari rekanan yang mendapatkan proyek
B. pemimpin gerombolan di Medan sudah ditangkap aparat kepolisian.
C. peran wanita sangat besar dalam pembangunan bangsa ini.
D. tidak bahasa orang itu lemah lembut kepada siapa pun.
E. Ary akan kembali berlayar ke Eropa minggu depan.

7. UM UNDIP 2009
Kalimat manakah yang mengandung makna piktorial?
A. Wah,dasar perempuan!
B. WC itu kotor sekali.
C. Meja itu bundar
D. Satu tahun sama dengan dua belas bulan.
E. Itu buku saya

8. UM UNDIP 2009
Rumah besar itu terbakar tadi malam.Pola kalimat tersebut sama dengan pola kalimat
A. Presiden menyerahkan penghargaan kepada para ilmuwan yang berprestasi luar biasa.
B. Pedagang itu menawarkan barang dagangannya.
C. Dia memperhatikan polisi mengatur lalu lintas.
D. Kami berteduh di bawah pohon.
E. Berita itu disiarkan ke seluruh dunia.

9. UM UNDIP 2009
Perkembangan Penggunaan Kontrasepsi Modern di Indonesia Tahun 1977-2007
Bidang Studi BAHASA INDONESIA 6
KUMPULAN SOAL – SOAL UJIAN MASUK PERGURUAN TINGGI NEGERI

Grafik di atas dibaca:…

A. Pemakai kontrasepsi modern sejak tahun 1977 sampai dengan 2007 terus mengalami peningkatan
B. Pemakaian kontrasepsi modern sejak tahun 1977 sampai dengan 2007 terus mengalami
peningkatan,kecuali tahun 1991
C. Pemakaian kontrasepsi modern sejak tahun 1977 sampai dengan 2007 terus mengalami
peningkatan,kecuali tahun 1991 dan 2007
D. Pemakaian kontrasepsi modern sejak tahun 1977 sampai dengan 2007 terus mengalami
peningkatan,kecuali tahun 2007
E. Pemakaian kontrasepsi modern antara tahun 1977 dan 2007 dalam kisaran 20 persen sampai dengan 70
persen

10. UM UNDIP 2009


Kata bulan yang dipakai hanya dalam arti denotative terdapat pada kalimat:
A. Ia kejatuhan bulan
B. Jangan mau jadi bulan-bulanan
C. Ia berbulan madu ke Bali
D. Telah dua bulan ia pergi
E. Bulan muda dua hari lagi

11. UM UNDIP 2009


Dalam makalah ini membicarakan hubungan tingkat pendidikan orang itu ( ibu) dengan prestasi akademik
anak.
Kalimat tersebut merupakan kalimat tidak efektif. Agar efektif,maka pembetulannya adalah

A. Makalah ini dibicarakan hubungan tingkat pendidikan orang tua (ibu) dengan prestasi akademik anak.
B. Dalam makalah ini membicarakan tingkat pendidikan orang tua (ibu) dengan prestasi akademik anak.
C. Makalah ini membicarakan hubungan tingkat pendidikan orang tua (ibu) dengan prestasi akademik anak.
D. Makalahnya membicarakan hubungan tingkat pendidikan orang tua (ibu) dengan prestasi akademik anak

Bidang Studi BAHASA INDONESIA 7


KUMPULAN SOAL – SOAL UJIAN MASUK PERGURUAN TINGGI NEGERI

E. Di dalam makalah ini dibicarakan hubungan tingkat pendidikan orang tua (ibu) dengan prestasi akademik
anak.

12. UM UNDIP 2009


Proses pendaurulangan platik menjadi bahan bakar minyak (BBM) ini dilakukan secara bertahap. Cara in
dilakukan supaya kandungan kimia pada plastic sesuai seperti kandungan kimia pada solar dan bensin. Selain
itu,proses ini antara lain untuk memisahkan antara dammar eksposisi dari plastic,kemudian plastik dihancurkan
untuk selanjutnya dicairkan dalam bak minyak dengan suhu 40 derajat celcius.
Petikandi atas merupakan wacana laporan penelitian pada bagian…

A. Latar belakang
B. Tinjauan pustaka
C. Landasan teori
D. Pembahasan
E. Penutup

13. UM UNDIP 2009


Bioteknologi pertanian modern dikembangkan melalui dua kegiatan dasar,yaitu kultur jaringan dan rekayasa
genetika. Teknik kultur jaringan memungkinkan kita mengisolasi jaringan sel-sel tumbuhan,kemudian
menanamnya di luar tumbuh-tumbuhan itu sendiri.
Intisari dari berita di atas adalah

A. Kegiatan dasar kultur jaringan dan rekayasa genetika


B. Dua kegiatan dasar modern dilakukan melalui bioteknologi pertanian
C. Pengisolasian jaringan dan sel-sel tumbuhan
D. Bioteknologi modern akan dikembangkan
E. Penanaman kultur jaringan diluar tumbuhan.

14. UM UNDIP 2009


(1) Suasana melodramatis juga tak ketinggalan dalam buku ini.
(2) Semisal cerpen-cerpen “Mentari Senjata”, ”Alunan Musik di Lantai Dansa”, “Mama”,”Kau Telah
membunuh Anakku”, “ Bertobat” dan “Di Manakah Kau?” (3) Dalam cerpen “ Mentar Senja” karya Bing Hu
dikisahkan sepasang anak muda yang berpacaran. (4) Pada akhir cerita tak diduga mereka sebenarnya saudara
kandung dar orang tua mereka yang berpisah ketika masih berpacaran. (5) Dalam cerpen ini faktor
kebetulannya sangat menonjol bak film percintaan atau serial telenovela.

Petikan tersebut merupakan resensi. Hal tersebut dapat dilihat dari kalimat ke…

A. (1)
B. (2)
C. (3)
D. (4)
E. (5)

15. UM UNDIP 2009


Tak ada yang tahu siapa sesungguhnya pengarang hikayat Hang Tuah. Tahun persis kapan pertama kali kisah
ini muncul pun hingga kini tak bias dipastikan. Tak ubahnya karya-karya sastra lama umumnya, kisah Hang
Tuah ini pun diduga pertama kali dikenal berupa cerita lisan. Kemudian, dalam bentuk naskah tertulis,hikayat
Hang Tuah diduga pertama kali ditulis pada abad ke-16
Unsur cuplikan yang dibahas dalam cuplikan di atas…

A. Penokohan
B. Alur
C. Latar
D. Kepengarangan
E. Sudut pandang
Bidang Studi BAHASA INDONESIA 8
KUMPULAN SOAL – SOAL UJIAN MASUK PERGURUAN TINGGI NEGERI

16. UM UNDIP 2009


Sengsara gerangan takdirnya untung
Sebagai nasib si bunga betung

Hanyut di sungai terkatung-katung

Diejekkan kera dan lutung

Berdasarkan bentuk dan pilihan katanya, penggalan puisi di atas tergolong…

A. Puisi klasik
B. Puisi baru
C. Puisi modern
D. Puisi terjemahan
E. Puisi melayu

17. UM UNDIP 2009


Sudah lelah
Ke hutan-hutan seorang penyair

Merindu pohon-pohon dan belukar

Dikejar diam ke dalam batu-batu

(“Bahasa Gelombang “,Syauqi Alief Atar)

Kata hutan-hutan dan belukar mengimajinasikan masalah…

A. Pencarian penyair yang sulit


B. Kebahagiaan dalam hidup
C. Kehidupan sehari-hari penyair
D. Tantangan di balik pengejaran
E. Orang-orang yang selalu dikejar oleh musuh.

18. UM UNDIP 2009


Aku tahu Emak tentu tidak akan dating. Tidak mau,katanya tidak pantas.”Sekolah itukan tempat priyayi
lho,Gus. Emakmu ini,ndak ilok kalau berada di tempat itu”.Oala, Mak! Priyayi itu zaman dulu,sekarang ini
orang sama saja,yang membedakan itu kan isinya,” aku menekankan telunjuk ke keningku.”Itulah Gus yang
Emak maksudkan priyayi. Emak tidak mau ke tempat yang angker itu. Nanti Emakmu itu hanya akan jadi
tontonan saja, karena plonga-plongo kayak kerbau”.

(Cerpen”Emak yang Perkasa”,Agus Fakri H)

Watak tokoh Emak dalam penggalan cerpen di atas adalah…

A. Jujur,baik,pengertian
B. Rendah hati,rendah hati
C. Tidak sombong,baik,optimis
D. Penuh pengertian,lugu
E. Rendah hati,lugu,penuh pengertian

19. UM UNDIP 2009


Petualang: Nona sudah cukup pengalaman. Sudah bias menafsirkan sendiri dengan tafsiran yang setepat-
tepatnya.

Bidang Studi BAHASA INDONESIA 9


KUMPULAN SOAL – SOAL UJIAN MASUK PERGURUAN TINGGI NEGERI

Perempuan: Hah, alangkah sayangnya bahwa Tuan-tuan yang mengaku manusia-manusia terhormat,tidak tahu
cara menilai kehormatan diri pribadi.

Politikus: Cukup! Bicara Nona sudah kelewat batas susila!

Konflik yang dibangun dalam dialog di atas berupa…

A. Konflik kepentingan
B. Konflik batin
C. Konflik budaya
D. Konflik ideology
E. Konflik pemikiran

20. UM UNDIP 2009


Kalau begitu Zaelani meninggal pada hari kedua setelah disunat?Padahal,tidak banyak darah yang keluar dari
lukanya. Zaelani itu bersifat penurut dan pendiam. Sebelum disunat,beberapa hari yang lalu ia mengurung diri
untuk mempersiapkan kondisinya.
Amanat yang tersirat dalam penggalan cerpen di atas adalah…

A. Kematian tak terduga


B. Penyebab kematian
C. Takdir Tuhan
D. Kegagalan dalam selamat
E. Sifat pendiam seseorang

U M UG M 2 0 09
B AH A S A I N D O N E S I A

Petunjuk A digunakan dalam menjawab soal nomor 1 sampai dengan nomor 20.

Bacalah bacaan berikut dengan cermat, kemudian jawablah pertanyaan nomor 1 'sampai dengan 3.

Begitu matahari terbit di Batu Kambar, Desa Hinas Kiri, Kecamatan Batang Alai Timer, Kabupaten Hulu
Sungai Tengah, Kalimantan Selatan, ratusan warga ke luar rumah. Ada yang langsung pergi ke pahumaan (ladang),
ada yang pergi ke pasar, ada pula yang berangkat ke sekolah. Warga di pedalaman Pegunungan Meratus memang
terbiasa bangun pagi karena mereka jugs sudah terbiasa tidur sore akibat tidak adanya aliran IWO. Karen itu,
aktivitas pagi di Pegunungan Meratus betul-betul menggeliat dan dinamis. Hari itu kebetulan hari pasar tradisional
yang hanya ada sekali dalam sepekan. Masyarakat dari berbagai balai adat Dayak Meratus, yang jarak tempuhnya
hingga 18 jam dengan jalan kaki, berduyun-duyun ke pasar untuk membeli keperluan hidup "modem", mulai dari
bahan pangan, sandang, hingga papan.
Tradisionalisme, mulai dari cara makan, cara mandi, cara berinteraksi, hingga pandangan hidup, betul-
betul masih melekat dan menjadi identitas warga pedalaman Pegunungan Meratus. Lambat lawn keterisolasian
pedalaman memang bisa ditembus, tetapi hingga kini kehidupan alami masih melekat. Baik di hulu Sungai Selatan
maupun di hulu Sungai Tengah yang mempunyai komunitas adat terbesar di Kalimantan Selatan suasananya sama-
sama masih tradisional. Tradisionalisme di satu sisi menjadi kebanggaan, tetapi di sisi lain menyisakan pertanyaan
akan capaian sebuah kesejahteraan masyarakat.

1 . U M UG M 2 0 09
Judul yang tepat untuk bacaan di atas adalah…
A. Kisah Kehidupan dari Batu Kambar, Desa Hinas Kiri
B. Kehidupan Tradisional Masyarakat Pegunungan Meratus.
C. Identitas Warga Pedalaman Pegunungan Meratus.
D. Tradisionalisme Menjadi Kebanggaan bagi Masyarakat Pegunungan Meratus.
E. Selama 18 jam Berjalan Menuju Pasar untuk Keperluan Hidup "Modern".

2 . U M UG M 2 0 09
Inti kalimat keenam dalam paragraf pertama bacaan di atas adalah
A. Masyarakat berjalan ke pasar selama 18 jam.
B. Masyarakat membeli keperluan hidup "modern".
C. Masyarakat berduyun-duyun ke pasar.
Bidang Studi BAHASA INDONESIA 10
KUMPULAN SOAL – SOAL UJIAN MASUK PERGURUAN TINGGI NEGERI

D. Masyarakat Dayak Meratus terdiri atas berbagai balai adat.


E. Masyarakat Dayak Meratus berjalan kaki ke pasar.

3 . U M UG M 2 0 09
Dalam bacaan di atas terdapat pemyataan implisit berikut, kecuali
A. Warga Pedalaman Pegunungan Meratus tidak mengenal kendaraan bermotor.
B. Warga Pedalaman Pegunungan Meratus belum menggunakan listrik.
C. Tradisionalisme di satu sisi adalah kesedihan atas keterbelakangan.
D. Pasar tradisional hanya ada satu kali dalam seminggu.
E. Masyarakat Pegunungan Meratus belum menikmati kebutuhan sekunder dan tersier.

4 . U M UG M 2 0 09
Pada saat ini, kerisauan temyata bukan hanya milik para petani yang kesulitan mendapatkan pupuk. Bambang
Riyanto, seorang perajin jenang dan tempe keripik asal Purwokerto, bahkan harus pontangpanting mencari
minyak tanah agar usahanya itu tetap berlangsung. Kerisauan jelas membayang ketika akan memasuki musim
liburan akhir tahun lalu. "Musim panen"bagi industri kecil makanan oleh-oleh ini tentu tidak akan dibiarkan
lewat begitu saja. Untuk mendapatkan minyak tanah yang cukup untuk produksi, ia harus mengerahkan seluruh
anggota keluarganya, bahkan para tetangganya, untuk ikut antre membeli minyak tanah.

Pikiran pokok paragraf di atas adalah


A. kerisauan para petani dan perajin makanan oleh-oleh di Purwokerto karena sulitnya pupuk dan minyak tanah.
B. "musim panen" bagi industri kecil makanan oleh-oleh.
C. kerisauan Bambang Riyanto, seorang perajin jenang dan tempe keripik asal Purwokerto.
D. kelangkaan minyak tanah di Purwokerto.
E. suka duka perajin jenang dan tempe keripik asal Purwokerto.

5 . U M UG M 2 0 09
Jangan dikira hanya Italia yang mempunyai menara miring. Indonesia pun mempunyai bangunan bersejarah
"berbau miring". Istana Tampaksiring, Bali, memang tidak miring seperti Pisa, tetapi letaknya di desa yang sejak
zaman baheula melegenda sebagai tanah miring. Kata tampaksiring dicomot dari bahasa setempat, tampak dan
siring yang artinya "telapak miring". Pembangunan Istana Tampaksiring tak bisa dipisahkan dari kiprah mantas
Presiden RI Bung Karno (BK). Konon, pemilihan tempat, proses pembangunan, hingga finishing Istana
Tampaksiring, semua melibatkan BK. la sudah mengenal lahan berpemandangan indah ini jauh sebelum disulap
menjadi istana. Sebelum 1955 (istana dibangun 1957) BK sudah sering istirahat di bekas tempat tetirah Raja
Gianyar yang dikeramatkan penduduk setempat.

Inti paragraf di atas adalah…


A. Istana Tampaksiring yang merupakan bekas tempat tetirah Raja Gianyar sering digunakan leh BK untuk
istirahat.
B. Seperti Italia, Indonesia juga memiliki bangunan yang didirikan di atas tanah berkontur miring.
C. Istana Tampaksiring dibangun di desa bertanah miring sehingga bangunan ini juga "berbau miring".
D. Raja Gianyar membangun Istana Tampaksiring pada tahun 1957 dengan melibatkan BK.
E. Istana Tampaksiring yang dibangun di desa bertanah miring dan berpemandangan indah tidak bisa dipisahkan
dari BK.

6 . U M UG M 2 0 09
Adat Badui yang ketat tercermin dalam pepatah "Yang panjang tidak boleh dipotong, yang pendek tidak boleh
ditambah". Hal ini sengaja ditekankan agar tradisi lama yang sesungguhnya tetap dapat dipertahankan. Namun,
tidak berarti masyarakat Badui tidak menerima sesuatu yang baru, atau sama sekali menutup sesuatu dari dunia
luar. Di sans sini teriihat prang Badui membuat pelbagai percobaan dengan mengubah kombinasi atau variasi
unsur-unsur adat walaupun tetap dalam batas-batas yang ditetapkan oleh tetua adat.

Inti paragraf di atas adalah


A. Adat Badui menekankan kemurnian tradisi lama sehingga sedapat mungkin mereka menolak berkontak dengan
dunia luar.
B. Adat Badui berkembang dengan inovasi unsur-unsur adat dan membuka diri terhadap campur tangan dari
dunia luar.
C. Adat Badui mempertahankan tradisi dan mengembangkannya dengan berbagai percobaan atas variasi unsur-
unsur adat.
D. Adat Badui telah mengubah tradisi lama dengan tradisi baru sehingga menjadi lebih maju dan modern.
E. Adat Badui mengikuti batas-batas yang ditetapkan oleh tetua adat yang menekankan kesederhanaan.

7 . U M UG M 2 0 09
Bidang Studi BAHASA INDONESIA 11
KUMPULAN SOAL – SOAL UJIAN MASUK PERGURUAN TINGGI NEGERI

Beberapa bulan lalu Dinas Kepurbakalaan, Budaya, dan Pariwisata Jambi memprakarsai napak tilas
Ekspedisi Pamalayu. Ekspedisi penyusuran Sungai Batanghari ini bertujuan mencatat segala potensi yang
ada di sepanjang rute untuk perencanaan dan pengembangnan pariwisata daerah tersebut. Penyusuran itu
mengingatkan kita pada Ekspedisi Pamalayu pada tahun 1286, yakni ekspedisi diplomatik Kerajaan
Singasari ke Kerajaan Melayu. Ekspedisi atas prakarsa Raja Kertanegara itu disertai pengiriman arcs
Amoghapasa. Menurut catatan sejarah, pengiriman aras itu menempuh rute Laut Jawa, laut bagian timur
Pulau Sumatera, hingga memasuki muara dan hulu Sungai Batanghari.

Dari bacaan di atas dapat disimpulkan bahwa


A. Pengiriman arca Amoghapasa ke Kerajaan Melayu menempuh perjalanan jauh.
B. Penyusuran itu untuk mengenang kebesaran Ekspedisi Pamalayu pada tahun 1286, yakni eskpedisi diplomatik
Kerajaan Singasari ke Kerajaan Melayu.
C. Ekspedisi Pamalayu tahun 1286 menunjukkan bahwa Kerajaan Singasari memiliki kekuatan maritim yang
kuat.
D. Napak Tilas Ekspedisi Pamalayu itu dilakukan dalam rangka mengembangkan pariwisata sepanjang rute
pelayaran yang dilewati utusan Raja Kertanegara ke Kerajaan Melayu.
E. Napak Tilas Ekspedisi Pamalayu itu dibiayai oleh Dinas Kepurbakalaan, Budaya, dan Pariwisata Jambi.

8 . U M UG M 2 0 09
Upacara wiwit termasuk salah satu tradisi penting dalam dunia pertanian masyarakat Jawa, khususnya dunia
pertanian yang berkaitan dengan budi daya tanaman pokok (padi). Hingga kini, upacara tersebut masih bisa
dijumpai di pelosok pedesaan walaupun semakin sedikit yang melestarikannya. Upacara wiwit semakin langka
dijumpai, seiring dengan munculnya banyak perubahan nilai-nilai budaya dalam kehidupan sosial komunitas
petani, semakin sempitnya lahan pertanian mereka, dan adanya perubahan sistem produksi pertanian di Jawa.

Dari bacaan di atas dapat disimpulkan bahwa


A. Langkanya upacara wiwit karena sistem pertanian di Jawa mulai berubah.
B. Langkanya upacara wiwit karena banyaknya perubahan nilai-nilai budaya.
C. Upacara wiwit masih bisa dijumpai di pelosok pedesaan di Jawa walaupun semakin sedikit yang
melestarikannya.
D. Upacara wiwit yang kini makin langka merupakan tradisi penting dalam budi daya tanaman padi di Jawa.
E. Upacara wiwit termasuk salah satu tradisi penting dalam dunia pertanian masyarakat Jawa.

9 . U M UG M 2 0 09
Wakil Presiden Bank of China, Zhu Min, menegaskan bahwa belanja konsumsi di Amerika Serikat, negara
dengan ekonomi terbesar di dunia akan turun 10% tahun ini akibat lesunya ekonomi global. Konsumsi Amerika
dalam beberapa tahun ini diperkirakan menyusut tajam. Penurunan belanja negeri Obama itu diprediksi akan
lebih besar lagi dalam tiga atau empat tahun mendatang. Padahal, Amerika dan Jepang merupakan pasar ekspor
terbesar produk Cina. Akibatnya, krisis di Amerika tersebut membuat pertumbuhan ekonomi Cina mulai melambat
beberapa bulan ini. Imbas krisis itu membuat banyak perusahaan di Negeri Tirai Bambu menjadi goyah.

Berdasarkan isi bacaan di alas, pernyataan berikut ini benar, kecuali


A. Cina banyak tergantung perdagangannya pada Amerika Serikat.
B. Penurunan belanja Amerika sebesar 10% akan diikuti lagi dengan penurunan konsumsi negara itu di tahun-
tahun berikutnya.
C. Belanja konsumsi sebesar-besamya merupakan satu-satunya jalan untuk memperbaiki ekonomi Amerika.
D. Banyak perusahaan di Cina menjadi tidak stabil setelah pembelian produknya berkurang di Amerika.
E. Zhu Min berpendapat bahwa krisis di Amerika membuat pertumbuhan ekonomi Cina tidak begitu signifikan
beberapa bulan ini.

1 0 . U M UG M 2 0 09
Dalam era globalisasi ini komputer telah dipergunakan dalam banyak bidang kehidupan, misalnya dalam bidang
kesehatan, pertahanan dan keamanan, termonuklir, perbankan, transportasi, dan sensus penduduk. Ditinjau dari
segi ukurannya, komputer yang dipergunakan itu cukup bervariasi, mulai dari komputer mini, misalnya jam
tangan digital, kalkulator dan telepon seluler; komputer mikro, misalnya laptop dan PC (personal computer);
hingga komputer makro, misalnya server dalam unit-unit LAN (Local Area Network) dan mainframe yang biasa
dipergunakan di bank-bank atau perusahaanperusahaan besar lainnya.

Paragraf di alas dapat diringkas menjadi


A. Komputer menjadi bagian penting dalam era globalisasi.
B. Komputer yang dipakai dalam kehidupan sehari-hari memiliki ukuran bervariasi.
C. Komputer digunakan dalam berbagai bentuk dan bidang kehidupan.

Bidang Studi BAHASA INDONESIA 12


KUMPULAN SOAL – SOAL UJIAN MASUK PERGURUAN TINGGI NEGERI

D. Komputer menjadi perangkat penting dalam mempermudah pekerjaan manusia.


E. Komputer digunakan di bank-bank dan perusahaan-perusahaan besar.

1 1 . U M UG M 2 0 09
Kalimat yang efektif adalah
A. Data perilaku manajerial yang dikumpulkan oleh peneliti dan tenaga teknisi dari lokasi penelitian diperoleh
dengan cars mewawancarai dan menyebarkan kuesioner kepada informan.
B. Data perilaku manajerial yang dikumpulkan oleh peneliti dan dibantu tenaga teknisi dari lokasi penelitian
diperoleh dengan metode wawancara dan kuesioner kepada informan.
C. Data perilaku manajerial yang dikumpulkan oleh peneliti dengan asisten tenaga teknisi dari lokasi penelitian
diperoleh dengan metode wawancara dan kuesioner kepada informan.
D. Data perilaku manajerial yang dikumpulkan peneliti dan dibantu tenaga teknisi dengan datang langsung ke
lokasi penelitian diperoleh dengan metode wawancara dan kuesioner kepada informan.
E. Data perilaku manajerial yang dikumpulkan oleh peneliti dan tenaga teknisi diperoleh lewat wawancara dan
kuesioner.

1 2 . U M UG M 2 0 09
Kalimat berikut tidak baku, kecuali
A. Dengan adanya 'liberalisasi" sistem pemilihan semakin membuat profit DPR mendatang sulit diprediksi, dan
tidak mempunyai jaminan kinerja mereka semakin membaik.
B. Menurut diagnosa dokter, pasien itu positif mengidap penyakit flu burung.
C. Dalam Bab I penelitian ini membicarakan tentang latar belakang, masalah, dan landasan teori.
D. Jadi, di negara kita tidak dikenal diskriminasi terhariap warga negara yang menyandang cacat,
E. Untuk merenovasi museum itu memedukan dana yang tidak sedikit.

1 3 . U M UG M 2 0 09
Yang merupakan kalimat baku adalah
A. Buku ini terdapat istilah-istilah kekerabatan pada orang Jawa dan Batak dalam susunan masyarakat yang
ditulis berdasarkan ilmu antropologi.
B. Dari beberapa pokok persoalan yang diberikan untuk membandingkan dua atau lebih dialek, antara lain dalam
bidang fonetik atau semantik.
C. Adapun yang akan saya uraikan di sini ialah kebersihan dan kesehatan, saya terdorong untuk
mengemukakannya, karena Bering dilalaikan orang dan sesungguhnya kebersihan dan kesehatan itu perlu,
sebab dengan bersih tentu akan menjadi sehat.
D. Karena nilai yang didapatkan lebih besar dari penolakan maka hipotesis nihil ditolak.
E. Jika data yang diolah tidak memenuhi syarat, balk dari segi kuantitas maupun kualitasnya, hasil penelitian di
lapangan itu tidak dapat dipertanggungjawabkan secara ilmiah.

1 4 . U M UG M 2 0 09
Kalimat berikut tidak baku, kecuali
A. Dengan teknik tersebut para astronom dapat memperkirakan ukuran planet meskipun belum diketahui
massanya, namun diperkirakan antara 5,7 hingga 11 massa bumi.
B. Landasan pacu Bandara Ahmad Yani Semarang tergenang sepanjang 500 meter dan harus menggunakan 23
pompa air untuk menyurutkan genangan.
C. Bandara menunda semua penerbangan sejak Minggu pagi hingga pukul 21.00 karena kondisi landasan pacu
masih buruk.
D. Komisi pemilihan umum berencana mengatur penetapan calon terpilih yang berpihak pada calon perempuan,
jika partai politik meraih tiga kursi DPR/DPRD, salah satunya mesti diberikan kepada calon perempuan yang
perolehan suaranya terbanyak.
E. Berdasarkan pada data di Dinas Bina Marga Pemkot Surabaya, maka titik-titik genangan itu menyebar hampir
di seluruh wilayah Surabaya mulai sisi paling barat, timur, hingga ke selatan.

1 5 . U M UG M 2 0 09
Beberapa waktu lalu berita tentang susu bermelamin, biskuit beracun, dan bahan kosmetik yang mengandung
bahan merkuri dan hidrokinon dengan cepat tersebar melalui media massa sehingga meresahkan pedagang dan
konsumen yang memiliki, menyimpan, memperjualbelikan, atau menggunakannya.

Inti kalimat di atas adalah


A. Berita meresahkan.
B. Berita tersebar.
C. Bahan beracun tersebar.
D. Pedagang dan konsumen resah.
E. Pedagang dan konsumen menggunakan.
Bidang Studi BAHASA INDONESIA 13
KUMPULAN SOAL – SOAL UJIAN MASUK PERGURUAN TINGGI NEGERI

1 6 . U M UG M 2 0 09
Dalam konferensi intemasional kebudayaan Melayu itu tidak diputuskan tempat penyelenggaraan konferensi
berikutnya.

Kalimat yang memiliki pola yang sama dengan pola kalimat di atas adalah
A. Obama, Presiden Amerika yang baru terpilih itu, mengawali kariernya dari bawah.
B. Seperti telah kita ketahui, perekonomian Amerika Serikat sangat berpengaruh.
C. Berdasarkan pengarahan pimpinan, pembangunan gedung baru itu dapat dilakukan secara bertahap.
D. Kita sadari bahwa bencana alam dapat datang kapan saja tanpa kita duga.
E. Telah kami bicarakan dalam rapat terdahulu bahwa kenaikan gaji pegawai baru bisa dilaksanakan bulan depan.

1 7 . U M UG M 2 0 09
Kata jual yang bermakna literal terdapat dalam kalimat
A. Sejarah membuktikan bahwa sebagian pribumi menjual bangsanya sendiri pada tangan penjajah.
B. Kau tidak akan mendapat jodoh kalau terlalu jual mahal.
C. Bagi sebagian anak muda, blog adalah media untuk jual tampang.
D. Para penipu berkedok hariiah biasanya menjual Hama perusahaan tertentu.
E. Oknum paramedic di RS swasta itu terlibat aksi jual beli gigi palsu.

1 8 . U M UG M 2 0 09
Semua pimpinan partai kini sedang memeras otak untuk memenangkan pemilu.

Gabungan kata yang semakna dengan memeras otak adalah…


A. Penyanyi itu memamerkan suara emasnya.
B. Buruh tarsi itu setiap hari membanting tulang.
C. Pemerintah daerah tidak boleh hanya berpangku tangan ketika melihat rakyatnya menderita karena banjir.
D. Sulit berdiskusi pada orang yang keras kepala.
E. Pemimpin bertangan besi itu kemarin meninggal dunia.

1 9 . U M UG M 2 0 09
Ibu Ria adalah klien saya. Masih gesit, masih baca puisi bersama-sama teman seumurnya. Sekitar dua minggu
menjelang Natal saya bertemu dia di kantornya. Dia bercerita bahwa anaknya yang akan membuat garage sale
mengusulkan melego meja makan dan meja di ruang tamu rumahnya.

Yang dimaksud dengan garage sale adalah…


A. obralan
B. pameran makanan.
C. pesta di gudang.
D. ulang tahun.
E. reuni.

2 0 . U M UG M 2 0 09
Ekspor timah dari Indonesia yang merupakan penghasil timah terbesar kedua di dunia setelah Cina semakin
berkurang setelah pemerintah menutup pertambangan ilegal di Bangka dan Belitung. Upaya ini menjadikan timah
sebagai salah satu komoditas yang berperforma terbaik di London Metal Exchange pada tahun-tahun terakhir.

Kata upaya dalam teks ini sangat tepat bila diganti dengan…
A. usaha.
B. ikhtiar.
C. kebijakan.
D. kebijaksanaan.
E. tindakan.

1. SIMAK UI 2009
Cara penulisan gabungan kata yang benar terdapat dalam kalimat…
A. Hendaknya kita senantiasa bekerjasama dalam menyelesaikan segala permasalahan.
Bidang Studi BAHASA INDONESIA 14
KUMPULAN SOAL – SOAL UJIAN MASUK PERGURUAN TINGGI NEGERI

B. Surat yang Anda kirim belum Anda tandatangani.


C. Masyarakat di desa terpencil itu berterima kasih atas bantuan yang Anda berikan.
D. Dia harus mempertanggung-jawabkan perbuatannya.
E. Orang itu di-" meja hijau" -kan.

2. SIMAK UI 2009
Penulisan gabungan kata di bawah ini benar semuanya, kecuali...
A. tanggung jawab, kerjasama, tandatangan, terima kasih
B. semipermanen, mancanegara, pascasarjana, saptakrida
C. acapkali, adakalanya, barangkali, bagaimana
D. mata ajar, meja tulis, orang tun, simpang empat
E. kacamata, dukacita, sapu tangan, halalbihalal

3. SIMAK UI 2009
Makna gabungan kata mereka berdaulat dapat juga dilihat pada gabungan kata dalam kalimat
A. Mereka mengobrol sambil sesekali bersenda gurau.
B. Kami merasa senasib sepenanggungan.
C. Orang itu malu-malu kucing ketika dipersilahkan makan oleh tuan rumah.
D. Dia menjadi hilang semangat setelah mengetahui pembalap unggulannya kalah.
E. Pelari itu bersiap-siap sambil pasang kuda-kuda.

4. SIMAK UI 2009
Sebagai bentuk penghargaan, Presiden Yudhoyono mengumumkan kebijakan pemberian beasiswa penuh hingga
strata pendidikan Doktor di Universitas manapun di dunia kepada peraih medali emas olimpiade sain.
Kata yang penulisannya baku adalah…
A. beasiswa
B. Doktor
C. Universitas
D. manapun
E. sain

5. SIMAK UI 2009
Dr. Handini, peneliti terbaik tingkat internasional, menerima hadiah uang sebesar 100.000 dolar AS.
Gagasan utama kalimat tersebut adalah...
A. Dr. Handini peneliti terbaik yang menerima hadiah
B. Dr. Handini peneliti terbaik tingkat internasional
C. Dr. Handini menerima hadiah
D. Dr. Handini penerima hadiah sebagai peneliti terbaik tingkat internasional.
E. Dr. Handini menerima hadiah uang sebesar 100.000 dolar AS

6. SIMAK UI 2009
Berdasarkan penyelidikan ikan lou han secara bersistem, kritis, dan ilmiah melalui berbagai percobaan,
nongnong merupakan tempat menyimpan makanan cadangan yang berisi protein dan zat gizi pelengkap lainnya..
Nongnong itu dapat menjadi besar jika diberi makanan ternak dengan kandungan protein dan lemak yang tinggi
seperti spirulina.

Kata-kata berikut ini berhubungan dengan apa yang tergambarkan dalam kutipan di atas
A. penelitian tes, nutrisi
B. riset, cara, persediaan
C. pengamatan, cara, pakan
D. riset, nutrisi, pakan
E. pengamatan, memuat, nutrisi

7. SIMAK UI 2009
Di antara reptil raksasa yang hidup di zaman purba, ternyata, terdapat dinosaurus yang hanya seukuran ayam.
Mungkin, inilah dinosaurus terkecil yang pernah hidup di Amerika Utara jutaan tahun silam. Spesies dinosaurus
yang diberi nama Albertonykus borealis tersebut merupakan anggota kelompok dinosaurus yang disebut
Alvarezsaurus. Kelompok satwa purba itu pernah mendiami pule kawasan Asia dan Amerika Selatan.

Pernyataan yang sesuai dengan kutipan di atas adalah...


A. Albertonykus borealis adalah dinosaurus terkecil di dunia.
B. Albertonykus borealis ditemukan jutaan tahun silam.
Bidang Studi BAHASA INDONESIA 15
KUMPULAN SOAL – SOAL UJIAN MASUK PERGURUAN TINGGI NEGERI

C. Albertonykus borealis hidup di Asia dan Amerika Selatan.


D. Albertonykus borealis soma dengan alvarersaurus.
E. Albertonykus borealis ditemukan di America Utara.

8. SIMAK UI 2009
Tenis meja berasal dari Eropa pada abad pertengahan sebagai kombinasi dari permainan tenis kuno, lawn
tennis, dan badminton. Tanis meja mulai populer di Inggris pada pertengahan abed ke-19 dengan beberapa
nama, antara lain pingpong, gossima, dan whiff-whaff . Permainan tersebut dikreasikan sebagai permainan
hiburan setelah makan malam. Selanjutnya permainan tenis meja mendapatkan wadah resmi pada 15 Januari
1926 atas prakarsa Dr. Goerge Lehman dari Jerman.

Maksud kalimat berakhir pada kutipan tersebut adalah...


A. tenis meja berasal dari Jerman
B. pencetus tenis meja adalah Dr. Goerge Lehman
C. organisasi tenis meja dunia terbentuk pada 15 Januari 1926 atas ide Dr. Goerge Lehman
D. tenis meja ditemukan oleh Dr. Goerge Lehman
E. tenis meja ditemukan pada 15 Januari 1926

9. SIMAK UI 2009
Manusia diizinkan Tuhan untuk memanfaatkan semua isi alam ini termasuk memakan daging binatang.
…………. manusia tidak diizinkan menyakiti, menyiksa, apalagi menyia-nyiakan binatang. Siapa yang menyiksa
binatang, berdosa besar. ………… siapa yang menolong binatang akan mendapat pahala.

Kata yang sesuai untuk mengisi rumpang di atas adalah...


A. tetapi; jadi
B. tetapi; sehingga
C. akan tetapi; tetapi
D. namun; dengan demikian
E. namun; sebaliknya

10. SIMAK UI 2009


Penulisan kata dengan ejaan yang BENAR terdapat pada kalimat…….
A. Olah raga bertujuan untuk mencapai tingkat kebugaran jasmani.
B. Dalam seminar tersebut, dibahas masalah prilaku anak-anak remaja.
C. Untuk menghangatkan badan kita dapat mengonsumsi mie rebus jawa.
D. Kenaikan gula darah seringkali disebabkan oleh stress
E. Setiap orang hendaknya dibekali oleh keempat ketrampilan berbahasa.

11. SIMAK UI 2009


Penggunaan tanda baca yang BENAR terdapat pada kalimat…………..
A. Akhirnya, setelah lama ditunggu, lahirlah sang putera mahkota.
B. Nah, oleh karena itu kamu harus segera memilikinya.
C. Jadi setiap kebaikan, sekecil apa pun, pasti akan berbuah kebaikan pula.
D. Ada tiga komponen latihan kebugaran yaitu: pemanasan, inti, dan pendinginan.
E. Jangan-jangan dia salah sangka atas tindakanmu itu.

12. SIMAK UI 2009


Sebagai seorang profesional, seorang guru harus memiliki pengetahuan dan persediaan strategi pembelajaran
yang memadai.

Kalimat di atas merupakan perluasan dari kalimat...


A. Sebagai seorang profesional.
B. Pengetahuan dan persediaan strategi pembelajaran.
C. Seorang guru yang profesional
D. Seorang guru adalah seorang profesional
E. Seorang guru harus memiliki pengetahuan dan persediaan

13. SIMAK UI 2009


Keterampilan membawa kamus merupakan keterampilan yang penting dimiliki. Keterampilan itu akan sangat
membantu Anda dalam mencari dan menemukan arti kata secara cepat. Jika hal itu tidak dimiliki, bisa jadi Anda
akan menghabiskan waktu setengah jam atau lebih hanya untuk mencari arti sebuah kata.

Bidang Studi BAHASA INDONESIA 16


KUMPULAN SOAL – SOAL UJIAN MASUK PERGURUAN TINGGI NEGERI

Kalimat berikut yang tepat mengakhiri paragraf tersebut adalah…


A. Oleh karena itu, beli dan milikilah kamus agar mudah menemukan arti kata.
B. Dengan demikian keberadaan kamus di perpustakaan sekolah sangat penting.
C. Jadi, Anda perlu mengoleksi berbagai kamus agar mudah mencari arti kata.
D. Dengan demikian, milikilah kamus dan biasakan membaca kamus dengan cermat.
E. Oleh sebab itu, biasakanlah membaca kamus, secara tepat dan cepat.

14. SIMAK UI 2009


Kalimat yang baku terdapat dalam …..
A. Meskipun menggunakan hitungan waktu, hari, dan tahun yang sama, tetapi prosesi ritual selamatan orang
meninggal berbeda satu dengan yang lain.
B. Akan tetapi, mempelajari adat budaya Jawa asli secara lengkap tidaklah mudah karena begitu rumit dan
kompleks.
C. Dalam perkawinan antara suku Jawa dengan suku lain, tata cara Jawa sering diterapkan bergantian dengan
tata cara suku lain tersebut.
D. Dari sekitar 306 suku bangsa Indonesia, suku Jawa mencapai lebih dari separuh jumlah penduduk Indonesia.
E. Dan demi keselarasan, maka sebagai konsekuensi menjadi orang Jawa, ia harus akomodatif dan toleran.

15. SIMAK UI 2009


Kalimat yang efektif terdapat dalam…
A. Akibat dari perlakuan itu berangsur-angsur telah merubah fungsi keris sebagai senjata menjadi benda regalia
bertuah.
B. Mudah dipahami kalau masyarakat Jawa mengidentikkan keris dan berbagai senjata tradisi seperti tombak,
patrem, pedang, dan lain sebagainya sebagai tosan aji.
C. Tosan berarti besi atau baja.
D. Sedangkan aji berarti sangat berharga sehingga pantas dihormati.
E. Keris adalah merupakan produk budaya hasil dari perpaduan teknologi metalurgi dengan karya seni yang luar
biasa indah.

16. SIMAK UI 2009


Seorang empu yang benar-benar matang biasanya mampu menciptakan pamor keris yang baru yang idenya
diserap dari alam.

Gagasan pokok kalimat tersebut adalah..


A. empu yang benar-benar matang
B. empu mampu menciptakan pamor keris
C. ide penciptakan pamor keris diserap dari alam
D. penciptaan pamor keris
E. seorang empu

17. SIMAK UI 2009


Ada dua rumah besar bergaya istana tradisional khas Cina di Medan. Satu terletak di pusat kota. Satu lagi di
Kecamatan Palau Berayan. Kedua rumah tersebut adalah milik Tjong A Fie, miliuner Cina Medan.

Pernyataan-pernyataan berikut sesuai dengan kutipan diatas, kecuali…


A. Di Medan dan di Palau Berayan terdapat istana tradisional Cina.
B. Tjong A Fie memiliki rumah besar.
C. Tjong A Fie penduduk Medan.
D. Rumah Tjong A Fie sangat megah.
E. Tjong A Fie kaya raya

18. SIMAK UI 2009


Tjong A Fie adalah seorang kuli emigran asal Cina yang kemudian berhasil menjadi pemilik perkebunan,
industrialis, bankir, dermawan, dan pemimpin masyarakat Cina di Medan. Ia sangat disegani oleh penjajah
Belanda dan penguasa daratan Cina. la juga mitra dagang Sultan Deli dan sahabat orang-orang pribumi Medan
dan sekitarnya.
Pernyataan yang sesuai dengan kutipan di atas adalah…
A. Tjong A Fie pada mulanya seorang kuli.
B. Tjong A Fie adalah penguasa masyarakat Medan.
C. Tjong A Fie adalah orang yang menakutkan bagi Belanda.
D. Tjong A Fie dihormati penjajah dan pribumi.
E. Tjong A Fie mempunyai catatan panjang dalam hidupnya.

Bidang Studi BAHASA INDONESIA 17


KUMPULAN SOAL – SOAL UJIAN MASUK PERGURUAN TINGGI NEGERI

19. SIMAK UI 2009


Penggunaan tanda baca titik dua (: ) yang benar terdapat pada
(1) Para karyawan mempunyai tuntutan yang keras: direktur mundur atau dimundurkan.
(2) Mereka ingin memiliki kebebasan dasar seperti : bebas dart rasa takut den bebas untuk menentukan jalan
hidup.
(3) (Yani, 1987: 125-177)
(4) Para pejabat Pemda Kalteng mengatakan: relokasi itu sulit dilakukan dalam waktu dekat.

20. SIMAK UI 2009


Kalimat yang baku adalah…
(1) Walaupun banyak mendapat protes, namun pemerintah tetap akan menaikkan harga BBM.
(2) Walaupun banyak mendapat protes, pemerintah tetap akan menaikkan harga BBM.
(3) Walaupun banyak mendapat protes, tetapi pemerintah tetap akan menaikkan harga BBM.
(4) Pemerintah tetap akan menaikkan harga BBM, walaupun banyak mendapat protes.

SOAL BAHASA INDONESIA TAHUN 2008

UMB UI

1. UMB UI 2008
Sebagai Negara kepulauan dengan 65 persen penduduk yang tinggal di wilayah pesisir, pemanasan global yang
berdampak pada naiknya permukaaan laut harus dilihat sebagai ancaman yang serius bagi Indonesia. Sementara
itu, sebagai Negara agraris, produktivias pertanian negeri ini pun terancam tidak kalah serius akibat dampak
perubahan iklim.
Dari kutipan di atas dapat disimpulkan hal – hal berikut KECUALI
A. Pemanasan global mengancam bidang pertanian dan kekuatan di Indonesia.
B. Ancaman terbesar dari pemanasan global diihadapi oleh penduduk yang tinggal di wilayah pesisir.
C. Penduduk yang hidup dari pertanian tifak mengalami pemanasan global karena sudah mengalami perubahan
Iklim.
D. Naiknya permukaan laut dan perubahan iklim merupakan akibat dari pemanasan global.
E. Pemanasan global mengancam Indonesia sebagai Negara kepulauan dan Negara agraris.

2. UMB UI 2008
Selain dapat memberikan gambaran data secara efektif, penggunaan grafik juga dapat mempermudah
penyampaian ide yang kompleks. Cirri utama grafik adalah sederhana, tetapi jelas. Grafik memberikan gambaran
perbandingan atau gambaran asosiasi antara dua atau beberapa variable. Selain itu, grafik juga menggambarkan
dan mengikhtisarkan hubungan antara data statistic dengan bagian – bagian lain secara komprehensif, padat,
singkat, dan sederhana. Grafik merupakan bentuk penyajian visual yang dipakai untuk membandingkan jumlah dta
saat-saat yang berbeda.
Kesimpulan yang dapat diambil dari paragraf di atas ialah
A. Ide yang kompleks dapat disampaikan dengan singkat dan mudah melalui grafik.
B. Ciri utama grafik adalah sederhana, tetapi jelas.
C. Grafik merupakan gambaran perbandingan atau gambaran asosiasi antara dua atau lebih variable.
D. Grafik dapat memberi gambaran lebih efektif kepada orang lain.
E. Grafik merupakan penyajian visual yang dipakai untuk membandingkan jumlah data.

3. UMB UI 2008
Sebagaimana telah ditetapkan dalam GBHN, bahwa pembangunan pendidikan dititikberatkan pada peningkatan
setiap jenjang dan jenis pendidikan.
Kalimat di atas dapat diperbaiki dengan menghilangkan kata
A. sebagimana
B. telah
C. dalam
D. bahwa
E. pada

Bidang Studi BAHASA INDONESIA 18


KUMPULAN SOAL – SOAL UJIAN MASUK PERGURUAN TINGGI NEGERI

4. UMB UI 2008
Bila ada seorang individu yang memaksakan kebenarannya pada individu lain seharusnya system sosial yang ada
mengkritiknya. Kalau ada seorang individu yang memamerkan bahwa dia mampu melakukan apa saja dengan
uannya, seharusnya system sosial yang ada menyadarkannya bahwa itu belum dapat diterima. Kalau ada seorang
individu yang bersedia membeli sebatang pohon tanaman hias seharga 1,2 miliar, sebenarnya system sosial tidak
mendukungnya. Dengan begitu, system sosial yang ada akan menjadi semakin kua. Rasa kebersamaan akan benar-
benar dapat dirasakan oleh semua anggota masyarakat.
Isi ringkas paragraf tersebut adalah
A. Perilaku seorang individu seharusnya memperhatkan system sosial masyarakatnya.
B. System sosial harus menjadi satu-satunya tolak ukur dalam kehidupan bermasyarakat
C. System sosial hendaknya dapat berperan sebagai pengontrol dan pengendalian perilaku individu
D. Kepentingan individu merupakan bagian dari system sosial
E. System sosial merupakan pembentuk rasa kebersamaan antaranggota masyarakat

5. UMB UI 2008
Pendidikan merupakan usaha untuk menumbuhkan dan mengembangkan potensi-potensi pembawaan baik jasmani
maupun rohani sesuai dengan nilai-niilai yang ada dalam masyarakat dan kebudayaan. Pendidikan merupakan
kebutuhan mutlak yang harus dipenuhi sepanjang hidup . Tanpa pendidikan mustahil manusia dapat hidup
berkembang sejalan dengan aspirasi untuk maju, sejahtera, dan bahagia menurut konsep pandangan hidup
mereka.
Generalisasi yang tepat untuk melengkapi paragraf di atas adalah
A. Pemaknaan pendidikan sebagai salah satu pilar pendidikan bangsa tidak terlepas dari pembatasan pemaknaan
pendidikan itu sendiri.
B. Hanya melalui proses pendidikanlah, kita dapat berharap bagi kepentingan kemajuan peradaban bangsa ini.
C. Pendidikan adalah proses yang berisi berbagai macam kegiatan yang cocok bagi individu untuk kehidupan
sosialnya dan membantu meneruskan adat dan budaya serta kelembagaan sosial dari generasi ke generasi.
D. Oleh karena itulah, kita harus memperhatikan definisi pendidikan yang dikemukakan para ahli.
E. Pendidikan pada dasarnya adalah usaha sadar untuk mengembangkan kepribadian dan kemampuan di dalam
dan di luar sekolah dan berlangsung seumur hidup.

6. UMB UI 2008
Sepanjang 971 kilometer di Pantai Utara Jawa kini dalam keadaan memprihatinkan. Bahkan telah mengalami
degradasi fungsi. Khususnya ketika abrasi, reklamasi, dan pencemaran yang mengakumulasi, telah
mengakibatkan rusaknya ekosistem pantai.
Kata – kata yang tepat untuk mengganntikan kata-kata yang bercetak tebal dalam kalimat di atas adalah
A. Penyimpangan fungsi, pengerukan tanah pantai, lingkungan
B. Kegagalan fungsi, pengambilan alih tanah pantai, komponen lingkungan
C. Kemerosotan fungsi, pengurukan tanah pantai, keanekaragaman lingkungan
D. Pengalihan fungsi, tukar guling tanah pantai, fauna flora
E. Ketidaksamaan fungsi, pembangunan tanah pantai, sumber daya pantai

7. UMB UI 2008
Dalam perkembangannya, perbedaan paham yang sudah mengakar pada diri setiap insan tumbuh menjadi konflik
sehingga menodai pertemuan yang sudah dibangun bertahun-tahun
Kalimat inti dari kalimat luas tersebut adalah
A. Paham menjadi konflik
B. Paham sudah mengakar
C. Perbedaan tumbuh
D. Perbedaan menodai pertemuan
E. Perbedaan sudah mengakar

8. UMB UI 2008
Regulasi yang memihak konsumen dan pengembangan serta pembangunan insfrastruktur dan utilitas bagi
perumahan diharapkan segera diwujudkan
Pada kata yang tepat untuk kata regulasi dalam kalimat tersebut adalah
A. Pengaturan
B. Peraturan
C. Aturan
D. Keteraturan
E. Keberaturan

9. UMB UI 2008

Bidang Studi BAHASA INDONESIA 19


KUMPULAN SOAL – SOAL UJIAN MASUK PERGURUAN TINGGI NEGERI

Walaupun sudah berkali-kali diperingatkan oleh pemerintah, penduduk desa tidak mau meninggalkan daerah yang
sudah dinyatakan tidak aman itu.
Pernyataan yang tepat untuk menggambarkan tindakan pemerintah dalam kalimat di atas adalah
A. Menegakkan benang basah
B. Berkering air liur
C. Menggantang asap
D. Berlidah di lidah orang
E. Membuang bunga terjerat

10. UMB UI 2008


Meskipun terkecil dan terletak di ujung utara Benua Afrika, Tunisia masih menyimpan banyak kekayaan, mulai
dari pesisir pantai Laut Mediterania sepanjang 1.400 kilometer hingga Gurun Sahara yang kerap menjadi lokasi
syuting film seperti Star Wars dan The English Patient.
Inti kalimat panjang tersebut adalah
A. Tunisia terletak di ujung utara Benua Afrika
B. Letak Tunisia mulai dari pesisir pantai Laut Mediterania hingga Gurun Sahara
C. Tunisia menjadi lokasi syuting film
D. Benua Afrika kecil
E. Tunisia menyimpan kekayaan

11. UMB UI 2008


Kalimat yang merupakan kalimat baku adalah
A. Ternyata jamu tidak hanya terkenal di Indonesia saja, tetapi di Negara- Negara lain pun mengkosumsi jamu
B. Untuk Negara Asia di antaranya ada Cina dan Thailand, sedangkan untuk benua Eropa, jamu banyak
ditemukan di Eropa Barat dan Jerman
C. Sedangkan omzet produksi industri jamu Indonesia di dunia berjumlah 300 juta dolar AS
D. Penurunan nilai ekspor terjadi karena adanya isu bahwa pada obat-obatan herbal terdapat kandungan bahan
kimia berbahaya
E. Dan masalah kwalitas jamu yang mengandung campuran kimia kini dipermasalhkan oleh Negara importir
karena ada yang meninggal setelah mengkonsumsi jamu tersebut.

12. UMB UI 2008


Sejak lahirnya konsep pemikiran baru dalam ilmu kedokteran yang dicetuskan oleh Profesor Linus Pauling, yakni
tentang Ortomolecular medicine yang dasarnya adalah studi biologi molekuler sebagai sains dasar, penelitian
medis diarahkan pada molekul-molekul yang secara normal biologis fisiologis ada dalam tubuh manusia.
Inti kalimat panjang tersebut adalah
A. Konsep pemikiran baru dicetuskan oleh Profesor Linus Pauling
B. Ortomolecular medicine adalah sains dasar
C. Ortomolecular medicine dasarnya adalah studi biologi
D. Penelitian medis diarahakan pada molekul
E. Biologi molekuler merupakan sains dasar

13. UMB UI 2008


Di antara kalimat berikut, yang merupakan kalimat baku adalah
A. Untuk membangun kampong wisata, maka masyarakat harus menjadi actor kunci atau perkasa.
B. Untuk memaknai kehidupan, peradaban manusia selalu ditandai dengan perubahan.
C. Klien mengajak Talia dan Francesca berlibur ke Panama, dimana konglomerat itu memiliki vila.
D. Bersahabat dengan teknologi di era informasi, memang menjadi sebuah tuntutan
E. Musim penghujan harga sejumlah komoditi sayuran di beberapa daerahmelambung.

14. UMB UI 2008


Ada ribuan TKI yang terpaksa dipulangkan dari Malaysia karena mereka masuk ke Negara itu secara ilegal.
Kalimat di atas berasal dari kalimat inti
A. Ribuan TKI dipulangkan
B. Ribuan TKI masuk ke Malaysia
C. Ada ribuan TKI
D. Ribuan TKI terpaksa
Bidang Studi BAHASA INDONESIA 20
KUMPULAN SOAL – SOAL UJIAN MASUK PERGURUAN TINGGI NEGERI

E. Ribuan TKI illegal

15. UMB UI 2008


Ikatan Komunitas Literasi Indonesia (IKLI) membuat system jaringan informasi di antara sesame komunitas
sehingga kelak aka nada pertukaran informasi seputar metode kegiatan dan jaringan bantuan serta memperkuat
posisi tawar komunitas ke berbagai pihak.
Kalimat inti dari kalimat luas di atas adalah
A. Ikatan Komunitas Literasi Indonesia membuat system jaringan informasi
B. Ikatan Komunitas Literasi Indonesia ada pertukaran informasi
C. Ikatan Komunitas Literasi Indonesia memperkuat posisi
D. Ikatan Komunitas Literasi Indonesia memperkuat komunitas
E. Pertukaran informasi memperkuat posisi tawar komunitas

16. UMB UI 2008


Kalimat berikut ini yang termasuk kalimat baku adalah
A. Sarana dan prasarana yang disediakan oleh pihak sekolah itu untuk memfasilitasi siswa yang aktif dan kreatif
agar dapat mengembangkan minat bakat mereka
B. Pada minggu kedua pertemuan kita akan membicarakan tentang pola-pola sinkretisme dan akulturasi budaya
yang terjadi di nusantara.
C. Jika disampaikan secara lebih sistematis dengan ilustrasi yang memadai, maka brosur ini akan memberikan
sugesti yang lebih kuat pada pembacanya.
D. Sebelum presiden memberikan kesempatan kepada hadirin untuk bertanya, maka terlebih dahulu meminta
seorang stafnya memandu jalanya Temu Wicara.
E. Kalau dibandingkan dengan situasi politik masa lalu, praktik KKN, yakni kecenderungan perilaku korupsi,
kolusi, dan nepotisme oleh jajaran birokrasi, saat ini, mungkin sudah sangat jauh berkurang.

17. UMB UI 2008


Tindakan anarkis itu sebagai suatu manifestasi kemarahan hatinya.
Padanan kata manifestasi yang tidak tepat adalah
A. Pengejawantahan
B. Perwujudan
C. Pengungkapan
D. Perlakuan
E. Pernyataan

18. UMB UI 2008


Penulisan bilangan dalam kalimat yang sesuai dengan aturan EYD adalah
A. Anda dapat membeli buku ini seharga Rp50.000,- ditoko saya.
B. Sebanyak Sembilan puluh enam kelurahan di Jakarta rawan demam berdarah dengue (DBD)
C. Pinjaman asosiasi pembangunan internasional biasanya diberikan dengan jangka waktu tiga lima tahun, masa
tenggang sepuluh tahun, dan tingkat suku bunga tetap 0,75% per tahun.
D. Ratusan orang yang berunjuk rasa mengaku perwakilan warga Dayak dari 13 kabupaten dan kota di
Kalimantan Timur
E. Harga kedelai impor naik sejak pekan lalu dari Rp6.500 per kilogram menjadi Rp7.000

19. UMB UI 2008


Kata bercetak miring yang huruf awalnya tidak perlu ditulis dengan huruf capital terdapat dalam kalimat
A. Sampah menumpuk di Pintu Air Manggarai, Jakarta Selatan, setelah mendapat banjir kiriman dari Bogor
B. Warga Kampung Pulo , kelurahan Kampung Melayu Jakarta Timur, menyingkirkan berbagai barang yang
terapung di dapur rumah mereka
C. Antrean truk pengangkut barang terjadi akibat salah satu dermaga di Pelabuhan Bakauheni, Lampung rusak
D. Pemerintah provinsi daerah khusus ibu kota Jakarta menguji coba layanan perpanjangan KTP melalui telepon
genggam
E. Alat berat dikerahkan untuk mengeruk badan Sungai Krukut di Daerah penjernihan, Jakarta Selatan

20. UMB UI 2008


Penggunaan tanda koma (,) yang salah dijumpai dalam kalimat.
A. Sebetulnya, pembangunan rumah sehat sederhana memiliki multi player effect yang lebih luas.
B. Sasaran pelatihan adalah para pimpinan perusahaan, bagian promosi public relations, humas, dan bagian lain
yang memiliki kompentensi dengan public.
C. Hanan mengharapakan, agar banyak pengembang ikut berperan dalam pembangunan rumah sehat sederhana.
Bidang Studi BAHASA INDONESIA 21
KUMPULAN SOAL – SOAL UJIAN MASUK PERGURUAN TINGGI NEGERI

D. Meskipun untung yang diperoleh tidak cukup besar, pembangunan rumah sehat sederhana ini memiliki
manfaat yang lebih besar.
E. Harga ikan di tempat pelelangan ikan Tanggulangin, Kecamatan Klirong, Kabupaten Kebumen, dikeluhkan
nelayan setempat.

21. UMB UI 2008


Kata serapan yang salah penulisannya terdapat dalam kalimat
A. Pembangunan fisik tanpa pembangunan mental seumpama bangunan tanpa pondamen
B. Basket adalah olahraga favoritnya
C. Perjanjian antara dua pihak disebut perjanjian bilateral
D. Gadis itu menjadi sangat ideal untuk menjadi peragawati
E. Aku jarang pergi ke bioskop

22. UMB UI 2008


Penulisan kalimat berikut ini mengikuti aturan EYD, KECUALI
A. Buku ini disusun untuk membantu mahasiswa yang akan mengambil mata kuliah Akuntansi Perusahaan pada
fakultas ekonomi.
B. Kami menulis diktat ini dengan maksud, agar mahasiswa lebih mudah mengikuti kuliah tentang ekonomi
Pembangunan.
C. Akuntansi terbagi dalam beberapa bidang sebagai berikut: akuntansi sosial, akuntansi pemerintahan, akuntansi
perusahaan, dan akuntansi organisasi nonprofit.
D. Diharapkan diktat yang berjudul Akuntansi Sosial ini bermanfaat bagi para peminat akuntansi pada umumnya
dan mahasiswa sekolah tinggi ilmu ekonomi pada khususnya.
E. Pengelasan dengan asetilena dilakukan dengan cara membakar bahan gas asetilena dengan O2.

23. UMB UI 2008


Penulisan kata serapan yang sesuai dengan aturan EYD adalah
A. Perlu adanya standarisasi dalam mutu produksi sabun.
B. Agaknya telah terjadi marginalisasi peran terhadap kelompok tertentu.
C. Perubahan yang positif sangat diharapkan entah dalam bentuk material ataupun non material.
D. Keterampilan managerial yang tinggi sangat diperlukan bagi setiap pemimpin.
E. Respon masyarakat terhadap rencana perbaikan kampung sangat baik.

24. UMB UI 2008


Anak yang haus kasih sayang biasanya akan mencari perhatian gurunya.
Proses pembentukan kata yang sama dengan kasih sayang terdapat dalam kalimat
A. Cucunya dibelainya dengan penuh kasih mesra.
B. Wanita itu mencintai darah dagingnya sendiri.
C. Dengan gagah berani ia menghadapi musuh-musuhnya.
D. Kami berdoa untuk keluarga kami di kampung halaman.
E. Ia mengungkapkan perasaannya dengan terus terang.

25. UMB UI 2008


Ombak tinggi dan angin kencang kembali …penyeberangan antarpulau, Merak – Bakauheni. Cuaca buruk yang
kerap … Selat Sunda satu bulan terakhir membuat perusahaan kapal cepat khusus penumpang… hingga miliaran
rupiah dan … gulung tikar.
A. Mengganggu, melanda, merugi, terancam.
B. Diganggu, dilanda, rugi, mengancam
C. Terganggu, terlanda, kerugian, diancam
D. Gangguan, melanda, dirugikan, ancaman
E. Mengganggu, terlanda, dirugikan, mengancam

UM UGM

1. UM UGM 2008
Investasi perlu didorong ke luar Jawa. Selain kekayaan sumber daya alamnya, investasi di luar Jawa juga lebih
menarik karena daya beli rata-rata masyarakat di luar Jawa sebenarnya lebih tinggi dibandingkan dangan daya
beli rata-rata penduduk Pulau Jawa. Namun, keterbatasan infrastruktur di luar Pulau Jawa masih menjadi
penghambat pengembangan investasi di luar sektor.

Bidang Studi BAHASA INDONESIA 22


KUMPULAN SOAL – SOAL UJIAN MASUK PERGURUAN TINGGI NEGERI

Dalam bacaan di atas tersirat bahwa


A. selama ini investasi lebih terpusat di Pulau Jawa.
B. belum ada investasi di luar Pulau Jawa.
C. investasi di Pulau Jawa sudah tidak menguntungkan lagi.
D. investasi di luar Pulau Jawa akan mendatangkan keuntungan besar.
E. investasi di luar Pulau Jawa terhambat oleh keterbatasan infrastruktur.

2. UM UGM 2008
Selain masih minim dan kurang terurusnya fasilitas, persoalan toilet umum yang kotor di kota berakar pada
masalah budaya masyarakat. Dalam budaya masyarakat Indonesia yang agraris, masyarakat pada zaman dahulu
pernah punya kebiasaan buang hajat di alam terbuka, seperti sungai, empang, hutan, atau semak belukar di
belakang rumah, tanpa harus membersihkannya. Kotoran dibiarkan saja karena bisa langsung diurai organisme
dalam tanah, tumbuhan, atau air. Di kota orang harus buang hajat di toilet, tetapi kebiasaan lama masih dibawa.
Buktinya, masih banyak pengunjung toilet toilet umum pergi begitu saja tanpa menyiram atau membersihkan
peturasannya.
Topik paragraf di atas adalah
A. persoalan toilet umum.
B. budaya masyarakat Indonesia.
C. organisme dalam tanah.
D. kebiasaan lama orang dalam buang hajat.
E. pengunjung toilet umum.

3. UM UGM 2008
Satu pelajaran penting yang dapat di ambil dari pelbagai macam upaya penyelesaian konflik Palestina dan Israel
adalah bahwa konferensi atau perundingan apa pun tidak mampu mengubah warna jalan-jalan di TimurTtengah.
Tatkala perdamaian menderu kencang di ruang konferensi, suara tembakan dan kekerasan acap kali mewarnai
jalan-jalan di kawasan itu. Persoalan konflik Palestina dan Israel bukan persoalan elite belaka. Persoalan ini
telah menyebar dan meluas menjadi persoalan masyarakat Timur Tengah secara umum.
Berdasarkan isi bacaan di atas, pernyataan berikut ini benar, kecuali
A. Konflik Palestina dan Israel bukan hanya persoalan kedua bangsa itu.
B. Dalam penyelesaian konflik Palestina dan Israel antara hasil perundingan dan kenyataan di lapangan sering
tidak sejalan.
C. Sampai saat ini belum ada perundingan yang dapat menyelesaikan konflik Pelestina dan Israel.
D. Hanya perundingan para elite kedua negara yang dapat menyelesaikan konflik Palestina dan Israel.
E. Masyarakat di Timur Tengah belum memiliki pandangan yang sama dalam menyelesaikan konflik Palestina
dan Israel.

4. UM UGM 2008
Pada tahun 2008 pertumbuhan ekonomi Amerika Serikat diperkirakan melambat menjadi 2% atau sedikit menurun
dibandingkan dangan tahun sebelumnya. Menurut IMF, sekitar 1% penurunan pertumbuhan ekonomi di Amerika
Serikat akan menurunkan pertumbuhan ekonomi di Asia sekitar 0,5 – 1%. Dampak resesi global yang berasal dari
resesi di Amerika Serikat akan mempengaruhi proyeksi perekonomian di Negara-negara asia, terutama Indonesia.
Ekonomi global diperkirakan akan mengalami penurunan pertumbuhan sebesar 0,4% dari 5,2% pada tahun 2007
menjadi 4,8% pada tahun 2008. Negara-negara di Asia Tenggara diperkirakan mengalami tekanan paling parah
akibat melambatnya ekonomi yang terjadi di Amerika.

Berdasarkan bacaan di atas, pernyataan berikut ini yang salah adalah


A. Penurunan pertumbuhan ekonomi di Amerika Serikat berdampak pada penurunan pertumbuhan ekonomi
global.
B. Pada tahun 2008 pertumbuhan ekonomi Amerika Serikat diperkirakan lebih rendah dari pada pertumbuhan
ekonomi global.
C. Pada tahun 2008 Amerika Serikat diperkirakan akan mengalami penurunan pertumbuhan ekonomi sebesar 1%.
D. Sebagai dampak penurunan pertumbuhan ekonomi Amerika Serikat, pertumbuhan ekonomi di Asia tidak akan
turun lebih besar dari pada besarnya angka penurunan pertumbuhan ekonomi di Amerika Serikat.

Bidang Studi BAHASA INDONESIA 23


KUMPULAN SOAL – SOAL UJIAN MASUK PERGURUAN TINGGI NEGERI

E. Jika pertumbuhan ekonomi Amerika Serikat turun 2%, sebagai dampaknya pertumbuhan ekonomi negara Asia
akan turun sebesar 1% - 2%.
5. UM UGM 2008
Kehidupan orang Malind, salah satu suku di Papua yang masih mengandalkan perburuan dan penokokan sagu
untuk mencari makanan, sangat dipengaruhi alam. Selama ribuan tahun kekayaan hutan ulayat Malind membuat
hidup mereka berkelimpahan. Sagu di mana-mana, saham(kanguru kecil) dan rusa berlarian, burung-burung
berkicau di sekitar kampung adat, dan ikan pun melimpah. Namun sekitar dua puluh tahun yang lalu, sebuah
sentakan budaya terjadi ketika lebih dari 48.000 hektare hutan ulayat dibuka menjadi sawah bagi para
transmigran.
Dari bacaan di atas dapat disimpulkan bahwa
A. Kehidupan suku Malind telah berubah dari sistem meramu dan berburu ke bercocok tanam.
B. Pembukaan hutan ulayat menjadi sawah telah mempengaruhi kehidupan suku Malind.
C. Kesejahteraan suku Malind semakin meningkat sejak dibukanya hutan ulayat menjadi sawah.
D. Sejak dibukanya hutan ulayat menjadi sawah, suku Malind tidak lagi dapat berburu.
E. Suku Malind belajar bercocok tanam di sawah dari para transmigran yang datang ke daerah itu.

6. UM UGM 2008
Terbatasnya lahan dan modal usaha di tengah semakin tingginya tuntutan kebutuhan hidup acap kali justru
mendorong kreativitas. Tidak pernah puas dangan hasil kerja yang sama bertahun-tahun, terobosan-terobosan
baru pun dilakukan untuk memperoleh hasil yang lebih baik. .… Mereka berhasil mengembangkan budi daya
ternak itik unggul organik yang merupakan persilangan dari itik alabio Kalimantan Timur dangan itik Mojosari
Jawa Timur.
Kalimat yang tepat untuk melengkapi titik-titik dalam bacaan di atas adalah
A. Para peternak itik di wilayah itu membuat penemuan baru.
B. Demikianlah yang terjadi pada para peternak itik di wilayah itu.
C. Dua jenis itik disilangkan oleh para peternak itik di wilayah itu untuk mendapatkan varietas itik baru.
D. Di wilayah itu para peternak itik menunjukkan kreativitasnya.
E. Itulah sebabnya para peternak itik di wilayah itu menunjukkan kreativitasnya.

7. UM UGM 2008
Asam folat selama ini dikenal luas sebagai suplemen wajib pada masa prakehamilan dan kehamilan …. timbul
anggapan bahwa zat ini hanya dibutuhkan oleh para calon ibu. …, baik pria maupun wanita sama-sama
membutuhkan asupan harian asam folat. …, kebutuhan harian asam folat pria dewasa sama besarnya dangan
kebutuhan wanita dewasa.
Kata yang tepat untuk mengisi tiga bagian kosong dalam bacaan di atas berturut-turut adalah
A. lalu, oleh katena itu, sedangkan.
B. dan, walapun, adalapun.
C. sehingga, padahal, bahkan.
D. meskipun, akan tetapi, jadi.
E. karena, sebenarnya, dangan demikian.

8. UM UGM 2008
(1) Jumlah ini meningkat tajam dari tahun sebelumnya yang hanya tiga kasus. (2) Menurut Direktur LBH
Palembang, Eti Gustina, pada tahun 2007 terjadi 39 kasus penyerobotan tanah milik warga di Provinsi Sumatra
Selatan. (3) Demikian salah satu kesimpulan refleksi akhir tahun LBH Palembang. (4) Konflik ini diantaranya
terjadi di Kabupaten Musi Banyuasin dan Ogan Komering Ilir. (5) Lembaga Batuan Hukum Palembang mencatat
peningkatan ketegangan akibat kasus penyerobotan tanah milik warga oleh sejumlah perusahaan perkebunan
tahun 2007.
Untuk menjadi sebuah paragraf yang runtut, kalimat-kalimat di atas harus disusun menjadi:
A. 1 – 2 – 4 – 3 – 5
B. 2 – 4 – 3 – 1 – 5
C. 5 – 1 – 3 – 4 – 2
D. 2 – 3 – 4 – 1 – 5
E. 5 – 4 – 3 – 2 – 1

9. UM UGM 2008
Untuk membangun tanggul penahan banjir, pemerintah akan merelokasi warga yang tinggal di bantaran sungai.
Yang dimaksud dangan bantaran dalam kalimat di atas adalah
A. bagian hilir.
B. bagian hulu.
C. bagian sisi kanan dan kiri.
D. bagian yang tidak dialiri air.
E. bagian lereng tebing.
Bidang Studi BAHASA INDONESIA 24
KUMPULAN SOAL – SOAL UJIAN MASUK PERGURUAN TINGGI NEGERI

10. UM UGM 2008


Kata yang mengalami perubahan makna karena asosiasi terdapat dalam kalimat
A. Persentase sarjana yang belum mendapatkan pekerjaan semakin bertambah.
B. Para pegiat perempuan semakin berani memperjualkan hak-haknya.
C. Karena kesantunannya, oleh temannya dia disebut sebagai pendeta.
D. Sesekali dia melontarkan kritikan pedas terhadap kebijakan atasannya yang dianggapnya perlu.
E. Pada waktu melaksanakan tugas jurnalistiknya, dia selalu berusaha menolak pemberian amplop dalam setiap
peristiwa yang diliputnya.

11. UM UGM 2008


Kalimat Kegiatan ekoturisme untuk mngkonservasi alam sering tidak sejalan dangan kebiasaan masyarakat
setempat memiliki pola yang sama dangan kalimat
A. Banyak pegiat ekoturisme mengakui bahwa kesalahan yang sering terjadi dalam mengembangkan ekoturisme
adalah pengemasan yang semata-mata mengikuti selera masyarakat.
B. Pembuatan patung kayu komodo pasti akan menghabiskan pepohonan yang sekarang sudah jarang.
C. Larangan pengambilan kayu waru laut di Pulau Komodo dilatarbelakangi oleh terbatasnya populasi vegetasi di
sana.
D. Haji Zainun terus membuat patung komodo di sela-sela kesibukannya sebagai nelayan.
E. Keterampilan membuat patung komodo merupakan sumber mata pencaharian baru warga Pulau Komodo.

12. UM UGM 2008


Kalimat yang tidak baku adalah
A. Sesuai dangan kesepakatan, pertunjukan itu akan diadakan di Jawa dan Sumatra.
B. Masa yang akan datang kenaikan yang tidak terkendali dari harga kebutuhan sehari-hari tidak boleh terjadi
lagi.
C. Ramalan BMG semakin kita perhatikan, terutama setelah muncul berbagai dampak tak terduka dari perubahan
iklim.
D. Di negara-negara bermusim empat, ramalan cuaca dan geofisika sudah lama menjadi bagian dari kehidupan
sehari-hari.
E. Untuk keperluan pelestarian lingkungan diperlukan tanggungjawab kolektif dan antisipatif dari para pengusaha
yang bergerak di bidang pengusahaan hutan.

13. UM UGM 2008


Kalimat Tugas utama pemerintah adalah memperhatikan kepentingan masyarakat kelas bawah mempunyai pola
yang sama dangan kalimat
A. Tingginya angka inflasi akan berpengaruh terhadap masyarakat kelas bawah.
B. Oligarki (kekuasaan di tangan segelintir orang) dan plutokrasi (pemerintahan oleh sekelompok orang kaya)
menjadi penumpang gelap sistem demokrasi.
C. Tidak satu pun partai politik di Indonesia terbebas dari sindrom oligarki dan plutokrasi.
D. Sistem kepartaian yang sederhana dapat mengerem pertumbuhan oligarki dan plutokrasi.
E. Partai politik dapat menempatkan kadernya untuk mengisi jabatan publik sesuai dangan kapasitas dan
kapabilitasnya.

14. UM UGM 2008


Kalimat yang baku adalah
A. Untuk mengatasi banjir Bengawan Solo, penanaman pohon disekitar Waduk Gajah Mungkur dan sepanjang
bengawan tidak dapat ditawar lagi.
B. Sedangkan penanganan jangka pendek dangan memperbaiki tanggul sepanjang bengawan yang rusak.
C. Juga dangan membuat bendungan kecil di puluhan titik sepanjang bengawan.
D. Di sepanjang bengawan menemukan lebih dari 20 tempat yang potensial untuk dibangun waduk.
E. Jika di tempat tersebut dibangun waduk kecil-kecil, akan bermanfaat mengurangi banjir.

15. UM UGM 2008


Kalimat berikut baku, kecuali
A. Pesan terpenting dari sejarah adalah pesan tentang kesalahan.
B. Pesan itu belum kami terima sampai sekarang.
Bidang Studi BAHASA INDONESIA 25
KUMPULAN SOAL – SOAL UJIAN MASUK PERGURUAN TINGGI NEGERI

C. Media massa ramai mengulas temuan rekening liar.


D. Dibutuhkan aturan yang jelas untuk melaksanakan pilkada ulang.
E. Indonesia kehilangan julukannya sebagai zamrud khatulistiwa.

16. UM UGM 2008


Tidak dapat disangkal bahwa kebanyakan orang pada saat ini lebih memilih menggunakan kamera yang ada pada
telepon genggam daripada secara khusus membeli kamera biasa karena kamera analog dianggap kurang praktis.
Inti kalimat tersebut adalah
A. Disangkal.
B. Orang memilih kamera telepon genggam.
C. Orang menggunakan kamera.
D. Orang membeli kamera.
E. Kamera analog tidak praktis.

17. UM UGM 2008


Kalimat Dalam pergaulan sehari-hari sering terlihat ada orang yang tidak dapat menerima saran orang lain
memiliki pola yang sama dangan kalimat
A. Dangan terbata-bata dikisahkannya peristiwa menyedihkan yang menimpa dirinya itu.
B. Disekitar kita ada orang yang mendorong dirinya untuk membaca evaluasi dan menghadiahi dirinya sendiri
jika hasilnya bagus.
C. Karena hampir setiap individu berinisiatif untuk berkembang dan berubah, tidak perlu dikumandangkan
gerakan perubahan.
D. Agar dapat terus bekerja di perusahaan itu, diikutinya berbagai kursus keterampilan.
E. Dalam lingkungan yang terbatas disampaikannya pesan yang agak rahasia itu.

18. UM UGM 2008


Penurunan cukai impor beras menjadi Rp. 450,- per kilogram, per 1 Januari 2008 dimaksudkan agar Bulog dapat
mengendalikan harga beras.
Kalimat tersebut akan benar ejaannya apabila
A. Rp.450,- ditulis Rp 450,00.
B. per kilogram ditulis perkilogram.
C. per 1 januari 2008 ditulis per-1 januari 2008.
D. impor ditulis import.
E. sebelum kata agar ditambahkan tanda koma (,).

19. UM UGM 2008


Kalimat yang penulisannya mengikuti EYD adalah
A. Sepuluh tahun yang lalu Keluarga Satrio tinggal di jalan RE Martadinata, No.5.
B. Sebagian besar dari kita beranggapan bahwa kebudayaan Timur lebih halus jika dibandingkan dangan
kebudayaan Barat.
C. Ia mengakui ke dalaman wawasan tokoh masyarakat kampung Naga itu.
D. Pengusaha besar itu menghadiahi atlit-atlit berprestasi internasional dangan bea siswa untuk belajar di luar
negeri.
E. Beberapa tokoh anti rasisme dari beberapa negara sepakat untuk menanda tangani suatu kesepahaman.

20. UM UGM 2008


Untuk kurangi pemanasan global maka masyarakat diminta untuk meminimalisir pemakaian kendaraan bermotor.
Kalimat di atas akan menjadi kalimat baku apabila diperbaiki dangan cara berikut, kecuali
A. Mengganti kata untuk pada awal kalimat dangan dalam.
B. Mengubah kata kurangi menjadi mengurangi.
C. Menambahkan tanda koma (,) sesudah kata global.
D. Menghilangkan kata maka.
E. Mengubah kata meminimalisir menjadi meminimalkan.

SNMPTN

1. SNMPTN 2008
Asumsi nilai moral budaya bangsa kita menjulang amat tinggi, melangit dan amat luhur, seperti tercermin dalam
nilai-nilai Pancasila. Meskipun demikian, dalam kenyataanya, asumsi moral yang amat hebat tersebut tidak
berpijak di bumi kenyataan karena hanya mengambang di awang-awang dan tidak ada kaitanya dengan kehidupan
Bidang Studi BAHASA INDONESIA 26
KUMPULAN SOAL – SOAL UJIAN MASUK PERGURUAN TINGGI NEGERI

nyata di tengah kehidupan masyarakat. Diantara Pancasila dan kehidupan nyata sehari-hari bangsa terdapat
jurang yang luas . Orang tidak merasakan bahwa nilai-nilai Pancasila menjiwai seluruh tingkah laku dan sikap
hidup setiap anggota masyarakat kita, baik yang berada dalam dunia birokrasi maupun diluar birokrasi.
Istilah yang paling tepat untuk menggambarkan adanya kesenjangan antara asumsi moral dan kenyataan sehari-hari
sebagaimana diilustrasikan dalam paragraf di atas adalah
A. kontradiksi
B. ironi
C. distorsi
D. persepsi
E. kontraindikasi

2. SNMPTN 2008
Pandai membaca peluang dasar dan tekun merupakan kunci untuk meraih sukses bagi mereka yang menjalani
profesi sebagai wiraswasta .Prinsip utama dalam dunia bisnis harus dipegang teguh, antara lain jangan takut
gagal, tetapi mempunyai keberanian untuk mengambil langkah dan bebaskan pikiran akan bertemu dengan
kesalahan. Dengan bekal tekat itulah Giyono pemuda di lereng Gunung Merapi berhasil merintis bisnis bibit
sayuran. Saat ini pun telah memiliki lebih dari sepuluh anak buah yang membantu usaha pembibitanya.
Simpulan yang dapat diambil dari paragraf diatas adalah
A. Giyono pemuda yang sukses dalam berwirausaha pembibitan sayuran.
B. Kunci kesuksesan terletak pada pandai membaca peluang pasar
C. Prinsip-prinsip dasar dalam dunia bisnis
D. Usaha pembibitan giyono telah berhasil membuka lahan kerja di wilayahnya
E. Ketekunan Giyono dapat menjadi teladan bagi pemuda lainnya.

3. SNMPTN 2008
Suhu dan derajat keasaman menjadi indikator konsentrasi bahan-bahan kimia berbahaya yang terlarut dalam air
sungai.
Kata indikator dalam kalimat tersebut sama maknanya dengan kata
A. Penentu
B. Petunjuk
C. Pembentuk
D. Pengukur
E. Penyebab

4. SNMPTN 2008
(1) Burung termasuk dalam jenis unggas yang bisa terbang . (2) Namun, ada burung yang tidak bisa terbang. (3)
Contohnya burung untuk (struthio camelus). (4) Burung unta adalah burung terbesar yang masih hidup dan tidak
bias terbang (5) Dahulu, burung ini terdapat dalam jumlah besar di Afrika dan Asia Barat Daya. (6) Sekarang ,
burungnya ini hanya ditemukan di beberapa daerah di Asia Timur. (7) Ada juga burung unta yang hidup di
peternakan di Afrika Selatan. (8) Sementara itu, di Australia Selatan, ada burung unta yang sudah dijadikan
hewan piaraan. (9) Seperti halnya burung lain yang tidak bisa terbang, burung unta sudah terbiasa pada
kehidupan di tanah dan dapat berlari dengan baik. (10) Dalam hal ini ayam tidak bisa disamakan dengan burung
unta.
Paragraf di atas akan merupakan paragraf yang memiliki kepaduan jika….
A. Kalimat (2) dan (3) digabungkan.
B. Kalimat (8) dan (9) dihilangakn.
C. Kalimat (10) dihilangkan
D. Kalimat (7) dihilangkan
E. Kalimat (7), (8) dan (9) digabungkan.

5. SNMPTN 2008
Otak manusia ibarat sebuah pisau. Otak manusia yang cerdas tidak akan berprestasi tinggi bila tidak belajar dan
berlatih. Otak manusia yang IQ-nya sedang-sedang saja akan mendapat prestasi gemilang bila belajar terus
menerus.Demikan pula dengan pisau. Sebilah pisau yang tajam akan menjadi tumpul bila tidak diasah.
Sebaliknnya sebilah pisau yang tumpul akan tajam bila diasah terus-menerus. Dengan demikian, bila kita ingin
menjadi manusia yang berprestasi hendaknya seperti pisau yang diasah terus.
Hal yang dianalogikan dalam paragraf di atas adalah
A. prestasi dan pisau
Bidang Studi BAHASA INDONESIA 27
KUMPULAN SOAL – SOAL UJIAN MASUK PERGURUAN TINGGI NEGERI

B. otak manusia dengan pisau


C. berlatih dengan diasah
D. manusia dengan pisau
E. kepandaian dengan ketajaman.

6. SNMPTN 2008
Seluruh rakyat mempunyai visi yang sama mengenai perjuangan bangsa.
Arti kata visi dalam kalimat tersebut adalah
A. pandangan ke depan
B. kemampuan untuk melihat pada inti persoalan
C. ketajaman penglihatan melalui kehalusan jiwa
D. apa yang tampak dalam khayalan
E. pengamatan

7. SNMPTN 2008
Di Yayasan Galuh, jumlah pasien usia remaja justru mendominasi, mulai dari remaja putus sekolah, remaja yang
masih sekolah, remaja yang tidak diurusi keluarga, hingga remaja yang memiliki obsesi tinggi, tetapi tidak
kesampaian.
Kalimat inti dari kalimat luas tersebut adalah
A. Jumlah pasien usia remaja mendominasi.
B. Pasien usia remaja memilki obsesi
C. Obsesi remaja tidak kesampaian.
D. Yayasan Galuh didominasikan oleh berbagai masalah remaja.
E. Mulai remaja putus sekolah sampai remaja obsesi tinggi ada di Yayasan Galuh.

8. SNMPTN 2008
Adanya disekuilibrium sosial budaya dapat mengganggu jalannya pembangunan.
Padan kata yang tepat untuk kata disekuilibrium dalam kalimat di atas adalah
A. Ketidaksinambungan.
B. Ketidakseimbangan
C. Ketidaksamaan
D. Keberlainan
E. Keberagaman

9. SNMPTN 2008
Rancangan Undang-Undang Badan Hukum Pendidikan yang tampaknya akan disahkan dalam waktu dekat ini
benar-benar menjadi instrumen hukum yang akan melegalisasi segala sesuatu yang bersifat internasional.
Kata instrumen dalam kalimat tersebut digunakan dengan makna
A. pedoman
B. dokumen resmi
C. sarana penelitian
D. patokan
E. alat

10. SNMPTN 2008


Kematian tokoh oposisi Benazir Bhuto ketika sedang berkampanye itu menyebabkan kerusuhan hebat yang
memakan banyak korban.
Kalimat di atas berasal dari kalimat inti
A. Kerusuhan menyebabkan
B. Benazir Bhuto berkampanye
C. Kerusuhan memakan korban
D. Kematian Benazir Bhuto
E. Benazir Bhuto menyebabkan kerusuhan

11. SNMPTN 2008


Diantara kalimat-kalimat berikut, yang bukan kalimat baku adalah
A. Dengan pendidikan diharapkan dapat membentuk manusia Indonesia yang demokratis, berkeadilan, dan tidak
diskriminatif.
B. Bahan ajar yang berperspektif gender sebaiknya dimulai sejak dini, mulai tingkat pendidikan dasar.
C. Ketika diwawancarai media Sarkozy mengatakan bahwa peningkatan kehadiran Prancis di Afganistan bukan
kuantitatif, melainkan kualitatif
Bidang Studi BAHASA INDONESIA 28
KUMPULAN SOAL – SOAL UJIAN MASUK PERGURUAN TINGGI NEGERI

D. Beberapa jam sesudah menerima Sarkozy, Karzai menerima kunjungan Rudd.


E. Berdasarkan laporan Dinas Pertanian Jawa Tengah, setiap dua bulan sekali dibuka produk pertanian Jawa
Tengah.

12. SNMPTN 2008


Setelah gas Pertanian di Porong meledak dan menewaskan serta melukai beberapa orang, pemerintah menyebut
luapan lumpur sebagai disaster.
Makna kata disaster dalam kalimat tersebut adalah
A. kecelakaan
B. musibah
C. bencana
D. kelalaian
E. kecerobohan

13. SNMPTN 2008


Dengan adanya bank syariah diharapkan dapat mendukung pengembangan ekonomi nasional, memfasilitasi
segmen pasar yang belum terjangkau atau tidak terminat dengan bank konvesional, dan dapat memfasilitasi
distribusi utilitas barang modal untuk kegiatan produksi melalui skema sewa-menyewa.
Ketidakbakuan kalimat tersebut terletak pada
A. Kurangnya tanda koma (,) sebelum kata diharapkan.
B. Pemakaian kata dengan
C. Gabungan kata bank syariah yang ditulis dengan huruf kecil
D. Pemakaian pengembangan yang seharusnya perkembangan
E. Pemakaian tanda koma, (,) sebelum kata dan

14. SNMPTN 2008


Silang pendapat yang terjadi antara Depertemen Kesehatan dan Badan Pengawasan Obat (POM) dalam
penanganan kusus formalin secara tidak langsung menggeser isu pokok yang berpihak pada kepentingan
konsumen, yaitu mendapatkan produk makanan yang aman bagi kesehatan.
Kalimat inti dari kalimat luas di atas adalah
A. Silang pendapat terjadi
B. Silang pendapat menggeser isu pokok
C. Kasus formalin berpihak pada kepentingan konsumen
D. Silang pendapat berpihak pada kepentingan konsumen
E. Kepentingan konsumen, yaitu mendapatkan produk makanan.

15. SNMPTN 2008


Untuk meningkatkan daya tahan tubuh, umumnya suplemen vitamin dikonsumsi, tetapi ternyata banyak pula
vitamin yang justru sangat berbahaya.
Kalimat di atas dikembangkan dari kalimat inti
A. Daya tahan tubuh meningkat
B. Meningkatkan daya tahan
C. Suplemen vitamin sangat berbahaya
D. Suplemen vitamin meningkat
E. Supplement vitamin dikonsumsi

16. SNMPTN 2008


Otonomi sekolah berperan dalam menampung konsensus umum tentang pemberdayaan sekolah yang menyakini
bahwa untuk meningkatkan kualitas pendidikan keputusan sedapat mungkin dibuat oleh mereka yang ada digaris
depan yang bertanggung jawab secara langsung terhadap pelaksanaan kebijakan serta guru dan kepala sekolah
yang terkena akibat kebijakan itu.
Kalimat tersebut dikembangkan dari kalimat inti
A. Otonomi sekolah berperan
B. Otonomi sekolah menampung konsensus
C. Otonomi sekolah meningkatkan kualitas
D. Otonomi sekolah dibuat
E. Otonomi sekolah bertanggung jawab

17. SNMPTN 2008


Semua pihak wajib menjunjung tinggi hasil kesepakatan bersama yang telah disusunnya.
Istilah yang tepat untuk menggantikan kata kesepakatan bersama adalah…
Bidang Studi BAHASA INDONESIA 29
KUMPULAN SOAL – SOAL UJIAN MASUK PERGURUAN TINGGI NEGERI

A. konsensus
B. konvensi
C. konsepsi
D. konklusi
E. konsesi

18. SNMPTN 2008


Penggunaan tanda baca dalam kalimat berikut ini sesuai dengan aturan EYD, kecuali
A. Dijalur Losari – Cirebon belum terlihat danya perbaikan, meskipun kerusakannya tergolong parah.
B. Daerah nyaris tidak berdaya, terutama dalam mengelola keuangan daerah.
C. Ketika acara seminar berlangsung, Anda bertugas sebagai notulis.
D. Kita sekarang memerlukan peralatan sekolah: sepatu, tas, dan alat tulis
E. Hak asasi manusia adalah hak yang dimiliki oleh setiap manusia sejak lahir yang tidak boleh dilanggar
dibatasi, ataupun dikurangi oleh siapapun.

19. SNMPTN 2008


Bentuk kata serapan yang benar terdapat dalam kalimat…
A. Radio pada mobil merupakan asesoris yang digemari oleh banyak konsumen
B. Tarian itu dianggap akseptebel untuk ditampilkan di depan tamu
C. Pada jaman penjajahan di kota Malang terbit banyak koran, koran-koran tersebut tidak dapat terbit secara
kontinyu
D. Pematuhan terhadap prinsip kerja sama dan prinsip kesopanan akan melahirkan komunikasi yang bonafide
sekaligus melahirkan pula wacana yang wajar
E. Komunikasi merupakan syarat esensial bagi terciptanya prasyarat tumbuhnya kegiatan yang terbuka, yang
tidak elit.

20. SNMPTN 2008


Penerapan EYD yang benar terdapat dalam kalimat….
A. Kegiatan perkuliahan dimulai,semua kewajibanpun administrasi harus sudah diselesaikan
B. H.Krupper, seorang ahli pra sejarah , melaporkan bahwa telah ditemukan situs Bukit Karang di daerah Langsa
C. Akhirnya, Diana mendapatkan izin belajar keluar negeri setelah berjuang bertahun-tahun
D. Hari jum’at yang lalu, hujan deras mengguyur Jakarta seharian
E. Kami tidak bisa menerima surat tersebut, karena faksimile di kantor kami sedang rusak

21. SNMPTN 2008


Penulisan kata serapan dalam kalimat-kalimat berikut ini benar, KECUALI yang terdapat pada kalimat
A. Setiap pagi kumandang azan itu membangunkan saya.
B. Muna bekerja di laboratorium itu sebagai bakteriolog
C. Orang yang mengusahakan bank disebut bankir
D. Keamanan di ibukota merupakan barometer keamanan di Indonesia
E. Sebelum meninggal, N sempat menyelesaikan otobiografinya

22. SNMPTN 2008


Kalimat berikut yang seluruh ejaannya ditulis sesuai dengan aturan EYD adalah
A. Tiga per empat bagian diberikan kepada orang lain
B. Sekalipun dia belum pernah ke Bali
C. Kemana pun perginya, ia tidak lupa membawa kamera
D. Kalaupun hari itu turun hujan lebat, saya tetap bersedia datang pada acara Anda
E. Mereka satu per satu datangnya tetapi tidak satu pun yang absen

23. SNMPTN 2008


Penulisan bilangan yang tidak sesuai dengan yang digariskan dalam EYD dijumpai dalam kalimat
A. Dari hasil penelitian yang dilakukan Yayasan Kusuma Buana di sekolah-sekolah di Jakarta, prevalensi anemia
disetiap sekolah antara 20 persen dan 35 persen,bahkan ada yang mencapai 60 persen.
B. Sekolah Menengah Kejuruan Negeri (SMKN) X, dua bulan yang lalu, dikunjungi oleh 20 guru dan 5 pejabat
Pemerintah Malaysia
C. Penghargaan Festival Teater diperebutkan oleh 15 finalis yang tampil di Teater Luwes Institut Kesenian
Jakarta dan Teater Studio Taman Iskandar Marzuki
D. Dari 422 laporan yang masuk di Komisi Kejaksaan, sebanyak 202 laporan dilanjutkan ke Jaksa Agung
E. Dari tiga ratus laporan masyarakat yang masuk ke Komisi Kejaksaan pada kurun April – Desember 2006, yang
diteruskan ke Jaksa Agung sebanyak dua ratus laporan

Bidang Studi BAHASA INDONESIA 30


KUMPULAN SOAL – SOAL UJIAN MASUK PERGURUAN TINGGI NEGERI

24. SNMPTN 2008


Studi antropologi ekologi yang diharapkan dapat mengindari berbagai persoalan asumsi – asumsi esensial ,
apriori, dan generalisasi abstrak adalah studi yang bermaksud melakukan suatu pembuktian secara emprik dengan
menempatkan praktik – praktik eksploitas sumber - sumber daya alam dan konsekuensinya dalam konteks –
konteks pengaruh sosial budaya internal dan eksternal yang historis.
Inti gabungan kata studi antropologi ekologi yang diharapkan dapat menghindari berbagai persoalan asumsi –
asumsi esensial, apriori, dan generalisasi abstrak dalam kalimat di atas adalah
A. Antropologi ekologi
B. Studi
C. Asumsi esensial
D. Apriori
E. Generalisasi abstrak

25. SNMPTN 2008


Murid – murid memperbaiki kesalahan pekerjaan rumah matematika. Perbaikan itu sudah diperiksa guru.
Pilihlah nomor dengan pasangan bentukan kata yang betul seperti contoh kalimat di atas.
A. Warga kota memilih calon yang paling popular. Pilihan dilaksanakan beberapa minggu yang lalu.
B. Sulit sekali menentukan masa depan keadaan negara kita. Para peramal saja tidak tidak mampu meprediksi
ketentuan seperti itu.
C. Para korban bencana luapan lumpur Sidoarjo tidak tahu lagi ke mana mereka harus mengadukan nasibnya.
Pengaduannya ditanggai dinggin di mana – mana
D. Para ahli hukum meniadakan ketentuan yang penting dari rencana undang –undang itu. Ketiadaan ketentuan itu
menimbulkan perdebatan seru di parlemen.
E. Pelapor terjadinya kejahatan dilindungi undang – undang. Laporan itu disampaikan secara rahasia.

SOAL BAHASA INDONESIA TAHUN 2007

UM UGM

Orang Indonesia yang gemar melancong ke luar negeri lebih kenal Singapura atau Eropa daripada budaya
negeri sendiri. Penelitian Departemen Kebudayaan dan Pariwisata tahun 2004 mengungkapkan bahwa mereka yang
bepergian ke mancanegara rata-rata baru pernah mengunjungi tiga dari 33 provinsi di Tanah Air.

Selain wisatawan asing, turis lokal sebetulnya dapat menjadi pasar pariwisata. Persoalannya adalah bahwa
faktor yang menarik bagi mereka untuk berwisata di negeri sendiri sangat lemah. Objek wisata di luar Bali misalnya,
rata-rata kurang terawat karena keterbatasan dana. Menjangkau lokasinya pun tak mudah karena sarana transportasi
terbatas. Berbagai retribusi memberatkan pengusaha wisata untuk meningkatkan mutu pelayanan.

Dukungan pemerintah daerah pada usaha kepariwisataan pun masih kurang.


Pada tahun 2000, melalui 13 pintu keberangkatan, tercatat 2,2 juta orang Indonesia yang berkunjung ke negeri asing.
Empat tahun kemudian melonjak menjadi 3,9 juta. Hingga November 2005, sudah 3,7 juta orang melancong ke negara
lain.

Menurut Sekjen Asosiasi Biro Perjalanan dan Wisata (ASITA), objek wisata di negeri orang menyajikan
atraksi menarik pada saat pergantian tahun. Bulan Juni dan Juli, masa libur sekolah, juga waktu favorit warga Indonesia
untuk berlibur ke luar negeri. Kelompok pelancong ke luar negeri terbesar ini adalah penduduk berumur 25 hingga 34
tahun, kelompok yang berada di produktivitas. Mereka umumnya berupa pasangan muda, pengantin baru, keluarga
dengan anak usia di bawah lima tahun, dan dari kelas ekonomi menengah ke atas.
Beban finansial yang belum begitu berat memungkinkan mereka dapat menyisihkan biaya berlibur.

1. UM UGM 2007
Pada Paragraf kedua terdapat kalimat yang tidak baku yaitu kalimat…..
A. kesatu
B. kedua
C. ketiga
D. keempat
E. kelima

2. UM UGM 2007

Bidang Studi BAHASA INDONESIA 31


KUMPULAN SOAL – SOAL UJIAN MASUK PERGURUAN TINGGI NEGERI

Topik yang di bicarakan dalam teks di atas adalah….


A. kecenderungan orang berlibur ke luar negeri
B. tujuan orang berlibur ke luar negeri
C. daya tarik wisata ke mancanegara
D. berkunjung ke Negara tetangga
E. pelancong ke mancanegara

3. UM UGM 2007
Berikut ini adalah pernyataan yang tidak sesuai dengan isi teks di atas.
A. Tempat wisata di Bali lebih terawat daripada tempat lainnya.
B. Bagian terbesar orang Indonesia yang berlibur ke luar negeri adalah kelompok usia produktif.
C. Jumlah orang Indonesia yang berlibur ke luar negeri meningkat dari tahun ke tahun .
D. Umumnya orang lebih suka mengunjungi tempat wisata di dalam negeri sebelum mengunjungi tempat wisata ke
luar negeri
E. Pada tahun 2004 jumlah orang Indonesia yang berkunjung keluar negeri mengalami kenaikan sebesar 1,7 juta
dibansingkan dengan tahun 2000.
4. UM UGM 2007
Mahasiswa idealis yang memiliki visi dan misi serta imajinasi bagus itu tahun depan akan mengikuti program
pertukaran mahasiswa Indonesia – Jepang yang merupakan realisasi kerja sama bidang pendidikan dan
pengajaran.
Pernyataan yang benar berkaitan dengan kalimat……
A. Yang menduduki fungsi subjek dalam kalimat di atas adalah mahasiswa.
B. Kalimat di atas dapat diubah menjadi Program pertukaran mahasiswa Indonesia – Jepang yang merupakan
realisasi kerja sama bidang pendidikan dan pengajaran tahun depan akan di ikuti oleh mahasiswa idealis yang
memiliki visi dan misi serta imajinasi bagus itu.
C. Kalimat di atas sama idenya dengan kalimat Tahun depan program pertukaran mahasiswa Indonesia-Jepang
akan diikutinya.
D. Kalimat di atas tidak dapat diubah menjadi kalimat pasif.
E. Dengan arti atau maksud yang sama, kalimat diatas dapat diubah menjadi Mahasiswa itu tahun depan akan
mengikuti program pertukaran mahasiswa Indonesia-Jepang karena memiliki visi dan misiserta imajinasi bagus.

5. UM UGM 2007
Pelayanan kesehatan di berbagai tempat di tanah air belum berorientasi kepada pasien. Hal ini ditandai oleh
maraknya pemberian obat antibiotik yang tidak sesuai dengan kondisi klinis. Pemakaian antibiotik yang melebihi
kebutuhan bisa mengakibatkan resistansi dalam tubuh pasien.Kuman penyakit dalam tubuh menjadi kebal. Oleh
karen itu,....
Kalimat yang tepat untuk melengkapi paragraf di atas adalah
A. Perlu ditumbuhkan sikap kritis masyarakat terhadap layanan kesehatan.
B. Pasien harus selalu mematuhi semua instruksi dari dokter
C. Pasien haris pasrah pada layanan kesehatan yang diterima.
D. Pasien harus diberi antibiotik di atas kebutuhan supaya daya tahan tubuh lebih tinggi.
E. Setiap penyakit harus diobati dengan antibiotik.

6. UM UGM 2007
Penulisan kata serapan yang benar terdapat dalam kalimat
A. Setiap wanita WNI yang menikah dengan pria WNA harus siap menanggung konsekwensi bahwa status
kewarganegaraan anaknya akan mengikuti suaminya.
B. Kita perlu segera mengkonkritkan gagasannya itu.
C. Penduduk Pasifik yang kebanyakan tinggal di pulau karang adalah kelompok yang paling beresiko.
D. Kami mengusulkan agar salah satu di antara kita bersedia mengkoordinir pelaksanaan kegiatan ini.
E. Dia berhasil memopulerkan jenis olahraga berbahaya itu di negaranya.

7. UM UGM 2007
Penulisan kata bilangan dalam kalimat berikut benar, KECUALI
A. Penulisan bilangan tiga dua pertiga adalah 3 23
B. Pada halaman 1.024 tertera nomor teleponnya 760130
C. Tamu undangan yang hadir dalam perhelatan itu diperkirakan mencapai 500 orang
D. Ulang tahun ke-20 adalah peristiwa penting bagi orang Jepang .
E. Ia telah membaca dua bab dari lima bab buku itu.

8. UM UGM 2007
Kalimat berikut tidak menggunakan tanda baca yang tepat. KECUALI.
Bidang Studi BAHASA INDONESIA 32
KUMPULAN SOAL – SOAL UJIAN MASUK PERGURUAN TINGGI NEGERI

A. Telah dikemukakan bahwa, bahan bakar merupakan energi yang sangat penting dalam kehidupan masyarakat.
B. Meskipun belums empurna, uraian yang disampaikannya sangat emnarik.
C. Investasi yang kita berikan ini, tidak seluruhnya berupa uang.
D. Mutu pelayanan harus selalu ditingkatkan, supaya tidak mengecewakan pelanggan.
E. Kerusakan lingkungan yang parah di Dataran Tinggi Dieng, boleh jadi akan menimpa kawasan lereng Gunung
Ungaran.

9. UM UGM 2007
Penulisan kata yang bercetak miring dalam kalimat berikut yang sesuai dengan EYD adalah….
A. Hasil analisa peneliti itu sangat berguna bagi masyarakat dan perkembangan.
B. Dalam sistim pendidikan yang baik, proses pendidikan dapat diselenggarakan di dalam maupun di luar kelas.
C. Konduite merupakan salah satu syarat untuk naik pangkat.
D. Pendapatnya sangat ekstrim sehingga para peserta rapat tidak menyetujuinya.
E. Tehnik penulisan ilmiah harus dapat dipahami oleh para mahasiswa di perguruan tinggi.

10. UM UGM 2007


Pembentukan kata serapan yang betul terdapat dalam kalimat
A. Di dunia olahraga yang di pentingkan adalah sportivitas.
B. Sikap yang kaku dari seorang pemimpin menimbulkan watak arogan dan otoritair.
C. Majelis juga mengusulkan, agar kalangan eksekutip dan legislatip peka terhadap aspirasi rakyat.
D. Mereka juga memprotes kwalitas beras yang mereka terima.
E. Rapat itu gagal karena yang hadir tidak memenuhi korum.

11. UM UGM 2007


Kalimat Koran Masuk Desa merupakan langkah yang strategis guna meningkatkan kecerdasan masyarakat memiliki
pola yang sama dengan…..
A. Pemerintah mengajukan beberapa usul guna membangun desa-desa tertinggal.
B. Pemerintah meningkatkan kesejahteraan masyarakat dengan proyek-proyek unggulan.
C. Usaha pemerintah berupa pemberian subsidi kepada rakyat kecil harus didukung.
D. Usaha itu adalah strategi yang tepat untuk membangun kesadaran rakyat.
E. Pemerintah berusaha meningkatkan kesejahteraan rakyat dengan upaya.

12. UM UGM 2007


Kalimat yang mencerminkan penalaran yang baik adalah…..
A. Karena kampanye yang melewati ruas jalan itu terjebak macet sehingga pertikaian simpatisan tidak dapat
dihindarkan.
B. Dia yang tertua dari ketiga saudaranya yang kini sukses menjadi pengusaha.
C. Harga fotokopi dengan mesin Xerox yang mengalami kenaikan drastis akhir-akhir ini.
D. Kontrak jasa bertujuan meningkatkan produksi lading-ladang minyak di wilayah Nime-Karim dengan biaya
yang efektif.
E. Mahasiswa yang sedang berunjuk rasa tersebut yang mana terpaksa menjadi pressure force yang harus
diperhitungkan.

13. UM UGM 2007


Setidak-tidaknya ada tiga hal yang dapat dilakukan oleh sebuah perusahaan untuk meningkatkan kualitas dan
disiplin pegawainya. Pertama, perusahaan menyediakan sarana kerja yang memadai. Kedua, dilakukan pemberian
contoh atasan. Ketiga, perusahaan menciptakan suasana kerja yang menyenangkan.
Empat kalimat di atas dapat diringkas menjadi satu kalimat tanpa mengubah ide dasarnya. Kalimat yang tepat
mencerminkan ide yang dikandung empat kalimat di atas adalah….
A. Kualitas dan disiplin pegawai perusahaan dapat ditingkatkan dengan penyediaan sarana kerja yang memadai,
pemberian contoh oleh atasan dan penciptaan suasana kerja yang menyenangkan.
B. Kualitas dan disiplin pegawai perusahaan dapat ditingkatkan dengan tiga hal, yakni disediakan sarana kerja,
diberikan contoh yang baik, dan perusahaan menciptakan suasana kerja yang menyenangkan.
C. Peningkatan kualitas dan disiplin pegawai perusahaan dapat dilakukan dengan menyediakan sarana kerja yang
memadai, pemberian contoh dari atasan, dan diciptakan suasana kerja yang menyenangkan.
D. Untuk meningkatkan kualitas dan disiplin pegawai, perusahaan sebaiknya menyediakan sarana, memberikan
contoh, dan mencpitakan kesenangan.
E. Agar kualitas dan disiplin pegawai perusahaan meningkat, kita harus menyediakan sarana, memberikan contoh,
dan suasana kerja pun tercipta.

14. UM UGM 2007

Bidang Studi BAHASA INDONESIA 33


KUMPULAN SOAL – SOAL UJIAN MASUK PERGURUAN TINGGI NEGERI

Setahun berikutnya, dengan merangkul Jaeckel, Lea Rosh menerbitkan sebuah inisiatif “ Berlin dalam Perspektif”
untuk menggolkan ide pembuatannya memoral untuk para kerutunan Yahudi yang dibunuh secara massal di Eropa.
Inti kalimat tersebut meliputi unsure -unsur
A. Lea Rosh, merangkul, Jaeckel
B. Lea Rosh, menggolkan, ide
C. Lea Rosh, menerbitkan, inisiatif
D. Setahun, Lea Rosh, menggolkan, ide
E. Keturunan Yahudi, dibunuh, di Eropa

15. UM UGM 2007


Kalimat yang menggunakan ragam baku adalah…
A. Dari hasil penelitian membuktikan bahwa epilepsi bukan penyakit menular.
B. Mengenai rancangan undang – undang itu belum disetujui anggota DPR.
C. Hari ini Indonesia akan memberangkatkan tiga belas mahasiswa untuk mengikuti olimpiade matematika di
Perancis.
D. Menurut penelitian Depbudpar 2005 mengungkapkan orang Indonesia yang berlibur ke luar negeri rata-rata
mengeluarkan dana 860 dollar AS dalam setiap kunjungan.
E. Karya sastra melukiskan hakikat yang nyata dari objek sebagai titik tolak, kemudian dengan perlahan bergerak
kearah imajinasi dan perasaan.

16. UM UGM 2007


Deretan kata yang mengemukakan ide yang utuh adalah…
A. Perubahan sistem seleksi mahasiswa baru khususnya di UGM.
B. Penegakan keadilan yang harus didukung segenap elemen bangsa.
C. Mmudarnya semangat gotong-royong yang perlu diperhatikan
D. Tingginya usia harapan hidup merupakan salah satu idnikator makin baiknya tingkat kesejahteraan masyarakat.
E. Kerjasama Indonesia- Timur Tengah yang digalakkan oleh kedua belah pihak.

17. UM UGM 2007


Perayaan imlek, misalnya, diakui sebagai hari libur fakultatif.
Hari libur fakultatif dalam kalimat tersebut maksudnya adalah…..
A. Hari libur yang berlaku untuk kalangan terbatas.
B. Hari libur yang dianjurkan
C. Hari libur yang belum diakui
D. Hari libur yang belum resmi
E. Hari libur yang tidak diwajibkan.

18. UM UGM 2007


Gaya untuk menghaluskan makna terdapat dalam kalimat…
A. Laut biru membentang seluas mata memandang.
B. Tubuhnya kurus kering tinggal kulit pembalut tulangs saja.
C. Semenjak anaknya meninggal, ingatannya agak terganggu.
D. Selama berminggu-minggu ia tidak pernah menampakkan batang hidungnya.
E. Kesedihannya ditampakkan pada rumput yang bergoyang.

19. UM UGM 2007


Walaupun hasil pertanian berupa buah-buahan dan sayur-sayuran itu cukup fluktuatif, produksinya masih dapat
ditingkatkan dengan pendekatan teknologi.
Kata fluktuatif dalam kalimat ini berarti
A. Bersifat naik turun
B. Bersifat turun terus
C. Bersifat naik terus
D. Bersifat naik lalu turun terus
E. Bersifat turun lalu naik terus

Bidang Studi BAHASA INDONESIA 34


KUMPULAN SOAL – SOAL UJIAN MASUK PERGURUAN TINGGI NEGERI

20. UM UGM 2007


Pembangunan perkebunan belum berhasil meningkatkan pendapatan petani kebun pada tingkat yang optimal.
Proses pembentukan kata me-kan pada kata meningkatkan dalam kalimat tersebut mempunyai makna yang sama
dengan proses pembentukan kata dalam kalimat
A. Pembangunan itu dinilai belum menguntungkan.
B. Metode itu akan dicobaterapkan untuk mengembangkan usaha perkebunan itu.
C. Rancang bangun itu diarahkan kepada sentra produksi dengan menggunakan salah satu instrument yang ada
D. Untuk menjalankan misinya, peran kantor cabang merupakan andalan yang penting.
E. Sejumlah nasabah telah memanfaatkan dana kredit program ini.

SPMB

1. SPMB 2007
Setiap universitas di negeri ini mempunyai afiliasi dengan universitas atau perguruan tinggi di luar negeri.
Kata yang tepat untuk menggantikan kata afiliasi adalah
A. pertalian
B. perlindungan
C. persahabatan
D. pengakuan
E. perujukan

2. SPMB 2007
Sebagai data dukung, berbagai informasi yang dikumpulkannya itu tidak valid.
Kata yang tidak tepat untuk menggantikan kata valid dalam kalimat tersebut adalah
A. sahih
B. berlaku
C. sesuai
D. tepat
E. lengkap

3. SPMB 2007
Alokasi anggaran untuk bidang pertahanan dan keamanan tahun ini dirasakan belum memadai dagi Polri untuk
memelihara keamanan dan ketertibab masyarakat.
Alokasi dalam kalimat tersebut memiliki makna
A. penyiapan
B. penjatahan
C. penyediaan
D. penetapan
E. pemenuhan

4. SPMB 2007
Indonesia masih sangat prospektif bagi investor sepatu dunia. Rendahnya tingkat perpindahan buruh dan kuatnya
infrastruktur jaringan pemasok bahan baku merupakan faktor fundamental untuk tumbuhnya industri sepatu.
Kata-kata yang tepat maknanya untuk menggantikan kata bercetak tebal tersebut adalah
A. memungkinkan, pemilik modal, prasarana, dasar.
B. menguntungkan, pemilik modal, sarana, mendasar.
C. diharapkan, pereka cipta, prasarana, pokok.
D. diakui keberadaannya, perancang, sarana, penentu.
E. dimungkinkan, pencipta, prasarana, penting.

5. SPMB 2007
Sedimentasi di Sungai Porong, Kabupaten Sidoarjo, Jawa Timur, mulai terjadi akibat pembuangan lumpur panas
selama dua pekan belakang ke sungai yang bermuara di Selat Madura itu.
Arti kata sedimentasi dalam kalimat tersebut adalah
A. pencemaran air karena zat beracun.
B. pemadatan air akibat gas beracun.
Bidang Studi BAHASA INDONESIA 35
KUMPULAN SOAL – SOAL UJIAN MASUK PERGURUAN TINGGI NEGERI

C. perubahan ekosistem air.


D. pengendapan benda padat karena pengaruh gaya berat.
E. pemisahan antara air, gas, dan benda padat.

6. SPMB 2007
Kalimat berikut yang ditulis sesuai dengan aturan EYD adalah
A. Mereka menyepakati, bahwa mulai tahun ajaran baru akan dilakukan kerja kelompok secara lebih intensif.
B. Untuk mencapai target yang diharapkan, perusahaan perlu melakukan peningkatan pengetahuan dan
keterampilan karyawannya.
C. Barang-barang itu perlu dikelompokkan dan diberkas dalam satu kotak karton tebal, agar memudahkan dalam
pengecekannya.
D. Sebagai akibat dari kurangnya pemahaman terhadap topik yang dibicarakan, Heni tidak bisa memberikan saran
apapun.
E. Bersama para Camat di Banyuwangi, rombongan tamu dari Jepang itu mengikuti jamuan makan malam.

7. SPMB 2007
Kalimat berikut yang ditulis sesuai dengan aturan EYD adalah
A. Saya tidak akan membeli mobil mahal, karena tidak punya uang.
B. Oleh karena itu saya harus rajin menabung.
C. Atas bantuan saudara saya mengucapkan terima kasih.
D. Malam makin larut; pekerjaannya belum selesai juga.
E. Semua siswa baik yang laki-laki maupun perempuan, mengikuti ujian.

8. SPMB 2007
Pengendalian bahaya banjir yang dilakukan pemerintah Indonesia tidak akan berhasil kalau PERDA yang
menaungi tidak dibuat. Dengan demikian usaha itu hanya akan membuang uang sia-sia. Pemerintah perlu segera
membuat PERDA khususnya yang menyangkut pembagian tugas antarinstansi.
Paragraf tersebut akan menjadi paragraf baku jika ejaannya diperbaiki sebagai berikut, KECUALI
A. huruf awal kata pemerintah ditulis huruf kapital.
B. kata PERDA ditulis dengan huruf kecil, kecuali huruf pertama.
C. sebelum kata usaha itu diberi tanda koma (,).
D. sebelum kata khususnya diberi tanda koma (,).
E. kata antarinstansi ditulis terpisah.

9. SPMB 2007
Kalimat berikut yang ditulis sesuai dengan pedoman EYD, KECUALI
A. Pakistan dan Malaysia akan bekerja sama melakukan riset, pengembangan, pelatihan, dan pendidikaan
keuangan Islam.
B. Tidak dapat dipungkiri bahwa pendidikan multi kultural telah meleburkan hegemoni pasar yang dikuasai laki-
laki ke dalam pangkuan kaum perempuan.
C. Bank pemerintah Pakistan telah memberikan izin bagi enam bank Islam dan sebelas bank konvensional untuk
mengembangkan sistem perbankan Islam.
D. Saat ini, virus mempunyai varian sasaran bukan hanya pada media penyimpanan, melainkan pada hampir
semua hal yang berhubungan dengan komputer dan sistem di dalamnya yang memungkinkan untuk diserang.
E. Dengan perkembangan internet dan jaringan yang cukup pesat, virus dilengkapi kemampuan menyerang
melalui jaringan komputer sehingga penyerangannya sangat cepat.

10. SPMB 2007


Kalimat berikut yang seluruhnya ditulis sesuai dengan aturan EYD adalah
A. Kehematan yaitu salah satu ciri kalimat efektif.
B. Meskipun demikian, hal itu tidak berarti bahwa mereka bebas dari tugas.
C. Jika informasinya jelas kalimat itu disebut kalimat efektif, karena dapat dipahami isinya.
D. Betapa pun panjangnya suatu kalimat yang penting adalah informasinya.
E. Dalam kalimat efektif tidak ada ide kalimat yang dapat ditafsirkan secara ganda maknanya.

11. SPMB 2007


Kalimat berikut yang ditulis sesuai dengan EYD adalah
A. Tidak seorang bupati pun hadir dalam acara halal bihalal di Surabaya kemarin.
B. Hampir di setiap daerah, terdapat Universitas tetapi mahasiswanya sangat sedikit.
C. Dalam setiap Bab buku yang berjudul Pernik-pernik Berbahasa, disajikan ringkas.
D. Selama menjadi direktur di Bank Araya, ia menunjukkan pengabdian yang baik.
Bidang Studi BAHASA INDONESIA 36
KUMPULAN SOAL – SOAL UJIAN MASUK PERGURUAN TINGGI NEGERI

E. Semua Fakultas Kedokteran di seluruh Indonesia saat ini melakukan akreditasi.

12. SPMB 2007


Setiap masyarakat tutur memiliki bahasa dan sekaligus menggunakan bahasa tersebut untuk berkomunikasi. Hal ini
berarti bahwa anggota masyarakat tutur selain harus menguasai kaidah bahasa juga wajib menguasai kaidah
berbahasa. Kaidah bahasa berkenaan dengan anasir lingua, sedangkan kaidah berbahasa berkenaan dengan anasir
sosial budaya.
Pernyataan berikut yang sesuai dengan paragraf tersebut adalah
A. Orang yang menguasai kaidah bahasa secara memadai mampu berkomunikasi dengan baik dan lancar.
B. Kaidah bahasa dan kaidah berbahasa adalah dua kaidah yang berbeda, tetapi pada prinsipnya sama.
C. Bagi orang yang menguasai kaidah berbahasa, performansi bahasanya akan lebih santun dan berterima.
D. Dalam kegiatan berkomunikasi, penguasaan kaidah bahasa lebih diutamakan daripada penguasaan kaidah
berbahasa.
E. Dengan penguasaan kaidah berbahasa secara baik, orang akan dapat menggunakan bahasa secara benar.

13. SPMB 2007


Suriname didominasi warga keturunan Kindustan, Afrika, dan Jawa. Ada lagi kelompok keturunan Cina, Belanda,
Arab, Yahudi, Brasil, dan campuran suku-suku Indian. Agama dominan agama Kristen, Hindu, dan Islam. Tiap
kelompok etinis kini berbaur. Mereka saling menghormati kebudayaan dan agama masing-masing. Sejak merdeka
dari Belanda 30 tahun lalu, penduduknya kini hampir setengah juta jiwa.
Masalah pokok yang dibahas dalam paragraf tersebut berhubungan dengan
A. macam-macam penduduk di Suriname.
B. agama-agama di Suriname.
C. pembauran kelompok etnis di Suriname.
D. jumlah penduduk Suriname kini.
E. keadaan Suriname kini.

14. SPMB 2007


Minyak ikan merupakan suplemen yang cukup populer. Selain digunakan untuk meningkatkan nafsu makan anak,
minyak ikan juga banyak dikonsumsi wanita hamil untuk meningkatkan kualitas kehamilannya. Minyak ikan
mengandung asam lemak yang banyak bermanfaat bagi kesehatan. Minyak ikan mengandung 25% asam lemak
jenuh dan 75% asam lemak tak jenuh. Asam lemak pada minyak ikan ada 3, yaitu asam lemak jenuh, asam lemak
tak jenuh tunggal, dan asam lemak tak jenuh ganda.
Simpulan yang cocok dengan paragraf tersebut adalah
A. minyak ikan populer.
B. minyak ikan mengandung asam lemak.
C. asam lemak pada minyak ikan ada tiga.
D. minyak ikan mengandung 75% asam lemak.
E. minyak ikan banyak gunanya.

15. SPMB 2007


Jerjak jembatan sungai yang mengalir deras itu hampir saja ambruk kemarin diterjang banjir.
Kalimat tersebut berasal dari kalimat inti
A. Jerjak diterjang banjir.
B. Jembatan ambruk.
C. Jerjak ambruk.
D. Sungai mengalir.
E. Sungai banjir.

16. SPMB 2007


Sekarang ini, hal penting yang sering dilupakan oleh para karyawan perusahaan yang sedang dilanda
kebangkrutan adalah pencarian solusi yang tepat yang dapat menguntungkan kedua belah pihah.
Kalimat inti dari kalimat luas tersebut adalah
A. Yang sering dilupakan karyawan adalah pencarian solusi.
B. Hal penting adalah pencarian solusi.
C. Karyawan perusahaan dilanda kebangkrutan.
D. Solusi yang tepat dapat menguntungkan.
E. Hal penting sering dilupakan.

17. SPMB 2007


Kalimat berikut ini tidak baku, KECUALI
Bidang Studi BAHASA INDONESIA 37
KUMPULAN SOAL – SOAL UJIAN MASUK PERGURUAN TINGGI NEGERI

A. Khawatir cadangan air tanah tersedot dan sulit mendapatkan air bersih, warga setempat menolak rencana
pembuatan sumur dalam di sekitar tempat tinggal mereka.
B. Bidang lingkungan hidup, dana yang dialokasikan sebesar Rp300 juta yang diarahkan pada kegiatan
konservasi, yaitu sumur resapan.
C. Untuk mengetahui penyebab keracunan, sisa makanan sebagai sampel telah dikirim ke laboratorium Dinas
Kesehatan Jawa Tengah.
D. Bagi pengusaha yang belum mampu menggaji karyawannya sesuai dengan ketentuan, diwajibkan membuat
surat penangguhan ke gubernur.
E. Untuk larangan praktik pelayanan kesehatan pasien bagi mantri kesehatan dan perawat sudah final.

18. SPMB 2007


Krisis ekonomi, yang turut berperan dalam penambahan jumlah pengangguran di Indonesia, juga ditambah
berbelitnya birokrasi di negara ini, yang menjadikan para investor asing enggan untuk menanamkan modalnya ke
Indonesia menyebabkan pemerintah harus berjuang memperbaiki struktur ekonominya.
Kalimat tersebut akan menjadi baku bila diperbaiki dengan cara
A. menghilangkan tanda koma (,) sesudah kata ekonomi.
B. mengganti juga dengan kata yang.
C. menghilangkan tanda koma (,) sebelum kata juga.
D. menghilangkan untuk.
E. menambah tanda koma (,) di depan kata menyebabkan.

19. SPMB 2007


Jumlah dana yang dialokasikan untuk pemberdayaan masyarakat di tingkat kelurahan cukup besar sehingga tidak
menutup kemungkinan peningkatan kualitas lingkungan yang jauh lebih baik daripada yang dicapai lahun lalu.
Kalimat tersebut tidak efektif sehingga perlu perbaikan dengan cara
A. mengganti kata dialokasikan dengan dianggarkan.
B. mengganti kata untuk dengan guna.
C. mengganti kata daripada dengan dibandingkan.
D. mengganti kata cukup dan jauh.
E. mengganti kata menutup dengan tertutup.

20. SPMB 2007


Di antara kalimat berikut ini, yang termasuk kalimat tidak baku adalah
A. Apabila ditanggung bersama, masalah ini akan dapat segera diselesaikan dan bebannya pun akan terasa lebih
ringan.
B. Ketika mengikuti suatu pertemuan formal atau sedang melakukan rapat, yang paling sopan adalah tidak
mengaktifkan HP.
C. Pada setiap kata yang dituturkannya selalu mengandung kritikan yang tajam pada pemerintah.
D. Semua orang memiliki kekuasaan bawaan, entah berkepribadian menarik atau berkharisma tinggi, untuk
memberikan pengaruh.
E. Dalam menghadapi konflik sosial dan krisi ekonomi yang berkepanjangan ini, berdoa dan berusaha merupakan
cara yang terbaik.

21. SPMB 2007


Diskusi itu diawali dengan pengajuan masalah oleh tiap-tiap kelompok. Masalah yang dikemukakan oleh setiap
kelompok dibahas bersama-sama dalam sidang itu. Dalam pembahasan ini, kelompok lainnya dapat menanggapi
dan menyampaikan pengarahan sesuai dengan visi kelompoknya. Setelah seluruh kelompok menyampaikan masalah
dan tanggapannya, diskusi diakhiri dengan penyusunan simpulan.

Bentukan kata yang tidak tepat dalam paragraf tersebut adalah


A. pengajuan
B. pengarahan
C. pembahasan
D. tanggapan
E. simpulan

Bidang Studi BAHASA INDONESIA 38


KUMPULAN SOAL – SOAL UJIAN MASUK PERGURUAN TINGGI NEGERI

22. SPMB 2007


Sejenis umbi-umbian banyak dijual di Pasar Kliwon.
Proses pembentukan kata umbi-umbian sejalan dengan proses pembentukan kata bercetak miring pada
kalimat berikut, KECUALI
A. Makanan sejenis kacang-kacangan tidak baik bagi penderita asam urat.
B. Angin puting beliung menumbangkan pohon-pohonan di wilayah tersebut.
C. Buah-buahan memiliki manfaat yang besar bagi kesehatan tubuh kita.
D. Penderita sakit lambung dilarang mengonsumsi manis-manisan buah.
E. Banjir besar di wilayah desa menghancurkan bunga-bungaan yang indah.

23. SPMB 2007


Pada era globalisasi, hampir setiap orang tidak hanya di perkotaan, tetapi juga di pedesaan dengan mudah dapat
memanfaatkan telepon. Penduduk di pedesaan tinggal mendatangi sarana telepon umum atau warung
telekomunikasi (wartel) untuk berkomunikasi dengan orang lain di tempat yang berjauhan. Wartel pada saat ini ada
yang telah dilengkapi sarana internet yang dapat digunakan oleh masyarakat.
Kalimat penutup yang tepat untuk melengkapi paragraf tersebut adalah
A. Dengan demikian, kehidupan pada era globalisasi menghendaki pelayanan yang serba cepat.
B. Pendek kata, teknologi komunikasi menyediakan sarana komunikasi yang canggih.
C. Dengan demikian, segala aktifitas menjadi lancar dengan adanya teknologi yang canggih di bidang
komunikasi.
D. Pendek kata, untuk memperoleh sarana komunikasi yang jauh lebih maju kian mudah.
E. Dengan demikian, masyarakat pedesaan pun dengan mudah mendapatkan informasi terbaru dari intarnet.

24. SPMB 2007


(1) Pembaharuan dalam bidang pendidikan di Indonesia dengan melaksanakan Manajemen Peningkatan Mutu
Berbasis Sekolah (MPBS) semakin meningkat hasil yang menggembirakan. (2) Berdasarkan data yang terhimpun
pada Diraktorat PLP dengan sampel 1000 sekolah dalam pelaksanaan selama tiga tahun terakhir ini menunjukan
bahwa sekolah menjadi semakin mandiri dan kualitas belajar siswa semakin tinggi. (3) Berpangkal dari hasil yang
realisasi tersebut, akhirnya MPBS ditetapkan menjadi salah satu pola meningkatkan mutu dan memajukan sekolah
yang berlaku secara nasional.
Paragraf tersebut akan menjadi paragraf yang baik jika direvisi dengan cara
A. memindahkan dengan melaksanakan manajemen peningkatan mutu berbasis sekolah (MPBS)
(kalimat 1) ke akhir kalimat.
B. Menghilangkan kata berdasarkan (kalimat2).
C. Memindahkan dengan sampel 1000 sekolah dalam pelaksanakan selama tiga tahun berakhir ini (kalimat 2) ke
awal kalimat.
D. Mengganti berpangkal dari (kalimat 3) dengan sebuah dengan.
E. Menghilangkan kata akhirnya (kalimat 3).

25. SPMB 2007


Enzim merupakan katalis organik yang diproduksi organisme hidup. Salah satu enzim ini adalah protease, yaitu
enzim yang berfungsi memutus ikatan peptida dari suatu polipeptida. Enzim protease banyak digunakan, khususnya
dalam dunia industri. Hampir 60% jumlah enzim yang diproduksi adalah protease. Industri yang menggunakan
protease antara lain industri keju, deterjen, perikanan, serta industri minuman beralkohol.
Ringkasan berikut yang sesuai dengan isi paragraf tersebut adalah
A. Protease yang merupakan salah satu enzim banyak digunakan dalam industri minuman beralkohol.
B. Katalis organik yang dikenal dengan enzim protease 60% diproduksi oleh dunia industri.
C. Dunia produksi banyak memproduksi enzim protease untuk memutus ikatan peptida dari suatu polipeptida.
D. Salah satu enzim protease yang berfungsi memutus ikatan peptida banyak digunakan industri perikanan.
E. Sebanyak 60% enzim protease yang digunakan oleh industri deterjen diproduksi oleh organisasi hidup.

Bidang Studi BAHASA INDONESIA 39


KUMPULAN SOAL – SOAL UJIAN MASUK PERGURUAN TINGGI NEGERI

SOAL BAHASA INDONESIA TAHUN 2006

UM UGM

Naiknya permukaan air laut telah mendorong 100 orang penduduk sebuah pulau di Samudera Pasifik
berpindah ke lokasi yang lebih tinggi. Ini adalah dampak pemanasan global yang jelas terlihat kerugiannya bagi sebuah
komunitas. Pohon-pohon kelapa yang ada di pinggir pantai telah terendam air dan para penduduk Lateu di Pulau Tegue,
Vanuatu, mulai membongkar rumah kayunya dan berpindah ke pulau di dekatnya yang 600 meter lebih tinggi.
“Mereka tidak dapat tinggal lebih lama lagi di pantai.” Kata Taito Nakalevu, seorang ahli perubahan iklim di
Sekretariat Program Lingkungan untuk Regional Pasifik. Saat menghadiri konferensi untuk melawan perubahan iklim
di Montreal yang dihadiri 89 negara. Air pasang yang tinggi karena badai menjadi semakin besar dalam tahun-tahun
terakhir dan menyebabkan Lateu tidak lagi dipenghuni karena sering disapu banjir antara 4 hingga 5 kali dalam setahun.
“Kami melihat pasang yang tinggi menyapu pulau-pulau di sana.” Katanya Program Lingkungan PBB (UNEP)
menyatakan bahwa wilayah Lateu menjadi salah satu – kalau tidak boleh dikatakan yang pertama daerah yang secara
normal berpindah karena pengaruh buruk perubahan iklim.
Panel ilmuwan yang memberi saran pada PBB memperkirakan bahwa permukaan air laut akan naik paling
tidak satu meter pada 2100 karena melelehnya es sebagai dampak pemanasan global yang dipicu terjebaknya panas di
atmosfer. Gas yang dihasilkan dari pembakaran bahan bakar fosil di pembangkit listrik, pabrik, dan mobil adalah
penyebab efek yang disebut rumah kaca itu.
Banyak komunitas pantai lainnya yang terancam karena kenaikan air laut, misalnya New Orleans di AS,
Venesia di Italia, atau pemukiman di kutub utara. Mencairnya gletser dan gunung es menyebabkan erosi pantai oleh
gelombang air laut.

1. UM UGM 2006
Topik yang dibicarakan dalam kutipan di atas mengenai
A. banjir di kepulauan Samudera Pasifik
B. penyebab naiknya permukaan air laut
C. perubahan iklim global
D. hilangnya sebuah pulau di kawasan Samudera Pasifik
E. dampak pemanasan global

2. UM UGM 2006
Pernyataan yang sesuai dengan bacaan di atas adalah
A. Gas hasil pembakaran bahan bakar fosil menjadi salah satu penyebab efek rumah kaca
B. Penduduk Lateu tinggal di Papua New Guinea
C. Para ilmuwan memperkirakan bahwa permukaan air laut akan naik lebih dari satu meter
D. Mencairnya gletser dan gunung es di kutub Utara menyebabkan banjir di sebagian besar kawasan Samudera
Pasifik

3. UM UGM 2006
Pernyataan yang tidak benar yang berhubungan dengan pulau Tegua adalah
A. Rumah kayu merupakan rumah khas penduduk di Pulau Tegua
B. Di Pulau tegua terdapat banyak pohon kelapa
C. Badai menyebabkan air pasang yang tinggi di Pulau Tegua
D. Pulau Tegua berpenduduk 100 orang
E. Penduduk Lateu berpindah ke daerah lain karena tempat tinggalnya sering dilanda banjir

4. UM UGM 2006
Penulisan nama dan jabatan yang tepat terdapat pada kalimat
A. Ayahnya adalah seorang Gubernur
B. Siapa Bupati yang baru dilantik itu?
C. Seminggu yang lalu Kolonel Haryono meninggal dunia
D. Brigjen Sumadi baru dilantik menjadi Mayor jenderal
E. Presiden baru saja melantik beberapa Menteri.

Bidang Studi BAHASA INDONESIA 40


KUMPULAN SOAL – SOAL UJIAN MASUK PERGURUAN TINGGI NEGERI

5. UM UGM 2006
Pemakaian tanda baca yang tepat terdapat dalam kalimat :
A. Berbagai persoalan yang harus diselesaikan meliputi persoalan di bidang politik, ekonomi, sosial dan
keamanan.
B. Berbagai persoalan yang harus diselesaikan meliputi persoalan di bidang politik, sosial, ekonomi, dan
keamanan.
C. Berbagai persoalan yang harus diselesaikan meliputi persoalan di bidang politik, sosial dan ekonomi serta
keamanan.
D. Berbagai persoalan yang harus diselesaikan meliputi persoalan di bidang politik, sosial ekonomi dan
keamanan.
E. Berbagai persoalan yang harus diselesaikan meliputi persoalan di bidang politik, sosial, dan ekonomi serta
keamanan.

6. UM UGM 2006
Penulisan gabungan kata yang sesuai dengan EYD dalam kalimat di bawah ini adalah
A. Setiap Hari Raya Idul Fitri keluarga besar itu selalu mengadakan acara halalbihalal
B. Jika ingin membaca, ayah selalu harus mengenakan kaca mata
C. Adi saya selalu membawa sapu tangan hijau kesayangannya kalau sedang pergi ke luar kota
D. Mereka memberikan bantuan secara suka rela kepada para korban bencana alam itu
E. Mereka turut berduka cita atas meninggalnya salah seorang teman mereka

7. UM UGM 2006
Penggunaan bentuk kata majemuk dapat ditemukan dalam kalimat berikut, KECUALI
A. Akibat jatuh dari sepeda motor, kaki tangan Agus menderita patah tulang dan terpaksa harus dirawat beberapa
minggu di rumah sakit
B. Masyarakat Desa Makmur Sari mencurigai sepak terjangnya karena ia memiliki kemiripan wajah dengan
penjahat yang menjadi buronan polisi
C. Untuk memperoleh kredit perumahan rakyat, para konsumen harus menyetor uang muka sebesar 25 persen dari
harga tunai
D. Setelah musim tanam usai, kebanyakan para petani yang berasal dari desa pergi ke kota untuk mencari
pekerjaan sambilan, seperti tukang angkut barang, penarik becak, dan sebagainya
E. Warga Desa Kayu Wangi yang tanah perkarangannya terkena pelebaran jalan memperoleh ganti rugi dari
pemerintah

8. UM UGM 2006
Kata yang dibentuk secara tidak baku terdapat pada kalimat
A. Untuk meningkatkan daya saing, cara penyajian harus dirubah
B. Perubahan dalam hal penyajian merupakan syarat mutlak
C. Mereka meprotes keras pengubahan jadwal pertandingan itu
D. Pantia hendaknya tidak sekehendak hati mengubah jadwal pertandingan
E. Jadwal yang berubah setiap saat akan menyusahkan semua pihak

9. UM UGM 2006
Kalimat yang mengandung pilihan kata secara tepat adalah
A. Adalah satu hal yang sepatutnya diperhatikan apabila penelusuran minat dan bakat seorang anak dilakukan
sebelum memilih program studi
B. Penelusuran minat dan bakat seorang anak sebelum memilih program studi adalah satu hal yang sepatutnya
diperhatikan
C. Bahwa sebelum memilih program studi, penelusuran bakat dan minat seorang anak adalah satu hal yang
sepatutnya diperhatikan
D. Sebelum memilih program studi bahwa penelusuran minat dan bakat seorang anak adalah satu hal yang
sepatutnya diperhatikan
E. Merupakan satu hal yang sepatutnya diperhatikan bahwa penelusuran minat dan bakat seorang anak sebelum
memilih program studi

Bidang Studi BAHASA INDONESIA 41


KUMPULAN SOAL – SOAL UJIAN MASUK PERGURUAN TINGGI NEGERI

10. UM UGM 2006


ICEL (Indonesian Center for Environmental Law) yang merupakan advokasi SUTET bersama sejumlah LSM yang
terbagung dalam Konsorsium Dampak jaringan Listrik melaporkan kasus listrik tegangan tinggi yang mulai naik ke
permukaan sejak 1989. Unsur inti kalimat tersebut adalah
A. ICEL, melaporkan, kasus listrik tegangan tinggi
B. ICEL dan LSM, melaporkan, kasus listrik sejak 1989
C. ICEL, advokasi SUTET, LSM, melaporkan, kasus listrik tegangan tinggi
D. ICEL, advokasi SUTET, LSM, Konsorsium Dampak Jaringan Listrik, melaporkan, kasus listrik tegangan
tinggi, sejak 1989
E. ICEL, SUTET, LSM, kasus listrik

11. UM UGM 2006


Cahaya matahari berlomba menyusup lekukan kaku dinding-dinding beton kelam di jantung kota Berlin. Pola
kalimat yang sama dengan kalimat tersebut adalah
A. Bangunan itu dikelilingi dinding sepanjang 100 meter yang terpahatkan nama kota dan desa tempat peristiwa
itu berlangsung
B. Terobosan manajer televisi itu dilakukan untuk menggaet iklan agar melimpah
C. Holocaust Memorial berada di tengah megahnya Brandenburger Tor, lambang bersatunya dua Jerman
D. Matanya memandang jauh menyusuri lekuk-lekuk pekuburan Yahudi tertua di Eropa itu
E. Bangungan-bangunan megah nan anggun itu jelas kontras dengan ribuan batu nisan yang menyelimuti areal
seluas dua kali lapangan sepak bola.

12. UM UGM 2006


Kalimat di bawah ini yang merupakan kalimat majemuk campuran adalah
A. Dia sudah melamar pekerjaan ke mana-mana, tetapi belum juga berhasil mendapatkan pekerjaan
B. Selang beberapa waktu masuklah mereka
C. Saya menjaga rumah, sedangkan ibu pergi ke pasar
D. Adik mendapat nilai buruk, padahal dia sudah belajar giat
E. Ketika ayah datang, adik bermain bola dan kakak membaca buku

13. UM UGM 2006


Yang termasuk tunggal berikut ini adalah
A. Kulitnya menghitam dan rambutnya menipis
B. Ia mengakui ia pelakunya
C. Dalam kesempatan ini kami ingin menyampaikan ribuan terima kasih kepada hadirin sekalian yang telah
berjasa dalam proses pembangunan Aceh pasca Tsunami ini.
D. Ia lupa bahwa hari ini ada rapat
E. Walaupun hari ini libur, ia tetap bekerja

14. UM UGM 2006


Para mahasiswa melakukan protes turun ke jalan untuk menentang kebijakan pemerintah menaikkan harga BBM.
Kalimat ini terbentuk dari perluasan kalimat inti
A. Pemerintah menaikkan harga BBM
B. Mahasiswa turun ke jalan
C. Mahasiswa menentang kebijakan pemerintah
D. Mahasiswa melakukan protes
E. Mahasiswa menentang kenaikan BBM

15. UM UGM 2006


Di antara kalimat berikut ini yang menggunakan ragam kalimat baku adalah
A. Garis keturunan dari pihak ibu masih sulit diakomodasi dalam RUU Kewarganegaraan
B. Menginjak tahun 1970-an kemajuan ilmu dan tekonologi di segala bidang maju pesat dengan ditemukannya
microchip di mana merupakan komponen dasar komputer
C. Dari dialog itu tampak banyak sekali kelemahan daripada Undang-undang Nomor 62 itu
D. Ketidakadilan dalam RUU itu terlihat terutama dalam ketentuan yang mana mengatur perkawinan antar-bangsa
E. Banyak pengusaha nakal yang melarikan diri ke luar negeri dengan membawa asset dari negara secara
melanggar hukum

Bidang Studi BAHASA INDONESIA 42


KUMPULAN SOAL – SOAL UJIAN MASUK PERGURUAN TINGGI NEGERI

16. UM UGM 2006


Dua atau beberapa universitas yang besar bergabung menjadi satu. Istilah bergabung dalam konteks ini sepadan
dengan kata
A. kombinasi
B. merger
C. akomodasi
D. konvergensi
E. amalgamasi

17. UM UGM 2006


Bank Indonesia menilai bahwa gejolak harga minyak internasional yang terjadi saat ini tidak akan berpengaruh
yang signifikan terhadap nilai tukar rupiah. Alasannya, bahwa kebutuhan dollar AS untuk impor minyak
cenderung menurun.
Kata signifikan pada kalimat tersebut mengandung makna yang sama dengan kata berikut ini.
A. secara berlebihan
B. secara bermakna
C. besar
D. secara kuat
E. secara merata

18. UM UGM 2006


Dalam pasal 9 ayat 2 RUU Bahasa menyebutkan bahwa pidato kenegaraan yang disampaikan baik di dalam
maupun di luar negeri harus menggunakan Bahasa Indonesia yang baik dan benar
Agar kalimat tersebut baku, kata yang perlu dihilangkan adalah
A. dalam (sebelum kata pasal)
B. menyebutkan
C. bahwa
D. harus
E. dalam (sesudah kata depan)

19. UM UGM 2006


…panen padi belum merata, sejumlah pengumpul gabah mengaku kesulitan mencari stok gabah kering
panen…ada, rendemen hasil panen padi lokal jauh lebih rendah…biasanya. Kelangkaan stok gabah kering
panen…disebabkan tidak adanya panen,…berubahnya sistem pemesanan gabah.
Kata penghubung yang tepat untuk mengisi titik-titik di atas adalah
A. karena, jika, karena, dan
B. sebab, ketika, jika, makna
C. akibat, kalaupun, daripada, bukan, melainkan
D. karena, ketika, dari, tidak, tetapi
E. akibat, ketika, dan, kalau, maka

20. UM UGM 2006


Dari karakter fisik manusia Indonesia menunjukkan warna kulit. Lipatan kelopak mata, rambut yang secara umum
digolongkan ke dalam ras Mongoloid dan Australomelanesid
Kalimat tersebut menjadi baku jika diubah menjadi
A. Dari karakter fisik manusia Indonesia menunjukkan warna kulit, lipatan kelopak mata, dan rambut yang
secara umum digolongkan ke dalam ras Mongoloid dan Australomelanesid
B. Dari karakter fisik manusia Indonesia ditunjukkan bahwa warna kulit, lipatan kelopak mata, dan rambut yang
secara umum digolongkan ke dalam ras Mongoloid dan Australomelanesid
C. Karakter fisik manusia Indonesia menunjukkan bahwa warna kulit, lipatan kelopak mata, dan rambut yang
secara umum digolongkan ke dalam ras Mongoloid dan Australomelanesid
D. Karakter fisik manusia Indonesia ditunjukkan oleh warna kulit, lipatan kelopak mata, dan rambut yang secara
umum digolongkan ke dalam ras Mongoloid dan Australomelanesid
E. Karakter fisik manusia Indonesia menunjukkan bahwa warna kulit, lipatan kelopak mata, dan rambut secara
umum digolongkan ke dalam ras Mongoloid dan Australomelanesid

Bidang Studi BAHASA INDONESIA 43


KUMPULAN SOAL – SOAL UJIAN MASUK PERGURUAN TINGGI NEGERI

SPMB

1. SPMB 2006
Lingkungan sosiobudaya Indonesia yang terdiri atas berbagai kelompok etnik yang tertabur dalam beribu-ribu
pulau memancar dalam bentuk pernyataan seni yang disebut seni daerah.
Kalimat inti dari kalimat luas tersebut adalah ....
A. Lingkungan sosiobudaya Indonesia terdiri atas berbagai kelompok etnik
B. Lingkungan sosiobudaya Indonesia bertabur dalam beribu-ribu pulau
C. Berbagai kelompok etnik tertabur dalam beribu-ribu pulau.
D. Lingkungan sosiobudaya memancar.
E. Berbagai etnik kelompok memancar.

2. SPMB 2006
Kalimat berikut yang termasuk kalimat baku adalah ....
A. Pada bagian pertama tulisan ini akan menelusuri arti dan dimensi mekanisme kekuasaan.
B. Kalau dilihat pada zaman sekarang, kolonialisme seperti zaman dahulu, yakni menduduki wilayah secara fisik,
barangkali sudah tidak ada.
C. Jika disajikan secara lebih rinci dan lebih operasional, Bab III laporan penelitian ini akan memberikan
informasi yang lengkap pada pembacanya
D. Sebelum moderator memberikan kesempatan kepada peserta seminar untuk bertanya, terlebih dahulu meminta
pemakalah menyajikan makalahnya.
E. Subsidi BBM yang diberikan oleh pemerintah itu untuk membantu rakyat miskin agar dapat meringankan
beban perekonomian mereka.

3. SPMB 2006
Sejak lahirnya konsep pemikiran baru dalam kedokteran, yang dicetuskan oleh Profeosr Linus Pauling, yakni
tentang ortomoleculer medicine yang dasarnya adalah studi biologi molekuler sebagai sains dasar, penelitian
medis diarahkan pada molekul-molekul yang secara normal biologis fisiologis ada dalam tubuh manusia.
Inti kalimat panjang tersebut adalah
A. Konsep pemikiran baru dicetuskan oleh Profesor Linus Pauling.
B. Ortomoleculer medicine adalah sains dasar.
C. Ortomoleculer medicine dasarnya studi biologi.
D. Penelitian medis diarahkan pada molekul.
E. Biologi molekuler merupakan sains dasar.

4. SPMB 2006
Kalimat Untuk mengembangkan Kota Batu sebagai kota pariwisata terkemuka di Indonesia mengandung makna
mendorong pertumbuhan dan perubahan Kota Batu sebagai pusat kunjungan para turis tidak baku.
Kalimat tersebut akan menjadi baku apabila
A. kata untuk dihilangkan.
B. kata yang ditambahkan sebelum kata terkemuka.
C. Kata mengandung diganti dengan memiliki.
D. Kata Kota Batu yang kedua diganti dengan kata tersebut.
E. kata bagi ditambahkan setelah kata kunjungan.

5. SPMB 2006
Penggunaan kata penghubung intrakalimat yang terdapat dalam kalimat
A. Walaupun sudah dilakukan pendataan secara cermat, tetapi masih juga ada keluarga miskin yang berlum
terdaftar untuk mendapatkan subsidi BBM.
B. Jika kurang koordinasi antara pihak eksekutif dan legislatif, maka semua peraturan yang dibuat selalu
menimbulkan masalah dalam pelaksanaannya.
C. Kondisi perekonomian negara akan semakin terpuruk jika pemerintah tidak segera menyatukan visi dan
misinya bersama anggota dewan.
D. Dengan diberlakukannya keputusan sepihak oleh pemerintah, maka dewan juga mengambil keputusan sepihak
tanpa menghiraukan suara rakyatnya.
E. Pemerintah tidak bermaksud untuk memberatkan kehidupan masyarakat, melainkan bermaksud untuk
menyelamatkan perekonomian negara.

Bidang Studi BAHASA INDONESIA 44


KUMPULAN SOAL – SOAL UJIAN MASUK PERGURUAN TINGGI NEGERI

6. SPMB 2006
Kalimat berikut yang termasuk kalimat baku adalah
A. Menurut Yeo bahwa birokrasi di Indonesia terlalu rumit sehingga menimbulkan banyak masalah bagi investor
yang mau bergabung.
B. Pihak biro harga BBM berpengaruh besar terhadap minat wisatawan untuk melakukan kunjungan.
C. Berdasarkan hasil survei menunjukkan bahwa sampai saat ini wisata pantai masih tetap diminati, terutama oleh
kaum muda.
D. Menurut pengamatan lingkungan, krisis air bersih dan masalah udara segar mulai merisaukan penduduk di
kota-kota besar.
E. Dengan adanya alam yang menelan korban ratusan ribu nyawa manusia menggugah nurani dan sikap
kedermawanan semua orang.

7. SPMB 2006
Untuk mengamankan jalur pantura, polisi menurunkan 2.000 anggota pasukan penembak jitu.
Berdasarkan konteks tersebut, tahapan proses pembentukan kelompok kata anggota pasukan penembak jitu adalah
A. pasukan  anggota pasukan  anggota pasukan penembak  anggota pasukan penembak jitu.
B. Pasukan  pasukan penembak  pasukan penembak jitu  anggota pasukan penembak jitu.
C. Penembak  penembak jitu  pasukan penembak jitu  anggota pasukan penembak jitu.
D. Penembak  pasukan penembak  pasukan penembak jitu  anggota pasukan penembak jitu.
E. Pasukan  pasukan penembak  anggota pasukan penembak  anggota pasukan penembak jitu.

8. SPMB 2006
Menulis berarti menterapkan semua pengetahuan yang dimilikinya. Dalam menulis, banyak proses yang harus
diperhatikan; diantaranya adalah pemakaian ejaan sesuai sengan kaidah bahasa yang dianut, pemakaian struktur
bahasa yang benar, dan pemakaian diksi yang mengungkapkan fikiran.
Dalam paragraf tersebut terdapat kesalahan tata tulis. Kesalahan yang dimaksud adalah sebagai berikut, kecuali ....

A. kesalahan penulisan menterapkan.


B. Kesalahan penulisan diantaranya.
C. Kesalahan tanda koma (,) sebelum dan.
D. Kesalahan tanda titik koma (;) setelah diperhatikan.
E. Kesalahan penulisan fikiran alih-alih pikiran.

9. SPMB 2006
Penulisan bilangan yang salah terdapat dalam kalimat :
A. Tugu Siger, yang mempunyai tinggi lima puluh meter, dibangun pada tahun anggaran 20052006.
B. Dalam Alquran pada halaman 1260 termuat firman Tuhan yang menyampaikan perintah untuk berbuat baik
kepada sasama.
C. Rapat akan diadakan pada pukul 13.00 – 15.00 di Gedung 5.
D. Dana luncuran Anggaran Belanja dan Pendapatan Negara tahun 2000/2001 sebesar Rp. 15,5 trilliun baru
terlaksana sekitar tiga puluh persen.
E. Setelah harga minyak naik pada Oktober 2005, impor minyak terus menurun menjadi dua belas juta barel pada
Desember 2005, 8,2 juta barel pada Januari 2006, dan 6,4 juta barel untuk impor Februari 2006.

10. SPMB 2006


Penulisan kalimat berikut ini sesuai dengan aturan EYD, kecuali ....
A. Apabila diamati dengan cermat, dari sekian banyak uraian yang mengutarakan fungsi pertunjukan, yang paling
lengkap adalah fungsi seni pertunjukan yang dikemukakan oleh R.M. Soedarsono dalam bukunya berjudul
“Seni Pertunjukan Indonesia Di Era Globalisasi (1997)” yang diterbitkan oleh Gadjah Mada University Press,
Yogyakarta.
B. Kendatipun di dalam berkomunikasi manusia pada umumnya berinteraksi untuk membina kerja sama
antarsesamanya dalam rangka membentuk, mengembangkan, dan memainkan kebudayaannya dalam arti yang
seluas-luasnya, adakalanya manusia berselisih paham atau berbeda pendapat dengan yang lainnya.
C. Profesi seseorang, terutama profesi rendah dan diharamkan oleh agama, sering digunakan oleh pemakai bahasa
untuk mengumpat atau mengekspresikan kejengkelannya.
D. Dilambangkan sebagai matahari, Mundinglaya selalu memberikan penerangan kepada dunia pada waktu siang,
sedangkan Nyi Dewi Asri dilambangkan sebagai bulan yang memberikan kesejukan dan cahaya pada waktu
malam.
E. Kecenderungan yang muncul di kota-kota di Negara Dunia Ketiga yang makin metropolis adalah laju
pertumbuhan penduduk yang begitu pesat dan tidak bisa lagi diantipasi oleh daya dukung kota secara layak.

Bidang Studi BAHASA INDONESIA 45


KUMPULAN SOAL – SOAL UJIAN MASUK PERGURUAN TINGGI NEGERI

11. SPMB 2006


Penulisan kalimat berikut mengikuti aturan EYD, kecuali ....
A. Buku ini disusun untuk membantu mahassiswa yang akan mengambil mata kuliah Akuntansi Perusahaan pada
fakultas ekonomi.
B. Kami menulis diktat ini dengan maksud, agar mahasiswa lebih mudah mengikuti kuliah tentang Ekonomi
Pembangunan.
C. Akuntansi terbagi dalam beberapa bidang: akuntansi sosial, akuntansi pemerintahan, akuntansi perusahaan,
dan akuntansi organisasi nonprofit.
D. Diharapkan diktat yang berjudul Akuntansi Sosial ini bermanfaat bagi para peminat akuntansi pada umumnya
dan mahasiswa sekolah tinggi ilmu ekonomi pada khususnya.
E. Penjelasan dengan asetilena dilakukan dengan cara membakar bahan bakar gas asetilena dengan O2.

12. SPMB 2006


Penulisan kalimat yang seluruhnya sesuai dengan aturan EYD adalah ....
A. Kata feed-back ‘balikan’ sudah sering digunakan sebagai istilah
B. Bang Kadir sering disebut sebagai ‘Pahlawan’, ia sendiri tidak tahu sebabnya.
C. Karena postur tubuhnya Gakih mendapat julukan “Si Gendut”.
D. Kalau tidak diizinkan saya tidak akan pergi.
E. Esai Ratna yang berjudul Kesehatan Reproduksi dimuat di “Tempo”.

13. SPMB 2006


Sampai saat ini, India belum meratifikasi Perjanjian non-Proliferasi Nuklir (NPT).
Padanan yang tepat untuk kata meratifikasi dalam kalimat tersebut adalah
A. mengumumkan
B. mengesahkan
C. memperbaiki
D. merevisi
E. melaksanakan

14. SPMB 2006


Kelas akselerasi adalah untuk memenuhi kebutuhan siswa yang memiliki potensi dan bakat akademis luar biasa.
Istilah akselerasi dalam kalimat tersebut berarti
A. tambahan
B. percepatan
C. unggulan
D. peningkatan
E. khusus

15. SPMB 2006


dengan posisi di tepi pantai dan di .... tiga sungai besar Eropa, yaitu Sungai Rijn, Sungai Maas, dan Sungai
Schelde, Belanda mendapat .... dari laut dan sungai. Oleh sebab itu, di ..... raksasa ini dibuat banyak bendungan
guna menahan pasang air laut. Namun, hal itu masih belum membuat Belanda aman dari amukan air. Akhirnya,
..... proyek Delta Plan.
Kata-kata ragam keilmuan yang tepat untuk mengisi bagian yang rumpang dalam paragraf tersebut adalah
A. hulu, gangguan, teluk, disusunlah
B. hilir, marabahaya, delta,direncanakan
C. hilir, godaan, teluk, diprogramlah
D. hilir, ancaman, delta, dicanangkanlah
E. hulu,kerusakan, pantai, diproyeksikan

16. SPMB 2006


Sebelum mengikuti seleksi, setiap peserta wajib mengisi formulir pendaftaran yang telah disediakan oleh panitia.
Kata yang tepat untuk menggantkan kata formulir dalam kalimat tersebut adalah
A. angket
B. kisi-kisi
C. boring
D. daftar isian
E. biodata

Bidang Studi BAHASA INDONESIA 46


KUMPULAN SOAL – SOAL UJIAN MASUK PERGURUAN TINGGI NEGERI

17. SPMB 2006


Sepanjang sejarah peradaban manusia, selama itu pula kusta ada di dunia. Stigmatisasi sebagai manusia terkutuk
karena dihinggapi penyakit menular tak tersembuhkan ini hingga kini masih menjadi ganjalan utama dalam
memutus rantai penularan. Akibatnya, meskipun secara signifikan terjadi penurunan angka prevalensi, kasusu-
kasus baru selalu bermunculan.
Padana kata yang tepat dalam bahasa Indonesia untuk kata stigmatisasi pada kalimat kedua dalam paragraf
tersebur adalah ....
A. petanda
B. penanda
C. pertanda
D. penandaan
E. proses penandaan

18. SPMB 2006


Hampir setahun yang lalu pemerintah menyatakan tekad untuk memperbanyak pembangunan infrastruktur jalan
tol. Tahun 2005 ini hanya tersisa beberapa hari. Belum satu pun proyek yang ditawarkan dalam Indonesia
Infrastructure Summit 2005 dimulai pembangunannya. Pemerintah sudah menunjuk tiga belas investor untuk
menggarapnya. Namun, sebagian besar investor yang sudah ditunjuk pemerintah untuk menggarap proyek tersebut
ternyata belum juga merealisasikan proyek.
Bagian kalimat tetapi pembangunan infrastructure ternyata belum terlaksana dapat ditambahkan pada salah satu
kalimat dalam paragraf tersebut.
Bagian kalimat itu paling sesuai untuk ditambahkan pada kalimat.
A. pertama
B. kedua
C. ketiga
D. keempat
E. kelima

19. SPMB 2006


Meskipun tidak cukup mempunyai lahan untuk bahan baku etanol, negara-negara di Uni Eropa tidak ketinggalan
memanfaatkan bensin hijau. Pada tahun 2003 parlemen eropa menyetujui rancangan regulasi penganekaragaman
sumber energi. Mereka memasang target konsumsi biofuel untuk sektor transportasi sebesar 2 persen pada tahun
2005 ....
Kalimat yang benar dan baku untuk melengkapi titik-titik tersebut adalah
A. Jumlah ini diharapkan naik menjadi 5 persen pada lima tahun mendatang.
B. Jumlah ini, sesuai dengan harapan, naik menjadi 5 persen pada lima tahun mendatang.
C. Diharapkan jumlah ini naik menjadi 5 persen pada lima tahun mendatang.
D. Sehingga jumlah ini diharapkan naik menjadi 5 persen pada lima tahun mendatang.
E. Dan jumlah ini diharapkan naik menjadi 5 persen pada lima tahun mendatang.

20. SPMB 2006


Beberapa analis asing maupun domestik di pasar modal berpendapat, kesimpangsiuran susunan personalia
kabinet merupakan faktor utama yang menjadi penyebab indeks harga saham gabungan terpuruk. Sekalipun
demikian, sentimen negatif bursa regional yang berguguran turut memperparah keadaan pasar saham. Akibatnya,
indeks harga saham gabunganpun tidak mampu mengatasi keterpurukannya.
Dalam paragraf di atas, terdapat beberapa kesalahan ejaan dan pemilihan kata. Paragraf tersebut akan menjadi
paragraf yang baik jika direvisi dengan cara-cara sebagai berikut, KECUALI ....
A. gabunganpun (kalimat 3) ditulis gabungan pun.
B. maupun (kalimat 1) diganti dan.
C. Tanda koma (,) (kalimat 1) dihilangkan dan diganti bahwa.
D. Sekalipun demikian (kalimat 2) diganti selain itu,
E. Analis (kalimat 1) diganti analisis.

21. SPMB 2006


Subjudul teks utama berikut tepat untuk mengembangkan sebuah artikel yang berjudul “Ekspresi Budaya
Masyarakat Tutur Etnik dalam Nyanyian Rakyat”, KECUALI ....
A. Bentuk Verbal Nyanyian Rakyat sebagai Ekspresi Budaya
B. Wujud Nilai-nilai Budaya Etnik dalam Nyanyian Rakyat.
C. Kedudukan Nyanyian Rakyat di Kalangan Etnik.
D. Pola Interaksi Etnik dalam Nyanyian Rakyat.
E. Berbagai Kajian Nyanyian Rakyat Etnik.

Bidang Studi BAHASA INDONESIA 47


KUMPULAN SOAL – SOAL UJIAN MASUK PERGURUAN TINGGI NEGERI

22. SPMB 2006


Masalah remaja yang muncul biasanya berhubungan dengan kehidupan remaja dalam keluarga dan sekolah.
Selain itu, remaja sendiri juga menimbulkan berbagai masalah. Misalnya, ketegangan psikis remja yang ditandai
dengan tingginya tingkat emosi remaja dan juga perubahan fisik yang terlalu cepat. Kondiri dalam diri remaja
sendiri sudah menimbulkan berbagai masalah yang cukup pelik. Kondisi lingkungan yang tidak mendukung juga
menyebabkan masalah remaja itu semakin bertumpuk.
Kalimat yang tepat untuk mengawali teks tersebut adalah
A. Sumber masalah remaja meliputi sumber dari dalam diri remaja dan lingkungannya.
B. Masalah remaja berhubungan dengan berbagai hal.
C. Masalah remaja bersumber dari ketegangan psikis remaja.
D. Sumber masalah remaja menyebabkan perubahan fisik dan ketegangan.
E. Sumber masalah remaja mengakibatkan bertumpuknya masalah yang dihadapi remaja.

23. SPMB 2006


Kalau kita perhatikan secara seksama, di negeri kita tercinta ini banyak pejabat yang tidak istikamah. Ada sebuah
contoh realita kecil di lingkungan kita, yakni adanya razia, terutama narkoba dan minuman keras. Kalau memang
penyelenggara negeri ini cinta dan sayang kepada rakyat dan bangsanya, seharusnya mereka bersikap tegas dan
bijaksana. Mereka seharusnya berani melarang segala bentuk prosuk, baik narkoba maupun minuman keras.
Pemerintah semestinya tidak lagi memberikan izin kepada perusahaan narkoba dan minuman keras untuk tetap
beroperasi.
Teks tersebut berisi
A. tuntutan agar pemerintah menutup perusahaan narkoba dan minuman keras.
B. Saran agar pemerintah bersikap tegas dan bijaksana dalam melakukan tindakan.
C. Protes kepada pejabat yang tidak istikhamah dalam menangani masalah bangsa
D. Sikap kekesalan penulis terhadap pejabat yang selaltu tidak tuntas dalam menangani kasus.
E. Hasil pengamatan penulis terhadap kerja pemerintah yang hanya setengah-setengah.

24. SPMB 2006


Indonesia merupakan negara yang memiliki potensi budidaya rumput laut seluas 1,2 juta hektare atau terbesar di
dunia. Akan tetapi, yang tergarap baru sekitar 10 persennya. Penyebabnya adalah usaha budidaya rumput laut
baru berkembang sejak tahun 1996 dan mulai meningkat pada tahun 1999. Kenyataan itu adalah bukti pengabaian
pemerintah sebelumna terhadap sektor kelautan dan perikanan. Kendati demikian, dari tahun ke tahun usaha
budidaya rumput laut di Indonesia cenderung meningkat. Jika pada tahun 1999 volume produksi nasional hanya
157.232 ton, pada tahun 2003 naik menjadi 285.653 ton. Sementara itu, volume ekspor pada tahun 1999, yang
hanya 25.084 ton atau senilai 40.162 ton atau senilai 20,511 juta dolar AS. Andaikata 50 persen dari potensi
rumput laut Indonesia dapat digarap optimal, dengan volume produksi setiap hektare 16 ton per tahun, akan
diperoleh hasil sebesar 9,6 juta ton. Jika harga dipasar dunia Rp. 4,5 juta per ton, total devisa yang akan diraih
mencapai Rp. 40 triliun.
Berdasarkan teks tersebut, pernyataan berikut ini benar, KECUALI
A. Potensi rumput laut di Indonesia yang sudah digarap baru sekitar 120 ribu hektare.
B. Dari tahun 1999 sampai dengan tahun 2003 produksi rumput laut nasional meningkat rata-rata sekitar 32.105
ton per tahun.
C. Harga rumput laut di pasar dunia pada tahun 1999 lebih rendah daripada tahun 2003.
D. Lebih dari 15 persen dari total produksi rumput laut nasional pada tahun 1999 dieskpor.
E. Kurang dari 15 persen dari total produks rumput laut nasioanal pada tahun 2003 diekspor.

25. SPMB 2006


Membangun infrastruktur, apa pun jenisnya, tidak semudah membalik telapak tangan. Selain membutuhkan dana
besar, pembangunan itu juga memerlukan waktu panjang untuk dapat beroperasi, minimal lima tahun. Sementara
itu, perusahaan yang bergerak dalam sektor infrastruktur memiliki keterbatasan modal. Untuk mewujudkannya,
dibutuhkan sumber pembiayaan agar perusahaan dapat berutang.
Pokok pikiran dalam paragraf tersebut adalah
A. biaya pembangunan proyek infrastruktur
B. syarat pembangunan proyek infrastruktur
C. ciri-ciri pembangunan proyek infrastruktur
D. sumber biaya proyek infrastruktur
E. waktu pembangunan proyek infrastruktur

Bidang Studi BAHASA INDONESIA 48


KUMPULAN SOAL – SOAL UJIAN MASUK PERGURUAN TINGGI NEGERI

SOAL BAHASA INDONESIA TAHUN 2005

UM UGM

Ethiopia adalah negara tertua di dunia. Negeri itu telah terbentuk sejak sebelum 4900 tahun yang lalu. Ethiopia terletak
di Afrika timur, tepatnya disebelah selatan pintu masuk Laut Merah (Terusan Suez). Negara tetangganya adlah Kongo,
Sudan, Somalia, dan Djiboti. Wilayah Ethiopia cukup luas, hampir dua kali luas Pulau Sumatra, Negeri ini sering
dilanda cuaca buruk dan musim kering berkepanjangan. Ethiopia adalah negara peternakan kelas 1 dunia. Jumlah sapi
dan kudanya menempati urutan ke10 dunia dengan 2,6 juta ekor sapi dan 1,5 juta kuda. Ethiopia juga merupakan
negara pengekspor kopi terbesar keenam dunia. Pada tahun 1985, akibat kemarau yang sangat panjang, negeri ini
menggemparkan dunia karena dilanda krisis pangan dan penduduknya menderita kelaparan hebat.

1. UM UGM 2005
Ide pokok bacaan di atas adalah ....
A. berdirinya negara Ethiopia
B. letak negara Ethiopia
C. usia negara Ethiopia
D. ciri-ciri geografis negara Ethiopia
E. keadaan alam negara Ethiopia

2. UM UGM 2005
Pernyataan berikut berhubungan, baik secara eksplisit maupun implisit, dengan bacaan di atas kecuali ....
A. Ethiopia menempati peringkat 10 dunia dalam hal ternak karena kualitasnya yang baik.
B. Kekeringan merupakan salah satu masalah serius yang dihadapi Ethiopia.
C. Ethiopia termasuk negara penghasil kopi dunia.
D. Krisis pangan yang terjadi di Ethiopia pernah menggemparkan dunia.
E. Jumlah ternak di Ethiopia cukup banyak.

3. UM UGM 2005
Agar hubungan antarkalimat menjadi jelas, kata atau ungkapan penghubung yang tepat antara kalimat pertama dan
kedua adalah ....
A. Dalam hal ini.
B. Karena.
C. Menurut catatan sejarah.
D. Oleh karena itu.
E. Sebab.

4. UM UGM 2005
Penulisan kata dengan huruf miring pada kalimat-kalimat di bawah, kecuali ....
A. Kata ateis berasal dari penggabungan a dan teis.
B. Fonem kedua kata akhir adalah kh.
C. Perhitungan yang rumit itu dilakukan dengan kalkulator.
D. Habis Gelap Terbitlah Terang adalah buku ciptaan R.A Kartini.
E. Dia berlangganan koran The Jakarta Post.

5. UM UGM 2005
Kata-kata yang dicetak miring dalam kalimat berikut yang penulisannya mengikuti ejaan resmi :
A. Menurut analisa para pakar sistem itu sudah tidak layak untuk diterapkan lagi.
B. Sudah enam tahun ia tidak naik pangkat karena kondite dan perilakunya buruk.
C. Sistem koordinasi antarbagian di perusahaan itu sangat baik.
D. Kongres itu tidak dapat dimulai karena dutabesar yang membukanya belum datang.
E. Jadual keduarnya produk tetap ditepati agar tidak mengecewakan konsumen.

6. UM UGM 2005
Pasangan kata jual beli dalam kalimat Sekarang dia membuka usaha jual beli mobil bekas, tidak setipe dengan
pasangan kata berikut :
A. tarik ulur
B. simpan pinjam
C. serah terima
D. bongkar pasang
E. tukar tambah

Bidang Studi BAHASA INDONESIA 49


KUMPULAN SOAL – SOAL UJIAN MASUK PERGURUAN TINGGI NEGERI

7. UM UGM 2005
Kalimat yang mengandung bentukan kata yang salah terdapat pada :
A. Perubahan pola makanan tradisional menjadi modern yang umumnya rendah serah dan tinggi lemak harus
dibayar dengan resiko terkena penyakit degeneratif.
B. Untuk mengurangi risiko terkena penyakit degeneratif, kita harus merubah kebiasaan makan-makanan yang
kaya lemak ke makanan kaya serat.
C. Rata-rata penduduk Indonesia mengkonsumsi serat 10,5 gr per hari.
D. Jumlah rata-rata konsumsi serta penduduk Indonesia baru sepertiga dari konsumsi serat yang dianjurkan.
E. Kebutuhan serat 25-35 gr per hari dapat dipenuhi apabila kita mengonsumsi 23 porsi nasi dari beras tumbuk
yang masih ada kulit arinya, 1-2 porsi biji-bijian, dan 6 porsi buah dan sayur.

8. UM UGM 2005
...., .... bahwa air yang dikonsumsi masyarakat di daerah itu mengandung sejumlah zat berbahaya akibat
pencemaran.
Bagian kalimat yang tepat untuk melengkapi kalimat tersebut adalah ....
A. Setelah penelitian , diketahui
B. Setelah dilakukan penelitian, mengetahui
C. Setelah meneliti diketahui
D. Setelah diteliti, diketahui

9. UM UGM 2005
Di antara pernyataan di bawah ini yang dapat disebut kalimat adalah ....
A. Karena modal yang ada di bank terbatas sehingga tidak semua pengusaha kecil memperoleh kredit.
B. Di dalam keputusan itu menunjukkan kebijaksanaan yang dapat menguntungkan masyarakat luas.
C. Pembenahan eksternal dengan melibatkan secara langsung lembaga informal di lingkungan daerah asal migran
dalam memasyarakatkan program.
D. Pada kejadian itu sebagian suporter Maluku melempari botol dan kursi lalu menyerbu ke daerah sekitar arena
tinju.
E. Pengeluaran biaya yang sangat besar hanya untuk sebuah pesta ulang tahun merupakan suatu pemborosan.

10. UM UGM 2005


Di bawah ini adalah kalimat-kalimat yang rancu, kecuali ....
A. Untuk pembangunan gedung itu menghabiskan biaya seratus juta rupiah.
B. Pelaksanaan pembangunan jembatan dilaksanakan selama satu tahun.
C. Struktur bahasa Indonesia berbeda dengan struktur bahasa asing.
D. Kepada Bapak dan Ibu sekalian harap mengambil snack dan minuman di ruang istirahat.
E. Dalam rapat itu membicarakan biaya SPP.

11. UM UGM 2005


Deretan kata yang merupakan kalimat adalah ....
A. Bunga bank yang selalu mengalami kenaikan cukup meresahkan
B. Gedung bersejarah yang senantiasa dikunjungi wisatawan asing dan domestik.
C. Moseum perjuangan yang telah direnovasi dengan biaya cukup besar.
D. Biaya praktik lapangan ditentukan oleh universitas dan telah mendiskusikannya dengan para mahasiswa yang
menyetujuinya
E. Beasiswa yang diberikan kepada para mahasiswa berprestasi.

12. UM UGM 2005


Dalam berwiraswasta memerlukan kreativitas untuk menyiasati apa yang ada dan melahirkan peluang bisnis.
Kalimat tersebut menjadi benar apabila ....
A. kata memerlukan diubah menjadi diperlukan
B. kata dalam diganti dengan kata dengan
C. kata memerlukan diubah menjadi diperlukan
D. kata kreativitas ditulis kreatifitasi
E. kata menyiasati diubah menjadi mensiasati

Bidang Studi BAHASA INDONESIA 50


KUMPULAN SOAL – SOAL UJIAN MASUK PERGURUAN TINGGI NEGERI

13. UM UGM 2005


Kalimat yang menggunakan ragam baku adalah ....
A. Di Eropa dan AS terdapat banyak kesempatan untuk mengembangkan ilmu pengetahuan.
B. Di wilayah kami setiap malam mengadakan koordinasi keamanan.
C. Bagi yang belum mendaftarkan diri diharap menghubungi bagian personalia.
D. Di desa yang terletak di pinggir sungai itu setiap tahun dilanda banjir.
E. Dalam pemeriksaan itu menyimpulkan bahsa pembunuhan sadis tidak selalu berhubungan dengan kelainan
jiwa.

14. UM UGM 2005


Jika ditempatkan di akuarium yang .... nya tepat, louhan akan ... pendar cahaya dari titik-titik mutiara di ... nya.
Kata-kata yang tepat untuk mengisi tempat-tempat kosong dalam kalimat itu ialah ....
A. pengairan, menyiratkan, sirip
B. penerangan, mengeluarkan, badan
C. pencahayaan, memancarkan, sisik
D. temperatur, membersihkan, ekor
E. ukuran, memantulkan, tubuh.

15. UM UGM 2005


.... semangat berkoperasi merupakan salah satu cara efektif untuk .... rentenir. Apalagi jika semangat itu .... oleh
perbankan, misalnya dengan memberikan kredit ... tanpa ... dengan proses ... yang mudah dan cepat.
A. tumbuh kembangnya, memerangi, ditanggapi, murah, jaminan, pencairan
B. menumbuhkembangkan, menghapus, didukung, lunak, agunan, pencairan
C. membudayakan, memusuhi, ditopang, besar, agunan, administrasi
D. membudidayakan, menghentikan, terdukung, lunak, agunan, administrasi
E. menumbuh dan mengembangkan, menghapus, disambut, cepat, birokrasi, pengurusan

16. UM UGM 2005


Gerak daratan itu tidak akan pernah usai sepanjang usia bumi ini.
Kalimat ini mengandung maksud bahwa ....
A. usia bumi ini akan berakhir ketika daratan di permukaan bumi berhenti bergerak.
B. daratan di muka bumi bergerak sepanjang luas permukaan bumi.
C. gerak daratan di muka bumi akan berakhir setelah mengelilingi seluruh permukaan bumi
D. usia bumi ini akan bertambah panjang apabila daratan di permukaannya masih terus bergerak.
E. daratan di muka bumi akan terus bergerak selama bumi ini masih ada.

17. UM UGM 2005


Hujan yang turun empat hari terakhir di wilayah Gunung Kidul mampu .... 41.752 hektare lahan padi yang semula
sudah layu. Bahkan seluruh tanaman pangan dapat ... kembali. Di wilayah utara, seperti Kecamatan Semin,
Ngawen, dan Nglipar sudah .... panen jagung dan kedelai.
Bentuk kata yang tepat untuk mengisi titik-titik di atas adalah ....
A. menghancurkan, merusak, berakhir
B. menyelamatkan, menghijau, mulai
C. membanjiri, ditanami, dilakukan
D. menggilas, menguning, disibukkan
E. menyelamatkan, dirusak, digalakkan

18. UM UGM 2005


(1) Teratai ternyata tidak hanya merupakan tanaman hias, tetapi juga merupakan tanaman penghasil makanan. (2)
Biji buah teratai merupakan sumber karbohidrat, protein, dan mineral. (3) Kandungan gizi teratai yang sudah diolah
menjadi tepung tidak kalah baik dengan beras maupun dengan terigu. (4) Sehingga dapat dijadikan sebagai pangan
alternatif nonberas untuk membuat makanan seperti apem, jenang, roti cincin, kue lapis, bipang, dan sebagainya.
(5) Selain biji bunganya, umbi teratai yang menyerupai umbi dapat direbus untuk dimakan.
Dalam paragraf di atas terdapat 5 pernyataan, satu di antaranya bukan kalimat, yaitu ....
A. pernyataan (1)
B. pernyataan (2)
C. pernyataan (3)
D. pernyataan (4)
E. pernyataan (5)

Bidang Studi BAHASA INDONESIA 51


KUMPULAN SOAL – SOAL UJIAN MASUK PERGURUAN TINGGI NEGERI

19. UM UGM 2005


Dalam rapat yang menghadiri para gubernur kemarin memutuskan bahwa UMR baru dapat dilaksanakan karena
beberapa pertimbangan.
Rangkaian kata di atas akan menjadi kalimat bauku jika dilakukan hal berikut :
A. Kata dalam dihilangkan dan kata memutuskan diubah menjadi diputusan
B. Kata menghadiri diubah menjadi menghadirkan
C. Kata memutuskan diganti disepakati
D. Kata dalam dihilangkan, kata mengadiri diubah menjadi dihadiri, dan kata memutuskan diubah menjadi
diputuskan.
E. Kata dalam dihilangkan dan kata menghadiri diganti dihadiri.

20. UM UGM 2005


Pernyataan yang membentuk kalimat yang baku adalah ....
A. Pada kesempatan itu menetapkan berlakunya tarif baru di bidang komunikasi
B. Munculnya anggapan bahwa gejala tersebut adalah gejala yang tidak mudah diamati
C. Perkembangan pedangan kaki lima pada saat sulit mencari pekercaan.
D. Karena penyakit jenis ini memang tidak mengnal status sosial ekonomi penderita.
E. Dalam menetapkan tarif dasar listrik, pemerintah menggunakan berbagai pertimbangan.

SPMB

1. SPMB 2005
Penulisan unsur serapan yang bercetak miring pada kalimat-kalimat berikut benar, KECUALI :
A. Salinan STTB Anda harus dilegalisir agar sah.
B. Hak asasi manusia harus dijunjung tinggi.
C. Pasien yang datang itu berasal dari kalangan kurang mampu.
D. Reumatik neneknya akhir-akhir ini sering kambuh.
E. Hipotensi itu telah dapat dibuktikan kebenaranya.

2. SPMB 2005
(1) Tiga Perusahaan yang berkecimpung di dunia teknologi informasi, bergandengan tangan untuk memberikan
solusi jaringan teknologi bagi pengusaha kecil dan menengah, (2) Ketiga perusahaan tersebut itulah Microsoft,
Cisco, dan Sun, (3) Paket solusi tiga kekuatan yang berharga 37.000 dolar AS tersebut diperuntukan bagi usaha
manufaktur yang banyak tersebar di berbagai kota, (4) Paket yang ditawarkan meliputi aplikasi Microsoft business
solution untuk finansial, (5) Paket tersebut dilengkapi dengan gigabit switch cisco dan dijalankan di atas Server
Sun x86.
Dalam kutipan di atas, terdapat kata bentukan yang tidak baku. Kata bentukan tersebut terdapat dalam :
A. kalimat 1
B. kalimat 2
C. kalimat 3
D. kalimat 4
E. kalimat 5

3. SPMB 2005
Menulis merupakan ketrampilan yang paling sulit dikuasai. Dalam proses menulis, banyak proses pembelajaran
pemakaian ejaan sesuai dengan kaidah bahasa yang dianut, pembelajaran penggunaan diksi maupun frase yang
mengungkapkan pikiran.
Dalam kutipan di atas, terdapat kesalahan tata tulis. Kesalahan yang dimaksud adalah sebagai berikut, KECUALI :
A. kesalahan penulisan diantaranya.
B. kesalahan penggunaan tanda titik koma (;)
C. kesalahan pemilihan kata maupun.
D. kesalahan penggunaan tanda koma (,) sebelum kata hubung dan.
E. kesalahan penulisan kata ketrampilan.

4. SPMB 2005
Kelas akselerasi adalah untuk memenuhi kebutuhan siswa yang memiliki potensi dan bakat akademis luar biasa.
Istilah akselerasi dalam kalimat di atas berarti :
A. tambahan
B. percepatan
C. unggulan
D. peningkatan
E. khusus

Bidang Studi BAHASA INDONESIA 52


KUMPULAN SOAL – SOAL UJIAN MASUK PERGURUAN TINGGI NEGERI

5. SPMB 2005
Kalimat Tentang pengembangan Yogyakarta sebagai kota pendidikan terkemuka mengundang makna mendorong
pertumbuhan dan perubahan kota Yogyakarta sebagai pusat pendidikan atau sebagai masyarakat belajar tidak
baku. Kalimat tersebut akan menjadi baku apabila :
A. ditambah kata yang di antara kata pendidikan dan terkemuka.
B. kata mengandungdiubah menjadi terkandung.
C. kata tentang diganti dengan kata dalam.
D. kata Yogyakarta yang kedua dihilangkan.
E. kata tentang dihilangkan.

6. SPMB 2005
1) Pertumbuhan industri tergantung pada tenaga listrik dan produk kimia.
2) Produk kimia cenderung menjadi sumber pencemaran lingkungan.
3) Kita harus menjaga kelestarian lingkungan.
Rangkaian yang tepat dari ketiga kalimat tersebut adalah :
A. Pertumbuhan industri bergantung pada tenaga listrik dan produk kimia yang cenderung menjadi sumber
pencemaran lingkungan dan kita harus menjaga kelestariannya.
B. Kita harus menjaga kelestarian lingkungan karena pertumbuhan industri cenderung pada tenaga listrik dan
produk kimia yang cenderung menjadi sumber pencemaran lingkungan.
C. Produk kimia cenderung menjadi sumber pencemaran lingkungan sehingga kita harus menjaga kelestarian
lingkungan karena pertumbuhan industri bergantung pada tenaga listrik dan produk kimia.
D. Pertumbuhan industri tidak hanya bergantung pada tenaga listrik, tetapi juga bergangung pada produk kimia
yang cenderung menjadi sumber pencemaran lingkungan sehingga kita harus menjaga kelestariannya.
E. Kita harus menjaga kelestarian lingkungan yang diakibatkan oleh pertumbuhan industri yang bukan saja
bergantung pada tenaga listrik, melainkan juga produk kimia cenderung menjadi sumber.

7. SPMB 2005
Mobil kakak ipar yang baru dibeli teman saya.
Kalimat di atas akan menjadi kalimat yang jelas maksudnya jika direvisi sebagai berikut :
A. Mobil kakak ipar saya yang baru, dibeli teman saya.
B. Mobil baru kakak ipar saya dibeli teman saya.
C. Mobil kakak ipar saya yang dibeli teman saya.
D. Mobil kakak ipar saya, yang baru, dibeli teman saya.
E. Mobik kakak ipar yang baru dibeli teman saya.

8. SPMB 2005
Kalimat berikut yang strukturnya benar adalah :
A. Meningkatkan kualitas pendidikan melalui penyusunan standar kompetensi nasional berdasarkan bidang-
bidang keahlian.
B. Membentuk jaringan komunikasi antarguru sebagai wahana untuk meningkatkan proses pembejaran.
C. Peningkatan mutu kualitas guru dengan melaksanakan pendidikan dan pelatihan merupakan langkah penting
untuk perbaikan mutu pendidikan.
D. Melalui peran serta masyarakat menjadikan ciri konsep pendidikan pada era otonomi.
E. Dengan mengembalikan pendidikan kepada masyarakat mengharapkan akan memberi peluang kepada lembaga
tesebut semakin ditingkatkan perannya.

9. SPMB 2005
Masyarakat yang memiliki bahasa dan menggunakan bahasa disebut masyarakat bahasa. Masyarakat desa adalah
masyarakat bahasa, sekelompok manusia yang ditandai oleh interaksi teratur dengan menggunakan isyarat-isyarat
verbal dan terpisahkan dari kelompok-kelompok lain di luar masyarakat itu. Dalam berbahasa secara tunggal atau
pun majemuk, masyarakat desa dapat dikaji dari aturan-aturan pemakaian bahasanya. Mereka pada umumnya
terikat oleh kesamaan kaidah-kaidah berbahasa untuk menghasilkan dan menafsirkan tuturan, dan kaidah-kaidah
untuk ragam-ragam bahasa.
Kalimat kedua paragraf di atas termasuk kalimat yang strukturnya tidak jelas. Agar menjadi kalimat jelas
maknanya, kalimat tersebut harus ditambah dengan kata :
A. merupakan
B. adalah
C. di mana
D. yang mana
E. yakni

Bidang Studi BAHASA INDONESIA 53


KUMPULAN SOAL – SOAL UJIAN MASUK PERGURUAN TINGGI NEGERI

10. SPMB 2005


Sejak lahirnya konsep pemikiran baru dalam ilmu kedokteran, yang dicetuskan oleh Profesor Linus Pauling, yakni
tentang Ortomolecular medicine yang dasarnya adalah studi biologi molekuler sebagai sains dasar, penelitian
medis diarahkanpada molekul-molekul yang secara normal biologis fisiologis adalah dalam tubuh manusia.
Inti kalimat panjang tersebut adalah :
A. konsep pemikiran baru dicetuskan oleh Profesor Linus Pauling.
B. ortomolecular medicine adalah sains dasar.
C. ortomolecular medicine dasarnya adalah studi biologi.
D. penelitian medis diarahkan pada molekul.
E. biologi molekuler merupakan sains dasar.

11. SPMB 2005


Pernyataan “makmur dalam keadilan dan adil dalam kemakmuran” hanyalah sebagai slogan yang dalam
kenyataannya sulit untuk direlisasikan.
Kata slogan dalam kalimat di atas memiliki padanan dengan kata :
A. reklame.
B. propaganda.
C. wawasan.
D. provokasi.
E. semboyan.

12. SPMB 2005


Segala hal yang dilakukan selama ini sebenarnya tidak konsisten.
Kata konsisten dalam kalimat tersebut berarti :
A. taat asas
B. ada gunanya
C. berbahaya
D. penting
E. tepat

13. SPMB 2005


Setelah terjadi kerusuhan di Poso Sulawesi Tengah, polisi segera membentuk sebuah tim untuk mengetahui
penyebab kerusuhan yang diperkirakan akibat provokasi pihak ketiga.
Kata yang tepat untuk melambangkan kegiatan yang dilakukan oleh kepolisian adalah :
A. pengamanan
B. penelitian
C. penyelidikan
D. pengamatan
E. pemeriksaan

14. SPMB 2005


Pekerjaan analisis ini dimaksudkan untuk mengenali retorika penulis Indonesia, yang terwujud dalam artikel
(khususnya yang diterbitkan di Harian Kompas). Retorika yang dimaksud dalam tulisan ini adalah pola berpikir
dalam mengungkapkan gagasan untuk menghasilkan karya tulis dalam bentuk artikel. Dengan melakukan analisis
artikel Harian Kompas, diharapkan dapat dikenal pola retorika Indonesia.
Berdasarkan isinya, kutipan di atas merupakan penggalan dari karya ilmiah, khususnya penggalan dari bagian :
A. kata pngantar.
B. pendahuluan.
C. kerangka teori.
D. pembahasan.
E. penutup.

Bidang Studi BAHASA INDONESIA 54


KUMPULAN SOAL – SOAL UJIAN MASUK PERGURUAN TINGGI NEGERI

15. SPMB 2005


Novelis Pramoedya Ananta Toer mengimbau semua penulis sastra mendokumentasikan karya-karya sastra di
Tanah Air. Dokumentasi sastra dapat menumbuhkan karakter dan kebanggaan sebagai bangsa. Selain itu,
dokumentasi sastra yang rapi juga memacu lahirnya tulisan sastra yang kreatif dan inovatif. Menulis sastra adalah
dialog dengan diri sendiri dan dialog dengan publik di seluruh dunia yang membacanya. Meskipun pentingnya
dokumentasi sudah disadari berbagai pihak, sayang sekali dokumentasi sastra belum menjadi bagian dari tradisi
bangsa kita.
Kalimat sumbang pada paragraf di atas terdapat pada :
A. kalimat 1
B. kalimat 2
C. kalimat 3
D. kalimat 4
E. kalimat 5

16. SPMB 2005


Menteri Perdagangan mengatakan bahwa masalah penting yang terkait dengan perdagangan, khususnya
perdagangan luar negeri, adalah bagaimana meningkatkan daya saing produk ekspor. Kunci daya saing tersebut
adalah meningkatkan produktivitas produk. Secara sektoral, persoalan peningkatan produktivitas itu memang
berbeda-beda. Namun, secara umum masalah yang timbul dari dunia usaha cukup banyak. Birokrasi yang
berbelit-belit dapat menimbulkan ekonomi biaya tinggi.
Kesetalian hubungan antarkalimat dalam paragraf di atas kurang lancar. Paragraf tersebut akan menjadi padu jika
direvisi sebagai berikut :
A. namun (kalimat keempat) dihilangkan.
B. bagaimana (kalimat pertama) diganti dengan upaya.
C. pada awal kalimat kelima ditambahkan di antaranya adalah.
D. kunci daya saing (kalimat kedua) diubah menjadi kunci meningkatkan daya saing.
E. secara sektoral (kalimat ketiga) diposisikan sebelum kata memang.

17. SPMB 2005


Perhatikan paragraf berikut.
Kanker perut kini masih menjadi penyebab kematian t er tinggi kedua…, dalam beberpa dekade terkahir ini
jumlah kasusnya cenderung menurun. … mungkin terjadi karena tingginya konsumsi buah-buahan dan sayuran
segar, serta berkurangnya kandungan garam dalam makanan akibat proses pendinginan. … di negeri ini kasus
kandungan akibat kanker perut masih menjadi masalah utama yang harus diatasi.
Agar paragraf itu menjadi susunan yang padu, tempat-tempat kosong itu harus diisi dengan kata-kata perangkai
yang berfungsi sebagai penghubung antarkalimat. Kata-kata perangkai itu adalah :
A. namun, hal itu akan tetapi.
B. meskipun demikian, oleh sebab itu, dengan demikian.
C. sedangkan, akibatnya, jadi.
D. namun, akibatnya, jadi.
E. adapun, namun, bahkan.

18. SPMB 2005


Kesadaran masyarakat mengenai masalah lingkungan sudah mulai tumbuh … tingkat kesadaran yang ada belum
cukup tinggi … mempengaruhi perilaku mereka … untuk menjadi motivasi yang kuat yang dapat melahirkan
tindakan nyata dalam usaha swadaya perbaikan hidup. Gerakan swalayan masyarakat dalam penanganan
masalah lingkungan hidup, di daerah perkotaan … perdesaan, masih harus terus lebih dikembangkan.
Kata-kata berikut ini yang paling tepat untuk mengisi bagian paragraf yang rumpang ialah :
A. meskipun, dalam, ataupun, baik.
B. meskipun, untuk, dan, maupun.
C. meskipun, untuk, dalam, apalagi.
D. sehingga, untuk, dalam, apalagi.
E. tetapi, dalam, dan, maupun.

19. SPMB 2005


Sehubungan telah diterapkan dalam GBHN, bahwa pembangunan pendidikan dititikberatkan pada peningkatan
setiap jenjang dan jenis pendidikan.
Kalimat di atas dapat diperbaiki dengan menghilangkan :
A. sebagaimana
B. telah
C. dalam
D. bahwa
E. pada
Bidang Studi BAHASA INDONESIA 55
KUMPULAN SOAL – SOAL UJIAN MASUK PERGURUAN TINGGI NEGERI

20. SPMB 2005


Pengembangan industri sangat penting karena industri dapat memberikan efek ganda yang lebih besar. Jika
industri otomotif, elektronik, dan motor berkembang, industri komponen pun akan meningkat. Kenaikan itu dengan
sendirinya akan menciptakan ribuan tenaga kerja baru di sektor jasa, seperti bengkel, distributor, dan pedagang
prdoduk komponen. Pada akhirnya, itu merupakan peluang ekonomi baru bagi masyarakat.
Pola pengembangan paragraf di atas adalah :
A. sebab-akibat-akibat.
B. sebab-akibat-sebab.
C. akibat-sebab-akibat.
D. akibat-akibat-sebab.
E. sebab-sebab-akibat.

21. SPMB 2005


Usaha bididaya ikan tunas semakin marak dilakukan di Australia dan Afrika Selatan. Hasilnya mulai masuk ke
pasar dunia. Bahkan, telah mengusai sekitar 20 persen dari total kebutuhan ikan tuna di Jepang. Mereka
memprediksikan bahwa permintaan ikan tuna hasil budidaya cenderung meningkat. Pasar ikan tuna dunia kini
lebih meminta yang masih segar atau hidup,bukan ikan tuna yang dibekukan. Itu berarti, ikan tersebut tidak
mungkin diperoleh dari penangkapan, melainkan dari hasil budidaya ikan tuna. Untuk itu, negara-negara seperti
Australia serta Afrika Selatan telah menangkap gelagat pasar dunia tersebut. Kedua negara tersebut
membudidayakan ikan tuna secara besar-besaran. Hal itu, menjadi ancaman serius bagi negara-negara produsen
ikan tuna dari hasil penangkapan, seperti Indonesia.
Ide pokok paragraf di atas adalah :
A. Australia serta Afrika Selatan telah menangkap gelagat pasar dunia.
B. Australia serta Afrika Selatan membudidayakan ikan tuna karena menangkap gelagat pasar dunia.
C. Kedua negara membudidayakan ikan tuna secara besar-besaran karena pasar dunia lebih meminta yang masih
segar atau hidup, bukan ikan tuna yang dibekukan.
D. Australia dan Afrika Selatan merupakan ancaman negara-negara produsen ikan tuna dari hasil penangkapan.
E. Australia dan Afrika Selatan merupakan ancaman produsen ikan tuna Indonesia.

22. SPMB 2005


Aksi penjarahan di kota-kota besar makin menggila. Setelah beberapa daerah di Pulau Jawa teratasi, kini giliran
di luar Jawa mulai marak. Penjarahan sering diikuti dengan aksi kekerasan yang berdampak pula kekacauan
masyarakat. Penggalan berita di atas menggunakan urutan pengembangan dengan menggunakan pertanyaan.
A. apa, mengapa, di mana.
B. siapa, di mana, kapan.
C. mengapa, siapa, apa.
D. mengapa, di mana, bagaimana.
E. apa, di mana, bagaimana.

23. SPMB 2005


Inovasi sekolah dengan melaksanakan Manajemen Peningkatan Mutu Berbasis Sekolah (MPMBS) semakin
meningkat hasil yang menggembirakan. Data yang terhimpun pada Direktorat PLP dengan sampel 1000 sekolah
dalam pelaksanaan selama tahun 1999, 2000, dan 2001 menunjukkan bahwa hasilnya : 2% tergolong istimewa,
25% amat baik, 55% baik, 15% cukup, dan 3 % kurang. Berpangkal dari hasil yang realitis tersebut, akhirnya
MPMBS ditetapkan menjadi salah satu pola meningkatkan mutu dan memajukan sekolah yang berlaku secara
nasional. Dalam MPMBS tersebut, terdapat dua komponen yang sangat menonjol, yakni kemandirian sekolah dan
partipasi masyarakat dalam mendukung program-program yang disusun oleh sekolah. Data yang kelihatan jelas
dalam pengamatan bersama, kalau sekolah bisa melakukan upaya menstimulasi atau memberi semacam umpan
agar masyarakat tergerak bahkan terpancing untuk selanjutnya secara aktif berpartisipasi bantuan yang diperoleh
berupa dana, tenaga, atau sesuatu yang lain akan menjadi berlipat ganda jumlahnya.
Masalah berikut ini yang dibahas dalam kutipan di atas adalah :
A. persentase keberhasilan pelaksanaan MPMBS dari tahun ke tahun.
B. upaya-upaya sekolah daam meningkatkan mutu melalui MPMBS.
C. berbagai usaha yang dilakukan sekolah dalam melaksanakan MPMBS.
D. dukungan MPMBS dalam meningkatkan kualitas pembelajaran.
E. cara sekolah menstimulasikan masyarakat dalam menjalankan MPMBS.

Bidang Studi BAHASA INDONESIA 56


KUMPULAN SOAL – SOAL UJIAN MASUK PERGURUAN TINGGI NEGERI

24. SPMB 2005


Tren masyarakat dunia saat ini adalah kembali ke alam. Tren itu dapat dijadikan motivasi bagi pelaut untuk
kembali mengelola pertanian y ang berbasis faktor alam. Harga pupuk yang semakin melambung dapat digantikan
dengan pupuk kompas atau pupuk kotoran hewan ternak. Pencegahan hama dapat dilakukan melalui rantai
makanan sehingga organisme tertentu menjadi pengurai organisme lain. Apabila semua itu dapat dilakukan,
niscaya petani tidak lagi direpotkan dengan mahalnya pupuk yang terkadang malah langka dan oleh hama
tanaman yang semakin resisten terhadap insektisida. Dengan kembali ke alam, ketergantungan terhadap
penunjang pertanian, seperti pupuk, selama ini dapat diatasi melalui bahan yang ada di sekitar petani dan bersifat
alamiah.
Pernyataan berikut ini yang sesuai dengan isi paragraf di atas adalah :
A. Pertanian saat ini tidak lagi menggunakan pupuk kimia buatan pabrik.
B. Sampai saat ini, ketergantungan petani pada penunjang pertanian semakin tinggi.
C. Walaupun tanpa obat buatan pabrik, petani dapat mencegah penyakit yang menyerang tanaman.
D. Pupuk kompas dapat menunjang produktivitas pertanian lebih tinggi daripada pupuk buatan pabrik.
E. Insektisida membahayakan kesehatan manusia.

25. SPMB 2005


Pada tahun 1984, Indonesia telah berhasil mencapai swasembada beras. Hal ini berarti harga beras relatif
terjangkau oleh masyarakat. Meningkatnya ketersediaan beras, pendapatan pendudk, dan sistem nilai sosial pada
beras telah mengarahkan pola pada konsumsi pangan pokok pada beras. Oleh karena itu, diperlukan upaya-upaya
penganekaragaman pengan pokok untuk mengurangi ketergantungan pada beras sekaligus mempertahankan
swasembada beras, serta meningkatkan mutu gizi pangan penduduk. Pernyataan berikut yang sesuai dengan isi
paragraf di atas adalah :
A. keberhasilan Indonesia dalam mencapai swasembada beras dapat meningkatkan mutu pangan.
B. Peningkatan kecenderungan penduduk beralih ke pola konsumsi pangan pokok beras perlu diimbangi dengan
penganekaragaman pangan pokok.
C. Kerena swasembada beras dapat meningkatkan mutu gizi keluarga, pemerintah perlu mengusahakan pola
konsumsi pangan pokok yang lain.
D. Penganekaragaman pola konsumsi pangan pokok penduduk merupakan pekerjaan besar yang mendukung
program pemerintah.
E. Keberhasilan swasembada beras ditunjang oleh pemerintah dengan upaya perbaikan menu makanan rakyat.

SOAL BAHASA INDONESIA TAHUN 2004

UM UGM

Ilmu pengetahuan dan teknologi (iptek) modern sebagai bagian dari peradaban manusia hadir dan memasuki
percaturan politik ketika Isaac Newton membacakan buku Principia Philosophia Mathematica di hadapan para anggota
Royal Society of London pada tanggal 28 April 1686. Mulai saat itu, perkembangannya melesat sedemikian cepat
hingga mampu memicu timbulnya revolusi industri di Inggris yang kemudian menyebar ke kawasan Eropa daratan dan
Amerika. Sekarang di akhir abad XX, iptek modern telah mengalami perkembangan yang jauh lebit pesat.
Karena sedemikian berpengaruhnya perkembangan iptek modern terhadap kemajuan peradaban umat manusia,
ditambah lagi oleh adanya kenyataan akan keuniversalan sifat yang dimilikinya, iptek modern dengan mudah diterima
oleh masyarakat Indonesia. Bahkan, karena dirasa memiliki kemanfaatan bagi peningkatan martabat kemanusiaan
manusia, di samping juga peranannya sebagai sarana yang paling representatif untuk mengejar ketertinggalan bangsa
Indonesia terhadap bangsa-bangsa lain yang lebih maju, pembangunan bidang iptek pun mendapat prioritas

1. UM UGM 2004
Pernyataan-pernyataan di bawah ini dikemukakan dalam bacaan di atas , kecuali….
A. Iptek modern berkembang pesat sejak abab XX.
B. Iptek modern merupakan bagian dari peradaban manusia.
C. Iptek modern memiliki sifat universal di dalam dirinya.
D. Pembangunan bidang Iptek di Indonesia cukup mendapat prioritas.
E. Iptek memiliki kemanfaatan bagi peningkatan martabat kemanusiaan manusia.

Bidang Studi BAHASA INDONESIA 57


KUMPULAN SOAL – SOAL UJIAN MASUK PERGURUAN TINGGI NEGERI

2. UM UGM 2004
Menurut bacaan di atas, Iptek modern dengan mudah diterima oleh masyarakat Indonesia karena
A. Iptek modern berpengaruh pada kemajuan peradaban umat manusia.
B. Iptek modern memiliki manfaat bagi peningkatan martabat kemanusiaan manusia.
C. Iptek modern memiliki sifat yang universal.
D. Iptek modern merupakan sarana yang representatif untuk mengejar ketertinggalan bangsa Indonesia terhadap
bangsa-bangsa lain yang lebih maju.
E. Iptek modern bersifat fleksibel.

3. UM UGM 2004
Pikiran utama bacaan di atas adalah
A. Iptek modern merupakan bagian dari peradaban manusia.
B. Perkembangan iptek modern dimulai ketika Isaac Newton membacakan buku Principia Philosophia
Mathematica di hadapan para anggota Royal Society of London pada tanggal 28 April 1686.
C. Di akhir abad XX, iptek modern telah mengalami perkembangan yang jauh lebih pesat.
D. Iptek modern dengan mudah diterima oleh masyarakat Indonesia.
E. Iptek modern berperan sebagai sarana yang paling representatif untuk mengejar ketertinggalan bangsa
Indonesia terhadap bangsa-bangsa lain yang lebih maju.

4. UM UGM 2004
Kata – kata yang bercetak miring dalam kalimat berikut yang penulisannya mengikuti EYD adalah
A. Hakikat hidup adalah mengabdi kepada Tuhan Yang Maha Kuasa
B. Analisa para peneliti itu ditentang oleh kaum birokrat.
C. Harga obat Apotek Sari Waras lebih murah di bandingkan dengan apotek lainnya.
D. Surat kuasa tersebut telah disyahkan oelh pejabat yang berwenang.
E. Orang-orang yang disebut sebagai para normal biasanya memiliki kelebihan-kelebihan tertentu.

5. UM UGM 2004
Diantara kalimat-kalimat berikut yang benar ejaannya adalah
A. Pelantikan para guru di selenggarakan di gedung serba guna.
B. Kemarin, Gubernur Yogyakarta meresmikan taman wisata di Sleman.
C. Jarak museum itu dari pusat kota kurang lebih 1 km.
D. Menjelang operasi, pasien itu berdoa kepada Tuhan Yang Maha Esa.
E. Secara kebetulan lokasi gedung itu berada diantara perbatasan dua wilayah.

6. UM UGM 2004
Pemakaian tanda baca di bawah ini benar, kecuali
A. Karena sering tidak masuk bekerja, pegawai itu mendapat peringatan keras dari atasannya.
B. Ibu pergi ke toko membeli barang-barang kebutuhan dapur termos, gelas, dan kompor.
C. Dalam tulisan tangan atau ketikan, huruf atau kata yang akan dicetak miring diberi satu garis di bawahnya.
D. Ia menyukai musik, teater dan jenis-jenis kesenian lainnya.
E. Laki-laki itu pintar, tetapi sombong.

7. UM UGM 2004
Dari rapat para guru menghasilkan kesepakatan bahwa murid itu harus diberi peringatan keras.

Rangkaian kata tersebut akan menjadi kalimat dalam ragam baku bila kata atau rangkaian kata…. dihilangkan.
A. bahwa
B. dari
C. para
D. Dari rapat
E. harus

8. UM UGM 2004
Arti akhiran –i dalam melewati pada kalimat Masalah lingkungan hidup melewati batas-batas Negara serupa
dengan akhiran –i yang terdapat dalam kalimat
A. Setiap hari petani mengairi sawahnya.
B. Setelah upacara selesai mereka memasuki ruang sidang.
C. Seharusnya kita dapat menghargai jasa orang lain.
D. Anak-anak melempari pohon mangga yang sarat buah.
E. Bapak Gubernur menugasi bawahannya tugas tertentu yang berat.

Bidang Studi BAHASA INDONESIA 58


KUMPULAN SOAL – SOAL UJIAN MASUK PERGURUAN TINGGI NEGERI

9. UM UGM 2004
Kalimat yang tidak mengikuti kaidah yang baku adalah
A. Para mahasiswa melakukan aksi turun ke jalan untuk memprotes partai politik yang masih mendaftarkan
politisi busuknya sebagai calon legislatif.
B. Banyak aktivis LSM yang menyerukan pemboikotan terhadap produk pabrik yang diduga merusak lingkungan
itu.
C. Olahraga yang dilakukan secara baik dan teratur sangat penting untuk menjaga kesehatan.
D. Berbagai pendapat-pendapat dokter hewan dilontarkan untuk mengatasi virus flu burung.
E. Jika dilakukan secara tertib dan tidak ada penyusupan, demonstrasi tidak akan mengundang kerusuhan.

10. UM UGM 2004


Arti kata ambang dalam kalimat Pencemaran udara di beberapa tempat di Yogyakarta sudah berada di ambang
membahayakan kesehatan adalah
A. tingkat
B. batas
C. baku
D. ukuran
E. standar

11. UM UGM 2004


Umumnya usaha kecil lahir dari usaha keluarga. Usaha kecil memang merupakan bagian dari upaya peningkatan
pendapatan keluarga.
Kata yang tepat untuk menggabungkan dua kalimat di atas adalah….
A. meskipun
B. jika
C. sedangkan
D. karena
E. tetapi

12. UM UGM 2004


Dalam kalimat berikut ini terdapat kata yang mengalami perluasan makna.
A. Sejak dulu hingga sekarang profesi guru identik dengan keprijatinan dan kemelaratan.
B. Banyak tabib terkenal diundang untuk menyembuhkan penyakit aneh sang putri.
C. Dewasa ini banyak bermunculan LSM yang memperjuangkan nasib perempuan.
D. Dalam lima tahun bengkel yang mereka kelola telah menghasilkan pemasukan tidak kurang dari lima puluh
juta sebulan.
E. Peran uang tidak pernah absen dalam setiap pementasan Srimulat adalah jongos.

13. UM UGM 2004


Kelompok Usaha Mandiri Al-Hidayah hanya menghasilkan kerajinan tangan peci rajut. Setelah beberapa lama
berlangsung pemasarannya mulai tersendat. Sebagai solusinya diperlukan…. produksi.
A. kreativitas
B. diversivikasi
C. inovasi
D. peningkatan
E. penambahan

14. UM UGM 2004


Tidak dapat disangkal … ada hubungan erat antara kondisi ekonomi dengan kekuatan pertahanan keamanan suatu
bangsa…kondisi ekonomi yang baik dapat memberikan dukungan pada terjaminnya pertanahan dan keamanan
negara. … pertahanan keamanan yang memadai dapat memberikan suasana yang sangat kondusif bagi
perkembangan ekonomi. …. Hubungan antara ekonomi dan pertahanan keamanan dapat diibaratkan dua sisi
sekeping mata uang.
Kata-kata yang tepat untuk melengkapi bacaan di atas adalah
A. bahwa, di sini, di lain pihak, dengan begitu
B. jika, di satu pihak, sementara di pihak yang lain, dalam hal ini
C. bahwa, di satu sisi, di sisi lain, sehingga
D. bahwa, di satu sisi, sementara di sisi yang lain, dengan demikian
E. kalau, dalam hal ini, di samping itu, sehingga

Bidang Studi BAHASA INDONESIA 59


KUMPULAN SOAL – SOAL UJIAN MASUK PERGURUAN TINGGI NEGERI

15. UM UGM 2004


Kalimat yang berisi opini adalah
A. Pergantian pimpinan universitas terjadi setiap 4 tahun sekali.
B. Menurut peraturan, rektor menduduki jabatan selama 4 tahun.
C. Rektor baru itu sangat disukai mahasiswa karena sifat keterbukaannya.
D. Rektor itu telah menduduki jabatan selama 4 tahun.
E. Jabatan sebagai rektor universitas terkenal telah dijalaninya dengan senang hati.

16. UM UGM 2004


(1) Demikian halnya kalau kita sedang bekerja.
(2) Karbohidrat dapat diibaratkan sebagai kayu bakar dalam proses pembakaran.
(3) Karbohidrat merupakan makanan yang dapat menjadi sumber tenaga.
(4) Semakin keras kita bekerja, semakin banyak pula sumber energi yang diperlukan
(5) Akan tetapi, jika hanya mengonsumsi karbohidrat tanpa protein, tubuh kita akan menjadi lemah.
(6) Semakin besar api yang kita inginkan, semakin banyak pula kayu bakar yang kita perlukan.
Urutan yang tepat agar kalimat – kalimat di atas menjadi paragraf yang baik adalah
A. (3) – (2) – (1) – (4) – (5) – ( 6)
B. (3) – (2) – (6) – (1) – (4) – ( 5)
C. (2) – (6) – (1) – (4) – (5) – ( 3)
D. (2) – (3) – (6) – (1) – (4) – ( 5)
E. (3) – (5) – (2) – (6) – (1) – ( 4)
17. UM UGM 2004
Wawancara berlangsung hangat, selain anak yang di wawancarai begitu komunikatif, juga kedua orang tuanya
bertindak sangat…. sangat antusias membantu proses wawancara yang berlansung cukup lama.
Kata yang tepat untuk melengkapi kalimat di atas adalah
A. aktif
B. reaktif
C. proaktif
D. kondusif
E. permisif

18. UM UGM 2004


Dari buku-buku dan banyak tulisan/ karangan di berbagai media massa yang beredar, dapat dilihat betapa banyak
kaum wanita yang merasa dikecewakan oleh pelayanan kesehatan modern, khusunya di bidang penyakit wanita.
Kalimat di atas merupakan perluasan dari kalimat inti
A. Media massa beredar
B. Wanita dilihat
C. Wanita dikecewakan
D. Peleyanan mengecewakan
E. Penyakit wanita

19. UM UGM 2004


Kalimat yang polanya sama dengan kalimat Akhir-akhir ini flu burung menyerang unggas di beberapa wilayah di
Jawa Tengah, dan DIY adalah…
A. Belakangan ini minat orang mengonsumsi ayam goreng menurun karena takut tertular flu burung.
B. Tiga tahun lalu chikungunya hanya meresahkan warga Kulon Progo dan Purworejo.
C. Dua tahun lalu chikungunya menyerang penduduk di daerah purworejo.
D. Bulan lalu unggas yang terinfeksi flu burung dimusnahkan dengan cara dikubur dan dibakar.
E. Bulan ini calon mahasiswa UGM mengikuti ujian tulis materi kognitif dan nonkognitif.
20. UM UGM 2004
1. Menurut Koordinator Umum Kegiatan, program ini digulirkan untuk mengurangi kemungkinan terjadinya
manipulasi dalam pelaksanaan Pemilu 2004 mendatang
2. Setelah mengumumkan akan menggelar KKN pemantau pemilu, Fisipol UGM juga membuat program baru
yang terkait dengan pemilu.
3. Hujan demokrasi berupa Pemilu 2004 benar- benar dimanfaatkan masyakarat secara maksimal.
4. Program baru ini adalah program pendidikan politik untuk masyakarat miskin kota yang digelar oleh Lembaga
Peneliti dan Publikasi Fisipol.
Urutan yang baik agar menjadi kalimat – kalimat di atas menjadi paragraf yang padu adalah…
A. 1 – 4 – 2 – 3
B. 3 – 2 – 4 – 1
C. 2 – 3 – 4 – 1
D. 3 – 1 – 2 – 4
E. 4 – 2 – 1 – 3
Bidang Studi BAHASA INDONESIA 60
KUMPULAN SOAL – SOAL UJIAN MASUK PERGURUAN TINGGI NEGERI

SPMB

Data statistik Organisasi Kesehatan Dunia menyebutkan bahwa di dunia terdapat lima ratus jenis penyakit, dan
hanya tiga puluh persen yang dapat disembuhkan dengan intervensi penggunaan obat. Yang tujuk puluh persen
selebihnya harus disembuhkan dengan cara yang sangat berlainan, baik dengan teknologi medis mutakhir atau bahkan
dengan cara pengobatan tradisional. Salah satu cara pengobatan tradisional adalah melalui terapi panas telapak tangan.
Terapi panas telapak tangan adalah salah satu dari sekian banyak pergobatan tradisional yang dilakukan dengan
cara menjerang penderita dengan panas aliran tubuh melalui telapak tangan. Telapak tangan tersebut tidak diletakkan
langsung pada tubuh si sakit, tetapi dengan perantaraan handuk basah yang diselubungkan pada tumbuh penderita.
Handuk tersebut akan mengepul mengeluarkan asap. laksana handuk basah yang terkena setrika panas. Cara yang
demikian ini, untuk satu kasus penyakit, dilakukan secara berulang-ulang.

Dari segi nalar, naral ilmu atau nalar logika, cara pengobatan ini memang sulit dimengerti dan tidak mudah dicerna
oleh akal orang awam. Namun, bagi yang berperhatian, apalagi yang sudah mendalami konsep pengobatan panas
telapak tanga, tentu saja cara pengobatan itu memang beralasan dengan dukungan pengetahuan yang metodis. Panas
tubuh orang normal senantiasa berada pada kisaran 37 derajat celcius. Terpeliharanya nilai normal suhu tubuh tersebut
semata-mata oleh adanya mekanisme sistem keharmonisan tubuh. Dalam hal tertentu, antara lain oleh rasa takut, resah,
cemas, kaget, dan atau sakit deman, suhu tubuh manusia dapat berubah.

1. SPMB 2004
Gagasan pokok paragraf II dikembangkan dengan menggunakan pertanyaan :
A. Apakah tujuan pengobatan terapi panas telapak tangan?
B. Bagaimanakah cara pengobatan terapi panas telapak tangan?
C. Berapakah kali dilakukan pengobatan dengan terapi panas telapak tangan?
D. Apakah dampak pengobatan dengan terapi panas telapak tangan?
E. Mengapa panas telapak tangan dapat dihunakan untuk pengobatan penyakit?

2. SPMB 2004
Pernyataan berikut ini yang sesuai dengan data statistik Organisasi Kesehatan Dunia adalah ....
A. Banyak orang yang lebih percaya mengobatkan penyakitnya ke dokter daripada melalui pengobatan
tradisional.
B. Penyakit yang dapat disembuhkan melalui cara minum atau menggunakan obat, jumlahnya sekitar 150 jenis.
C. Hampir semua penyakit dapat disembuhkan melalui teknologi medis mutakhir.
D. Pengobatan tradisional memiliki peranan yang sama dengan penyembuhan secara teknologi medis mutakhir.
E. Sebagaian besar penyakit yang diderita oleh seseorang disembuhkan dengan intervensi obat yang berbeda-
beda.

3. SPMB 2004
Kata intervensi penggunaan obat (kalimat 1, paragraf 1) teks di atas memiliki makna ....
A. akibat penggunaan obat
B. pengaruh penggunaan obat
C. dampak penggunaan obat
D. manfaat penggunaan obat
E. reaksi penggunaan obat

4. SPMB 2004
Pernyataan berikut ini yang sesuai dengan isi paragraf III teks di atas adalah
A. Terapi panas telapak tangan termasuk pengobatan tradisional yang tidak masuk akal.
B. Terapi panas tangan termasuk cara pengobatan yang tidak dapat dipelajari dari segi keilmuan.
C. Terapi panas telapak tangan merupakan cara pengobatan yang dapat dipertanggungjawabkan secara ilmuwan.
D. Terapi telapak tangan dapat dimengerti oleh orang awam dengan dukungan pengetahuan yang metodis.
E. Terapi panas telapak tangan banyak diminati oleh orang awam karena lebih sederhana dan tidak menakutkan

5. SPMB 2004
Judul yang tepat untuk teks di atas adalah ....
A. Terapi Panas Telapak Tangan Dalam Penyembuhan Penyakit.
B. Manfaat Terapi Panas Telapak Tangan.
C. Penyembuhan Penyakit dengan Terapi Panas Telapak Tangan.
D. Panas Telapak Tangan dan Terapinya dalam Pengobatan.
E. Pengobatan dengan Terapi Panas Telapak Tangan
Bidang Studi BAHASA INDONESIA 61
KUMPULAN SOAL – SOAL UJIAN MASUK PERGURUAN TINGGI NEGERI

6. SPMB 2004
.... yang proporsional terhadap virus flu burung sangat penting ... untuk kerugian yang lebih besar.
Kata yang tepat untuk mengisi rumpang di atas adalah ....
A. pembasmian  mengatasi
B. penanganan  mencegah
C. penanggulangan  menghentikan
D. pemberantasan  mengecilkan
E. pengurusan - menhambat

7. SPMB 2004
Pemakaian tanda baca yang benar terdapat pada kalimat ....
A. Sahlan, mengucapkan terima kasih atas bantuan Putut.
B. Semua peserta, yang tidak membawa kartu peserta, harus melaporkan diri ke panitia.
C. F. Ratulangi tinggal di Jalan Mawar 5, Pati, Jawa Tengah.
D. Ia harus pulang sekarang, karena sakit.
E. Sahabat saya Amin tinggal di Manokwari.

8. SPMB 2004
Penggunaan tanda baca yang mengikutu EYD terdapat dalam kalimat ....
A. Penggunaan obat tradisional, termasuk jamu yang makin marak dewasa ini adalah kenyataan yang patut
disyukuri.
B. Kendati pemakaian jamu cukup marak dan industri jamu juga berkembang, sungguh mengherankan kalau
ternyata produksi tanaman obat beberapa tahubn belakangan stagnan.
C. Dalam catatan H. Sampurno, M.B.A. Kepala Badan POM Indonesia memiliki 30.000 jenis tumbuhan.
D. Penurunan angka produksi menjadi ironi, kalau kita bandingkan dengan pasar yang sesungguhnya
membentang luas.
E. Yang cukup menggembirakan, adalah kenyataan bahwa beberapa rumah sakit sudah memasukkan obat
tradisional dalam terapi kepada pasien.

9. SPMB 2004
“Dengan adanya keikutsertaan virus influenza ungas pada wabah penyakit yang mematikan ribuan ayam petelur di
Jawa Tengah dan Jawa Timur, telah terbukti virus tersebut tidak hanya .... tetapi sudah .... melalui berbagai
metode diagnostik”.
Pasangan kata yang tepat untuk mengisi bagian yang kosong pada kutipan di atas adalah ....
A. diisolasi  dirujuk
B. ditemukan  diidentifikasi
C. dirujuk  diidentifikasi
D. ditemukan  diisolasi
E. diisolasi  diidentifikasi

10. SPMB 2004


Deretan gunung dalam kelompok Pegunungan Jaya Wijaya di Wamena dengan hutan menghijau disekitarnya
memang memunculkan pemandangan amat cantik.
Kalimat di atas merupakan perluasan dari kalimat inti !
A. Deretan gunung memunculkan pemandangan amat cantik.
B. Deretan gunung menghijau amat cantik.
C. Hutan menghijau memunculkan pemandangan.
D. Deretan gunung dalam kelompok Jayawijaya amat cantik.
E. Pegunungan Jayawijaya menghijau memunculkan pemandangan.

11. SPMB 2004


Pada kalimat Para pelaku sektor informal yang menempati trotoar-trotoar jalan dan tempat terlarang lain tetap
tidak berkurang, yang tetap tidak berkurang adalah ....
A. para pelaku
B. para pelaku sektor informal
C. para pelaku sektor infromal yang menempati trotoar-trotoar.
D. para pelaku sektor infromal yang menempati trotoar-trotoar jalan
E. para pelaku sektor infromal yang menempati trotoar-trotoar dan tempat terlarang lain.

Bidang Studi BAHASA INDONESIA 62


KUMPULAN SOAL – SOAL UJIAN MASUK PERGURUAN TINGGI NEGERI

12. SPMB 2004


Sebagaimana telah ditetapkan dalam GBHN bahwa pembangunan pendidikan dititikberatkan pada peningkatan
setiap jenjang dan jenis pendidikan.
Kalimat di atas dapat diperbaiki dengan menghilangkan kata ....
A. sebagaimana
B. telah
C. dalam
D. bahwa
E. pada

13. SPMB 2004


1) Peristiwa itu terjadi sejak aturan baru yang ketat itu diumumkan.
2) Dalam aturan itu setiap siswa yang terlambat dikenakan wajib lapor.
Penggabungan kedua kalimat tersebut yang benar adalah ....
A. Peristiwa itu terjadi sejak aturan baru yang ketat itu diumumkan, dimana setiap siswa yang terlambat
dikenakan wajib lapor.
B. Setiap siswa yang terlambat dikenakan wajib lapor sejak aturan baru yang ketat itu diumumkan.
C. Ketika aturan baru yang ketat itu diumumkan, setiap siswa yang terlambat dikenakan wajib lapor.
D. Peristiwa bahwa setiap siswa yang terlambat dikenakan wajib lapor dilakukan sejak aturan baru yang ketat itu
diumumkan.
E. Aturan baru yang ketat itu diumumkan, karena itu setiap siswa yang terlambat dikenakan wajib lapor.

14. SPMB 2004


Pendidikan merupakan proses pendewasaan bangsa yang akan menjadi modal utama pembangunan. karena itu,
pemerintah Indonesia .... anggaran belanja yang sangat besar dibidang pendidikan.
Kata yang tepat untuk mengisi bagian kosong pada kalimat di atas adalah ....
A. memanipulasi
B. mengakomodasi
C. menginvestasikan
D. mengasuransikan
E. mengalkulasi

15. SPMB 2004


Para cendekiawan sering berpikiran mendahului apa yang akan terjadi.
Kata yang tepat untuk melambangkan tindakan cendekiawan tersebut adalah
A. persuasi
B. prediksi
C. prevensi
D. provokasi
E. reaksi

16. SPMB 2004


Stasiun-stasiun televisi swasta yang menampilkan acara infotainment memang telah menjadikannya sebuah paket
hiburan yang selalu sarat dengan peristiwa-peristiwa sensasional.
Makna kata sensasional pada kalimat di atas adalah ....
A. menarik hati
B. bersifat menggemparkan
C. bersifat menyedihkan
D. membuat bahagia
E. membuat perasaan terharu

17. SPMB 2004


Segala hal yang dilakukan selama ini sebenarnya tidak konsisten.
Kata konsisten dalam kalimat tersebut berarti ....
A. taat asas
B. ada gunanya
C. berbahaya
D. penting
E. tepat

Bidang Studi BAHASA INDONESIA 63


KUMPULAN SOAL – SOAL UJIAN MASUK PERGURUAN TINGGI NEGERI

18. SPMB 2004


Dengan mengudap makanan ringan, perut akan merasa hangat. Apalagi kalau camilan itu dimakan pada waktu
cuaca mendung. Ibu rumah tangga bisa menyediakan camilan yang menyehatkan setiap hari. Untuk ibu yang
hobinya masak, hal ini bukan masalah akan tetapi apa jadinya kalau ibu itu seorang wanita karier yang sibuk
dengan urusan di luar rumah? Kalau sudah begini siapkan saja biskuit siap saji yang menyehatkan.
Pokok yang dibicarakan dalam teks di atas adalah
A. makanan camilan yang harus tersedia di setiap rumah tangga
B. fungsi camilan bagi setiap orang
C. biskuit sebagai satu-satunya camilan yang menyehatkan
D. penyediaan camilan dalam keluarga oleh seorang ibu.
E. penyediaan camilan yang menjadi alteratif para ibu.

19. SPMB 2004


Andalan provinsi ini adalah kemolekan alamnya. Karena itu, dunia pariwisata harus digalakkan. Lebih dari itu,
kegiatan pariwisata tidak boleh meninggalkan prinsip manajerial yang komprehensif. Dari sinilah kehidupan
pariwisata dikendalikan. Penerimaan daerah ini ternyata juga ditulangpungungi oleh migas, pendapatan asli,
pajak pertambahan nilai dan yang paling menggiurkan, penarikan retribusi parkir.
Berdasarkan teks di atas, urutan primadona pendapatan daerah adalah ....
A. pariwisata, migas, pajak, parkir
B. migas, pariwisata, pajak, parkir
C. parkir, migas, pariwisata, pajak
D. parkir, pajak, migas, pariwisata
E. pajak, parkir, migas, pariwisata

20. SPMB 2004


Dua pekan lalu, sekitar 200an perempuan berkumpul di sebuah hotel terkenal. Mereka mengenakan “busana
nasional” yang umumnya berupa kain sarung dari berbagai daerah dengan kebaya beragam bentuk. Begitu
banyak perempuan berkumpul untuk memeriahkan hari jadi sebuah perkumpulan yang mendedikasikan kegiatan
mereka untuk mempromosikan busana nusantara.
Inti paragraf di atas adalah ....
A. mereka berkumpul di hotel untuk peragaan busana nasional.
B. mereka berkumpul untuk mengadakan promosi pakaian daerah
C. mereka berkumpul dalam rangka ulang tahun organisasi
D. mereka berkumpul untuk menghadiri promosi mode busana nasional.
E. mereka berkumpul untuk saling bersilahturohmi.

21. SPMB 2004


Di antara kalimat berikut yang paling tepat untuk memorandum berupa pemberian informasi adalah .
A. Tolong berikan nama pegawai yang akan mengikuti penataran.
B. Mohon dipelajari dan ditindaklanjuti
C. Harap datang pada hari ini, pukul 10,00 WIB
D. Mohon diperhatikan bahwa pada hari Selasa tanggal 26 April akan diadakan pertemuan direksi.
E. Agar pertemuan berjalan lancar, Saudara datang tepat waktu.

22. SPMB 2004


Kata serapan yang berasal dari bahasa asing digunakan secara tepat dalam kalimat.
A. Para wakil rakyat yang duduk di DPR harus dapat menyampaikan inspirasi masyarakat kepada pemerintah.
B. Tampaknya isi pasal-pasal dalam perundangan itu tidak efektif untuk menanggulangi kejahatan.
C. Kami tidak suka membeli barang-barang yang kuantitasnya kurang baik.
D. Kita dapat menjangkau desa terpencil itu dengan menggunakan sarana komunikasi sepeda motor.
E. Orang itu dapat memanfaatkan waktu secara selektif sehingga sukses dalam usahanya.
23. SPMB 2004
Pada lembaga itu lansia menyibukkan diri dalam berbagai kegiatan. Mereka mengikuti kegiatan percakapan
bahasa Inggris, bekerja dengan komputer, melukis, atau bermain musik. Mereka juga dapat memilih cabang
olahraga yang mereka senangi dan yang sesuai dengan usia mereka. Bahkan, mereka dapat membantu dokter di
pusat pelayanan kesehatan yang tersedia di lembaga itu.
Kalimat yang paling tepat untuk menutup paragraf di atas adalah ....
A. Dengan demikian, mereka tidak lekas menjadi pikun.
B. Itulah kegiatan mereka sehari-hari.
C. Kesibukkan itu membuat mereka senang.
D. Bagaimana mereka akan kesepian dengan kesibukan itu?
E. Kegiatan yang demikian itulah yang menghibur mereka di lembaha itu.

Bidang Studi BAHASA INDONESIA 64


KUMPULAN SOAL – SOAL UJIAN MASUK PERGURUAN TINGGI NEGERI

24. SPMB 2004


Urutkan lima kalimat berikut sehingga menjadi paragraf yang utuh!
1) Bantuan itu dimaksudkan untuk pengembangan di bidang lingkungan.
2) Dalam kunjung ini Trimmer menyerahkan bantuan 13 juta rupiah.
3) Baru-baru ini Direktur Utama Papan Sejahtera, Yunia Trimmer berkunjung ke Kantor Pusat pengembangan
BIPA.
4) Kedatangannya disambut oleh Direktur BIPA dan beberapa stafnya.
5) Dalam mengembangkan programnya. Pusat pengembangan BIPA memperoleh bantuan dari berbagai pihak.
Rangkaian yang tepat dari lima kalimat di atas adalah ....
A. 3  4  2  1  5
B. 3  2  1  4  5
C. 5  3  4  2  1
D. 5  1  3  4  2
E. 3  4  2  5  1

25. SPMB 2004


Bedah jantung telah berkembang sangat pesat di akhir abad XX. Banyak peralatan operasi yang canggih dan juga
berbagai teknik operasi ditemukan. Pada mulannya operasi bedah jantung dianggap sebagai suatu jenis operasi
yang sangat besar dan berisiko tinggi. Tindakan operasi merupakan suatu bentuk pengobatan penyakit jantung
yang berakhir apabila tidak ada jalan lain lagi.
Kalimat berikut ini yang tepat untuk mengakhiri paragraf di atas adalah ....
A. Oleh karena itu, operasi tidak dilakukan pada pasien dengan keadaan yang kritis.
B. Oleh karena itu, operasi sering dilakukan pada pasien dengan keadaan yang kritis.
C. Oleh karena itu, operasi jarang dilakukan pada pasien dengan keadaan yang kritis.
D. Oleh karena, itu operasi menjadi satu-satunya harapan bagi pasien dengan keadaan kritis.
E. Oleh karena itu, operasi pada pasien dengan keadaan yang kritis memang dilakukan.

Bidang Studi BAHASA INDONESIA 65


KUMPULAN SOAL – SOAL UJIAN MASUK PERGURUAN TINGGI NEGERI

SOAL BAHASA INDONESIA TAHUN 2003

UM UGM

Harga manusia di muka obat melalui undang - undang praktis tidak ada. Apa benar demikian? Pembunuh
terselubung. Itulah julukan mengerikan yang paling tepat disandang oleh para pemalsu obat.Namun, sayang, di negeri
ini para “ pembunuh” itu masih bisa melenggang ringan karena hukuman bagi para pemalsu obat masih tergolong
ringan.Akibatnya, di pasaran masih terus beredar obat dan jamu, baik illegal maupun palsu, serta makanan dan
minuman yang mengandung zat adiktif yang terlarang untuk pangan, seperti boraks, formalin, dan pewarna tekstil.
Lebih mengejutkan lagi, vonis untuk para pecundang belumlah spadan dengan tindak kejahatan yang telah
mereka lakukan. Lihat saja, misalnya, pidana bagi para penyimpan dan penjual obat impor yang tidak terdaftar dan
palsu hanya 6 bualn penjara serta denda Rp. 300.000,00 sudsider satu bulan kurungan. Hukuman bagi penjual obat
tradisional mengandung bahan kimia hanyalah denda Rp. 500.000,00 sudsider dua bulan kurungan. Penjual makanan
kadaluwarsa didenda Rp. 100.000,00 sudsider tujuh hari kurungan. Sementara itu, penjual makanan yang dicampur
bahan kimia yang dilarang untuk makanan hanya dipidana penjara dengan masa percobaan empat bulan.

1. UM UGM 2003
Topik yang paling tepat untuk bacaan di atas adalah
A. martabat manusia tida ada nilainya
B. hukuman yang sangat ringan bagi “pembunuh”
C. larangan memproduksi makanan yang mengandung zat adiktif
D. pemasaran obat,jamu, dan makanan yang emngandung zat adiktif.
E. Pidana bagi para penjual dan pengimpor obat illegal.

2. UM UGM 2003
Hal-hal berikut terdapat dalam bacaan di atas kecuali
A. vonis terhadap para “ pembunuh” hendaknya sesuai dengan kesalahannya.
B. di toko banyak penjual jamu dan makanan impor
C. para pemalsu obat disebut para “pembunuh”
D. hukuman para “pembunuh” itu rata-rata kurang dari satu tahun.
E. hukuman yang berupa uang bisa digantikan dengan yang lain

3. UM UGM 2003
Karangan di atas bersifat
A. naratif
B. argumentatif
C. deskriptif
D. ekspositoris
E. kritik

4. UM UGM 2003
Ide utama bacaan di atas adalah
A. upaya peningkatan produksi obat dan jamu
B. upaya pemerataan distribusi obat dan jamu
C. upaya dan saran pemberdayaan hokum
D. upaya pencegahan produksi obat palsu
E. upaya pelanggaran masuknya obat impor

5. UM UGM 2003
Judul yang paling tepat untuk bacaan di atas adalah….
A. Ketergantungan Manusia pada Obat.
B. Semua Obat dan Jamu Harus Terdaftar di Departemen Kesehatan.
C. Kerja Sama antara Produsen dan Distributor Obat.
D. Pidana bagi Pengimpor Obat.
E. Pidana Ringan bagi Pesalsu Obat.

Bidang Studi BAHASA INDONESIA 66


KUMPULAN SOAL – SOAL UJIAN MASUK PERGURUAN TINGGI NEGERI

6. UM UGM 2003
Kalimat-kalimat berikut ini tidak dapat digunakan dalam karya ilmiah karena penulisan tanda bacanya tidak
mengikuti EYD kecuali
A. Penggunaan Obat tradisional termasuk jamu yang makin marak dewasa ini adalah kenyataan yang patut
disyukuri.
B. Kendati pemakaian jamu cukup marak dan industry jamu juga berkembang, sungguh mengherankan kalau
ternyata produksi tanaman obat beberapa tahun belakangan stagnan.
C. Dalam catatan Kepala Badan POM Indonesia memiliki 30.000 jenis tumbuhan.
D. Penurunan angka produksi menjadi ironi, kalau kita bandingkan dengan pasar yang sesungguhnya
membentang luas.
E. Yang cukup menggembirakan, adalah kenyataan bahwa beberapa rumah sakit sudah memasukkan obat
tradisional dalam terapi kepada pasien.

7. UM UGM 2003
Penulisan kata asing yang sesuai dengan ejaan resmi terdapat dalam kalimat
A. Jika kebocoran akibat virus diabaikan, penderita akan banyak kehilangan cairan, akhirnya akan mengalami
s h o c k.
B. Genangan air merupakan tempat berkembang biaknya nyamuk “ Aedes Aegepty “
C. Asuransi ini memiliki program professional mem-back up dokter yang mendapat gugatan malapraktik dari
pasien.
D. Lahar dingin ini “nggrojog” dari atas seperti ombak besar.
E. Ronaldo mencetak hattrick pada menit ke-89 sehingga pertandingan berakhir draw.

8. UM UGM 2003
Desentralisasi diharapkan bisa … daerah untuk…pendidikan penduduk dan menciptakan lapangan kerja di
daerah sehingga orang tidka perlu bermigrasi ke Jawa.
Pasangan kata yang sesuai untuk mengisi tempat kosong di atas adalah
A. ditingkatkan-memacu
B. meningkatkan-memacu
C. memacu-meningkatkan
D. dipacu-meningkatkan
E. dipacu-ditingkatkan

9. UM UGM 2003
…. sektor pengangkatan dan komunikasi bagi Kabupaten Wonogiri sebesar Rp229,8 miliar atau 10,2% dari total
kegiatan ekonomi.
Kata yang tepat untuk melengkapi kalimat di atas adalah
A. distribusi
B. kontribusi
C. kuantitas
D. komoditas
E. aktivitas

10. UM UGM 2003


Sikap berbahasa yang positif dan kebiasaan berbahasa Indonesia dengan baik dan benar perlu….dan… di
kalangan masyarakat.
Pasangan kata yang sesuai untuk mengisi tempat kosong di atas adalah
A. ditingkatkan-dikembangkan
B. meningkat-berkembang
C. meningkat-dikembangkan
D. dikembangkan-bertingkat
E. dikembangkan-meningkat

11. UM UGM 2003


Bentuk kalimat yang baku adalah
A. Budaya syawalan harus kita lestarikan dan kembangkan
B. Budaya syawalan kita harus lestarikan dan kembangkan
C. Budaya syawalan kita harus melestarikan dan mengembangkan
D. Budaya syawalan kita harus melestarikan dan kembangkannya
E. Budaya syawalan harus kita lestarikan dan dikembangkan

Bidang Studi BAHASA INDONESIA 67


KUMPULAN SOAL – SOAL UJIAN MASUK PERGURUAN TINGGI NEGERI

12. UM UGM 2003


Pola kalimat Sekolah kami selalu mendapat predikat sekolah bersih terdapat juga dalam kalimat
A. Salah satu sekolah negeri di kota kami membentengi siswa-siswanya dari pengaruh negatif televise dengan
pendidikan moral.
B. Dalam majalah ini, dimuat artkel-artikel yang bermutu ilmiah.
C. Dengan tugas sekompleks itu, idealnya guru BP dapat lebih dekat dengan siswa.
D. Dalam dasawarsa ini, dia ingin terus berprestasi dalam olahraga beladiri.
E. Dengan mengkomunikasi suatu lembaga kepada masyarakat, dapat ditempuh bermacam-macam cara.

13. UM UGM 2003


Urutkanlah enam kalimat berikut ini sehingga menjadi paragraf yang baik!
1. Dalam hal ini, pemandu harus dilengkapi dengan kemampuan member penjelasan secara rinci kepada
wisatawan.
2. Dalam menjual pariwisata, terutama ketika masuk sentra-sentra kerajinan, peran pemamndu sangat besar.
3. Mereka perlu penjelasan yang terperinci dan lugas tentang apa yang dilihatnya.
4. Dengan demikian, mereka akan merasa mendapatkan suatu pengalaman yang luar biasa, seakan-akan ikut
mengalami sendiri.
5. Wisatawan tidak hanya dibiarkan melihat demonstrasi proses produksi kerajiann tanpa penjelasan
6. Misalnya, ketika menyaksikan demonstrasi proses pemintalan sutra, mereka harus diberi penjelasan tentang
proses ini mulai dari pengadaan bahan hingga maksud pemintalan.
Urutan yang baik adalah
A. 3 – 4 – 5 – 1 – 2 – 6
B. 2 – 1 – 5 – 3 – 6 – 4
C. 1 – 2 – 4 – 5 – 6 – 3
D. 4 – 5 – 3 – 2 – 1 – 6
E. 2 – 1 - 4 – 3 – 5 – 6

14. UM UGM 2003


Kalimat yang mengandung cara melakukan perbuatan adalah….
A. Pemerintah mengumumkan harga BBM baru itu tadi malam.
B. Perundingan itu akan berlangsung di Jakarta.
C. Dengan tegas, mereka menolak usul itu.
D. Mereka bersedia mengalah demi keutuhan organisasi.
E. Tanpa dukungan anggota, program ini tidak akan berhasil.

15. UM UGM 2003


….Untuk tarif pelanggan rumah tangga, harga yang harus di bayar oleh pelanggan Indonesia adalah sekitar 3,72
hingga 9,97 sen dolar per kilowatt jamnya. Di Vietnam harganya jauh di bawah angka itu, yaitu antara 1,62
hingga 5,48 sen dollar. Sementara itu, harag listrik di Thailand hamper sama di Vietnam. Yang paling murah
adalah harga listrik di Laos, yaitu antara 0,13 hingga 1,29 sen dollar.Tingginya harag listrik di Indonesia ini
perlu dikaji. Mengapa Indonesia tidak dapat menjual listrik dengan harga murah seperti negara-negara ASEAN
lainnya, padahal jumlah pelanggan di Indonesia jauh lebih banyak?

Kalimat yang tepat digunakan sebagai kalimat pertama paragraf di atas ialah
A. Belum lama ini Bank Dunia melaporkan harga listrik di beberapa Negara ASEAN termasuk Indonesia,
Vietnam, Thailand, dan Laos
B. Menurut laporan Bank Dunia, harga jual listrik di Indonesia ternyata lebih mahal daripada tarif listrik di
beberapa Negara ASEAN seperti Thailand, Laos, dan Vietnam.
C. Menurut laporan Bank Dunia disebutkan harga listrik di beberapa Negara ASEAN seperti Indonesia, Vietnam
Thailand, dan Laos.
D. Menurut laporan Bank Dunia, Laos merupakan negara anggota ASEAN yang harag listriknya paling murah
dibandingkan dengan Indonesia, Vietnam, dan Thailand.
E. Dalam laporan Bank Dunia mengenai listrik di beberapa Negara ASEAN disebutkan bahwa harga listrik di
Vietnam dan Thailand hamper sama.

16. UM UGM 2003


Arti kata bodong pada kalimat Tidak sedikit institusi menawarkan gelar bodong adalah
A. gelar kesarjanaan
B. gelar kebangsawanan
C. gelar kehormatan
D. gelar palsu
E. gelar sementara

Bidang Studi BAHASA INDONESIA 68


KUMPULAN SOAL – SOAL UJIAN MASUK PERGURUAN TINGGI NEGERI

17. UM UGM 2003


Makna prediksi dalam kalimat Telah diprediksikan bahwa bukan hanya universitas yang besar yang membangun
kehadirannya di daerah berarti
A. dihitung
B. diramal
C. dikira
D. diperkirakan
E. perkiraan

18. UM UGM 2003


Kata eksekusi dalam Eksekusi pembongkaran rumah di bantaran sungi itu mendapat perlawanan dari penduduk
setempat mempunyai makna
A. penghapusan hukuman
B. mengurangkan hukuman
C. penguatan hukuman
D. pembatalan keputusan hukuman
E. pelaksanaan keputusan hukuman

19. UM UGM 2003


Kalimat yang mengawali paragraf pembukaan yang tidak dapat dipakai untuk surat resmi adalah
A. Menunjuk Surat Saudara tanggal….. Nomor….
B. Dengan ini, kami membalas surat Saudara tanggal… Nomor….
C. Berkenaan dengan surat Saudara tanggal… Nomor…
D. Surat Saudara tnaggal… Nomor… sudah kami terima dengan selamat
E. Dengan ini diberitahukan bahwa…

20. UM UGM 2003


Hal-hal berikut menyangkut prinsip-prinsip berbahasa dalam forum diskusi kecuali
A. kata-kata diucapkan dengan jelas.
B. menggunakan kalimat efektif
C. menggunakan bahasa yang baik dan benar.
D. member tekanan pada bagian kalimat yang di anggap penting.
E. mengutamakan kata – kata singkatan atau akronim.

21. UM UGM 2003


Sepanjang tahun 2002, tiga belas pelaku korupsi telah dibebaskan oleh majelis haki,. Sebannyak empat belas
terdakwa korupsi yang sudah di eksekusi pun tidak perlu menjalani hukuman karena hakim tak memerintahkan
untuks egera ditahan. Saat ini, timbul kesan bahwa hakim hanya berani menghukum terdakwa apabila terdakwa
tidak hadir di pengadilan.
Pikiran utama pernyataan di atas adalah
A. para koruptor selamat dari hukuman
B. tahun 2002 adalah tahun kebebasan koruptor
C. para hakim tidak punya keberanian menjatuhkan hukuman
D. vonis yang dijatuhkan harus melihat hadir atau tidaknya terdakwa.
E. setiap pesakitan yang dijatuhi hukuman tidak harus menjalani hukuman.

22. UM UGM 2003


Dua hari yang lalu, telah di gagalkan penyelundupan satwa liar ke Jepang. Belum lagi usai urusan itu, menyusul
dua orang Kuwait berupaya menyelundupkan 246 satwa liar juga dari ABndara Soekarno-Hatta pada pukul
23.30. Mereka di tangkap dan diperiksa oleh yang berwajib.
Kedua kalimat di atas dapat diungkapkan kembali dengan kalimat berikut
A. Pemerintah harus membuat undang-undang tentang penyelundupan
B. Tidak ada manfaatnya melepas penyelundup.
C. Nasib malang bisa menimpa orang yang tidak mau tahu aturan tertentu.
D. Para penyelundup tidak harus ditahan karena ketidaktahuannya
E. Penyelundup disiapkan menjadi orang yang tahu peraturan setelah diperiksa.

Bidang Studi BAHASA INDONESIA 69


KUMPULAN SOAL – SOAL UJIAN MASUK PERGURUAN TINGGI NEGERI

23. UM UGM 2003


Seandainya Von Koenigswald tahun 1934 tidak menginjakkan kakinya di bumi Sangiran, situs manusia purba ini
mungkin tidak akan setenar sekarang. Mengapa? Karena sejak kunjungan itu, nama Sangiran muncul dalam
ranah ilmu pengetahuan sebagai situs penemuan alat batu.
Pernyataan berikut ini sesuai dengan maksud yang terkandung dalam kutipan di atas kecuali
A. Sejak kedatangan Von Koenigswald ke Sangiran, daerah Sangiran menjadi terkenal.
B. Sangiran terkenal karena situs manusia purbanya.
C. Sejak tahun 1934, Sangiran di kenal sebagai daerah situs manusia purba.
D. Von Koenigswald mengunjungi Sangiran tahun 1934 ketika daerah itu belum terkenal.
E. Sebelum Von Koenigswald dating ke Sangiran, daerah Sangiran sudah dikenal di dunia ilmu pengetahuan.

24. UM UGM 2003


Dalam pemilihan ketua partai, Budi Santosa memperoleh tujuh belas suara atau terpaut tujuh suara dari
perolehan ketua terpilih, sedangkan satu calon lainnya hanya mendapat dua suara. Sementara itu, dua suara
menyatakan abstain dan satu suara dinyatakan gugur.
Berdasarkan isi bacaan di atas, pernyataan berikut yang salah ialah
A. Dalam pemilihan itu, terdapat tiga kandidat kelua partai
B. Pemenang dalam pemilihan itu mendapatkan dua puluh empat suara
C. Jumlah suara yang diperoleh pemenang lebih banyak daripada gabungan jumlah suara yang diperoleh calon
yang gagal
D. Jumlah suara yang diperoleh pemenang lebih banyak daripada gabungan jumlah suara yang diperoleh calon
yang gagal
E. Pemenang dalam pemilihan itu mendapatkan suara lebih dari separo jumlah pemilih.

25. UM UGM 2003


Awalnya, Pula Jawa terapung-apung di samudra. Seperti sepotong gabus, ia dihempaskan gelombang, terlempar
ke sana-sini. Agar pulau itu tetap bertahan di tempatnya, dibutuhkan sebuah paku untuk menancapkan ke bumi.
Paku itu berupa Bukit Tidar yang terletak di Magelang.
Bacaan di atas merupakan contoh
A. narasi
B. deskripsi
C. eksposisi
D. argumentasi
E. persuasi

SPMB
Jumlah penduduk dalam suatu negara merupakan salah satu potensi dasar yang dapat dimanfaatkan dalam
pembangunan. Potensi tersebut akan menjadi kekuatan dalam pelaksanaan pembangunan apalagi kualitas penduduknya
baik, hal ini antara lain tercermin pada tingginya tingkat kesehatan jasmani dan rohani, tingkat pendidikan dan
keterampilan, serta daya nalar penduduk negara tersebut. Disamping itu, kekuatan pembangunan tersebut akan menjadi
optimal apabila penduduk dapat berpartisipasi secara penuh dalam pelaksanaan pembangunan sesuai dengan kapasitas
dan kemampuan individu dan kemampuan kelompok.
Perkembangan jumlah penduduk Indonesia menurut jenis kelamin pada tahun 1980, 1985 dan 1990 menunjukkan
bahwa lebih dari separo jumlah penduduk Indonesia merupakan wanita. Dengan jumlah penduduk yang besar tersebut
partisipasi dan peranannya dapat didayagunakan serta dioptimalkan dalam derap pembangunan, bukan mustahil hasil
pembangunan yang telah dicapai pada saat ini masih dapat ditingkatkan lagi.

1. SPMB 2003
Pikiran utama kalimat terakhir kutipan di atas ialah ...
A. dengan jumlah yang besar
B. partisipasi dan peranannya
C. dapat didayagunakan serta dioptimalkan
D. hasil pembangunan yang telah dicapai
E. hasi pembangunan dapat ditingkatkan

2. SPMB 2003
“Nya” pada peranannya dalam kalimat terakhir kutipan di atas menunjuk pada ....
A. perkembangan penduduk
B. jumlah penduduk
C. jenis kelamin
D. penduduk Indonesia
E. penduduk wanita

Bidang Studi BAHASA INDONESIA 70


KUMPULAN SOAL – SOAL UJIAN MASUK PERGURUAN TINGGI NEGERI

3. SPMB 2003
Dari kalimat pertama bacaan di atas dapat disimpulkan bahwa ....
(1) banyaknya penduduk mempengaruhi besar kecilnya pemanfaatan penduduk untuk pembangunan
(2) penduduk merupakan aset yang dapat dimanfaatkan
(3) selain penduduk ada kekuatan-kekuatan lain yang dapat dimanfaatkan untuk pembangunan
(4) pembangunan memerlukan modal yang besar

4. SPMB 2003
Bacaan di atas menyatakan bahwa kekuatan pembangunan dapat optimal apabila ....
(1) tingkat kesehatan penduduk tinggi
(2) tingkat keterampilan dan daya nalar penduduk tinggi
(3) tingkat partisipasi penduduk tinggi
(4) jumlah penduduk wanita yang lebih banyak dari pria itu dapat didayagunakan

5. SPMB 2003
Paman sedang mengairi sawahnya.
Makna me – i pada kata mengairi sama dengan me – i pada kalimat ....
A. Ibu sedang membului ayam.
B. Ayah sedang menguliti kambing.
C. Adik memukuli Rina.
D. Bibi sedang memintai dana.
E. Dia menawari saya pekerjaan.

6. SPMB 2003
Penulisan yang tidak tepat terdapat dalam ....
A. aklamasi
B. aeronautika
C. fiksi
D. khromosom
E. apoteker

7. SPMB 2003
Anak-anak pria biasanya menyukai mobil-mobilan..
Kata ulang pada kalimat di atas sama maknanya dengan kata ulang di bawah ini, kecuali ....
A. orang-orangan
B. langit-langit
C. anak-anakan
D. dokter – dokteran
E. biri – biri

8. SPMB 2003
Pemakaian tanda baca yang betul terdapat pada ....
A. Indra rajin dan pandai, karena itu, nilainya bagus
B. Kemarin, semua orang hadir kecuali anda
C. Sari ingin bersekolah, tetapi ibu tidak mampu membiayainya
D. Kemarin ia datang, dan ia pergi
E. Jakarta, ibukota RI, berhawa panas

9. SPMB 2003
Unsur inti pada kalimat Mereka seharusnya sudah mengetahui tanggung jawabnya sendiri adalah ....
A. mereka sudah mengetahui
B. mereka mengetahui
C. mereka mengetahui tanggung jawabnya
D. mereka sudah mengetahui tanggung jawab
E. mereka seharusnya mengetahui
10. SPMB 2003
Di antara kalimat-kalimat berikut yang efektif adalah ….
A. Hanya dengan sedikit tenaga ia dapat memindahkan benda itu
B. Setelah pertigaan itu kamu, harus membelok
C. Dengan usaha yang keras anda lakukan tugas itu
D. Upacara itu dilakukan dengan khidmat
E. Waktu dan tempat kami haturkan.

Bidang Studi BAHASA INDONESIA 71


KUMPULAN SOAL – SOAL UJIAN MASUK PERGURUAN TINGGI NEGERI

11. SPMB 2003


Dua kata yang sama tulisannya, diucapkan beda dan artinya berbeda terdapat pada kalimat ….
A. Saya sangsi akan sanksi itu
B. Orang yang memakai kemeja coklat itu ke meja makan
C. Saya tahu ia suka makan tahu
D. Siapa bisa makan bisa ular
E. Bang Ali adalah nasabah bank BNI

12. SPMB 2003


Ungkapan yang mengandung pengertian apa yang dikerjakannya selalu gagal ialah ….
A. tangan panas
B. tangan dingin
C. tangan kosong
D. tangan hampa
E. tangan naik

13. SPMB 2003


Hasan, Kartini, Rusli, dan Anwar adalah tokoh dalam ....
A. Belenggu – Armyn Pane
B. Jalan Tak Ada Ujung – Muchtar Lubis
C. Salah Asuhan – Abdul Muis
D. Atheis – Ahdiat Kartamiharja
E. Layar Terkembang – STA

14. SPMB 2003


Majas metafora terdapat pada kalimat-kalimat berikut, kecuali ....
A. Dalam kebun tanah airku tumbuh sekuntum bunga
B. Aku ini binatang jalang
C. Rakyat adalah suara di langit tanah tercinta
D. Matahari adalah raja siang
E. Kerling danau di pagi hari

15. SPMB 2003


Makna kata membayarkan sama dengan makna kata berikut, kecuali ....
A. mengambilkan
B. membelikan
C. menjahitkan
D. menggandakan
E. membulatkan

16. SPMB 2003


Sebagai penutup surat resmi, kalimat yang tepat digunakan adalah ....
A. Atas perhatian Bapak, kami haturkan terima kasih
B. Terima kasih atas perhatiannya
C. Terima kasih sekali atas perhatian Bapak
D. Atas perhatian Ibu kami ucapkan makasih
E. Atas perhatian Ibu, kami ucapkan terima kasih

17. SPMB 2003


Peribahasa yang bermakna kepalang tanggung terdapat dalam ....
A. Ke langit tak sampai ke bumi tak nyata
B. Berdiang di abu dingin
C. Biduk tiris menanti karam
D. Biduk lalu kiambang bertaut
E. Dibakar tak hangus, direndam tak basah
18. SPMB 2003
Pola gabungan kata baju biru terdapat juga pada gabungan kata ....
A. do’a restu
B. ke Jakarta
C. rumah baru
D. panjang tangan
E. tangan dingin

Bidang Studi BAHASA INDONESIA 72


KUMPULAN SOAL – SOAL UJIAN MASUK PERGURUAN TINGGI NEGERI

19. SPMB 2003


Bibi bernyanyi.
Transformasi kalimat di atas tertera di bawah ini, kecuali ....
A. Bibi bernyanyi keroncong
B. Mengapa bibi bernyanyi
C. Siapa yang bernyanyi?
D. Bibi bernyanyi dengan merdu
E. Bibi sedang apa?

20. SPMB 2003


Kata berpegangan pada kalimat Ia berpegangan akar agar tidak hanyut terdiri atas ... morfem.
A. 2
B. 4
C. 3
D. 4
E. 5

21. SPMB 2003


Adik saya belajar matematika dengan serius.
Pola kalimat di atas tidak sama dengan pola kalimat di bawah ini, kecuali ...
(1) Ibu bernyanyi dengan merdu.
(2) Mereka mempelajari fisika selama dua tahun.
(3) Ketika Rudi datang, Ali pergi ke Jakarta.
(4) Ia kejatuhan mangga di teras.

22. SPMB 2003


Penulisan bilangan sesuai ejaan yang tepat terdapat pada kalimat ...
(1) Lima belas anak mengikuti lomba itu.
(2) Kedua anak itu akrab.
(3) Ia anak kedua dari keluarga Arman.
(4) 256 menghadiri pertemuan itu.

23. SPMB 2003


Datanglah engkau wahai maut
Lepaskan aku dari nestapa
Engkau lagi tempatku terpaut
Di waktu ini gelap gulita
(Amir Hamzah)
Menurut isinya puisi di atas disebut ....
(1) balada
(2) himne
(3) satire
(4) elegi

24. SPMB 2003


Pagi ini matahari itu bersinat terang sekali.
Inti frase kalimat di atas ....
(1) pagi
(2) matahari
(3) terang
(4) itu

25. SPMB 2003


Pengarang dan karyanya berikut ini benar kecuali
(1) Belenggu – STA
(2) Salah Asuhan – Amir Hamzah
(3) Atheis – Ahdiat Kartamiharja
(4) Layar Terkembang – STA (Sutan Takdir)

Bidang Studi BAHASA INDONESIA 73


KUMPULAN SOAL – SOAL UJIAN MASUK PERGURUAN TINGGI NEGERI

SOAL TAHUN 2002

SPMB

Minuman keras (miras) adalah jenis minuman yang mengandung alkohol. Alkohol termasuk zat adiktif, yaitu zat yang
dapat menimbulkan adiksi (ketagihan) dan ketergantungan.
Pemakaian miras dapat menimbulkan gangguan mental organik (GMO) yaitu gangguan dalam fungsi berpikir,
merasakan, dan berperilaku. Timbulnya GMO itu disebabkan reaksi langsung alkohol pada sel-sel saraf ousat. karena
sifat adiktif alkohol itu, orang yang meminumnya lama-kelamaan tanpa sadar akan menambah takaran/dosis sampai
pada dosis keracunan atau mabuk.
Mereka yang terkena GMO biasanya mengalami perubahan perilaku, seperti mau berkelahi atau melakukan
tindakan kekerasan lainnya, tidak mampu menilai realitas, terganggu fungsi sosialnya, dan terganggu pekerjaannya.
Perubahan fisiologis juga terjadi, seperti cara berjalan tidak mantap, muka merah, atau mata juling. Perubahan
psikologis yang dialami oleh mereka biasanya berupa mudah tersinggung, berbicara ngawur, atau kehilangan
konsentrasi.
Mereka yang sudah ketagihan biasanya mengalami suatu gejala yang disebut sindrom putus alkohol, yaitu rasa
takut diberhentikan minum alkohol. Badan mereka akan sering gemetar dan jantung berdebar-debar. Mereka akan
cemas, gelisah, murung, dan banyak berhalusinasi.

1. SPMB 2002
Topik bacaan di atas adalah ....
A. pemakaian minuman keras dalam jangka panjang
B. kemudaratan minuman keras
C. peranan minuman keras
D. pemrosesan minuman keras
E. gangguan mental organik akibat alkohol

2. SPMB 2002
Yang tercantum di bawah ini dikemukakan dalam bacaan di atas kecuali ....
A. alkohol termasuk zat adiktif
B. GMO timbul akibat reaksi langsung sel-sel saraf pusat terhadap alkohol
C. sindrom putus alkohol menjadi suatu penderitaan
D. perubahan perilaku akibat miras antara lain berupa banyak mengkhayal
E. yang terkena GMO biasanya menyukai tindakan kekerasan

3. SPMB 2002
Pernyataan di bawah ini cocok dengan isi bacaan di atas kecuali ....
A. Orang yang ingin tetap sehat lebih baik tidak meminum miras.
B. Kebiasaan meminum miras berbahaya bagi kesehatan fisik dan mental.
C. Sindrom putus alkohol tidak berbahaya asalkan persediaan miras cukup
D. Demi ketentraman lahir dan batin, jauhilah minuman keras
E. Hekmah meminum minuman keras hampir tidak ada.

4. SPMB 2002
Keinginan untuk menambah takaran dalam menikmati miras disebabkan :
A. banyaknya berhalusinasi
B. sifat adiktif alkohol
C. gangguan dalam fungsi berpikir
D. perubahan psikologis
E. kecenderungan tindakan kekerasan.

5. SPMB 2002
(a) Di samping memiliki kekuatan, ia juga memiliki kelemahan
(b) Namun, ia cenderung dianggap “lebih” karena tujuan hidup seorang pahlawan bukan semata-mata untuk
dirinya sendiri.
(c) Seorang pahlawan tidak berbeda dengan manusia lainnya.
Urutan yang tepat dari kalimat-kalimat di atas untuk membentuk paragraf adalah ….
A. (a) – (b) – (c)
B. (a) – (c) – (b)
C. (b) – (a) – (c)
D. (b) – (c) – (a)
E. (c) – (a) – (b)

Bidang Studi BAHASA INDONESIA 74


KUMPULAN SOAL – SOAL UJIAN MASUK PERGURUAN TINGGI NEGERI

6. SPMB 2002
(1) Setiap hari jumlahnya berlipat dua; dua helai pada hari kedua, empat pada hari ketiga, delapan pada hari
keempat, dan seterusnya. (2) Di sebuah kolam, menurut teka-teki itu, tumbuh sehelai daun teratai. (3) Jawabnya,
“Pada hari kedua puluh sembilan”. (4) Orang Perancis menggunakan teka-teki untuk mengajarkan pertumbuhan
penduduk yang berlipat ganda kepada anak sekolah. (5) “Kalau kolam teratai itu penuh pada hari ke tiga puluh,
kapankah kolam itu tertutup separuhnya oleh daun teratai?”
Urutan kalimat yang tetap untuk paragraf di atas adalah ….
A. (2) – (1) – (3) – (5) – (4)
B. (4) – (2) – (1) – (3) – (5)
C. (4) – (5) – (1) – (2) – (3)
D. (4) – (2) – (1) – (5) – (3)
E. (5) – (1) – (2) – (3) – (4)

7. SPMB 2002
Presiden membicarakan tentang serangan Amerika terhadap Afghanistan dalam sidang kabinet kemarin.
Kalimat tersebut akan menjadi kalimat yang baku jika diperbaiki menjadi …
A. Presiden membicarakan serangan Amerika terhadap Afghanistan dalam sidang kabinet kemarin.
B. Dalam sidang kabinet, Presiden membicarakan tentang serangan Amerika terhadap Afghanistan.
C. Presiden membicarakan tentang serangan Amerika terhadap Afghanistan
D. Presiden berbicara serangan Amerika terhadap Afghanistan di depan sidang kabinet kemarin
E. Presiden membicarakan tentang serangan Amerika terhadap Afghanistan kemarin di depan sidang kabinet

8. SPMB 2002
Data yang digunakan untuk menjawab semua permasalahan yang ada dalam penelitian ini dapat dipilah menjadi
dua, yaitu data utama dan data penunjang.
Agar menjadi kalimat efektif, kalimat tersebut diperbaiki menjadi :
A. Data penelitihan ini dipilah menjadi dua yaitu data utama dan data penunjang.
B. Data untuk menjawab permasalahan dalam penelitihan ini dapat dipilah menjadi dua, yaitu data utama dan data
penunjang.
C. Data yang digunakan dalam penelitihan ini dapat dipilah menjadi dua yaitu data utama dan data penunjang.
D. Data ada dua kelompok, yakni data utama dan data penunjang
E. Data terdapat dua jenis, yakni data utama dan data penunjang

9. SPMB 2002
Pembangunan pertanian tidak hanya membatasi pada pengembangan tanaman tradisional.
Kalimat di atas dianggap tidak efektif karena penggunaan kaya yang salah. Kata yang salah itu adalah ….
A. pembangunan
B. tidak
C. membatasi
D. tanaman
E. tradisional

10. SPMB 2002


Semua kalimat berikut adalah kalimat yang tidak menggunakan ragam baku kecuali ....
A. Dalam rapat itu membicarakan masalah hasil evaluasi belajar mahasiswa.
B. Buku itu bermanfaat bagi penulisan karya ilmiah.
C. Pada bahasa Indonesia yang baik dan benar membicarakan ejaan dan ragam.
D. Bagi pegawai negeri sipil tidak boleh ikut partai.
E. Rencana undang-undang disyahkan oleh DPR.

11. SPMB 2002


Kata berlayar yang dahulu hanya dipakai dalam kalimat seperti Nelayan tradisional berlayar mencari ikan setiap
malam kini juga dipakai dalam kalimat seperti Marilah berlayar bersama kapal pesiar kaimi. Kata itu mengalami :
A. penyempitan makna
B. perluasan makna
C. perubahan makna yang membaik
D. perubahan makna yang memburuk
E. perubahan makna menjadi makna kiasan

Bidang Studi BAHASA INDONESIA 75


KUMPULAN SOAL – SOAL UJIAN MASUK PERGURUAN TINGGI NEGERI

12. SPMB 2002


Kelompok kata yang di bawah ini mempunyai hubungan makna yang berlawanan secara mutlak kecuali ....
A. laki-laki – perempuan
B. mati – hidup
C. jantan – betina
D. salah – benar
E. ibu – anak

13. SPMB 2002


Kelompok kata berikut menunjukkan benda atau hal sejenis kecuali ....
A. panci, wajan, periuk
B. gurami, kakap, mujair
C. gunting, jarum, benang
D. sepatu, sandal, selop
E. guru, buku, disket

14. SPMB 2002


Kesedihannya begitu mendalam karena kehilangan kekasihnya.
Ungkapan yang tepat untuk menggambarkan kesedihan itu ialah
A. Hancur badan dikandung tanah, budi baik terkenang jua.
B. Bagaikan batu hitam tak tersanding
C. Air diminum rasa duri, nasi dimakan rasa sekam
D. Berani hilang tak hilang, berani mati tak mati.
E. Bagaikan menegakkan benang basah.

15. SPMB 2002


Situasi keamanan dan kekayaan alam yang dimiliki Kutai Barat merupakan modal untuk mengembangkan wisata
alam yang layak ditawarkan kepada para wisatawan.
Kalimat tersebut merupakan kalimat luas yang dibentuk dari kalimat inti dengan unsur-unsur :
A. situasi keamanan dan kekayaan alam, dimiliki, Kutai Barat.
B. situasi keamanan dan kekayaan alam, merupakan, modal
C. situasi keamanan dan kekayaan alam, mengembangkan, wisata alam
D. situasi keamanan dan kekayaan alam, layak ditawarkan.
E. situasi keamanan dan kekayaan alam, layak ditawarkan, wisatawan.

16. SPMB 2002


Pada empat tahur terakhir ini makin banyak pemerintah yang negaranya pernah dikuasai rezim yang zalim
membuka kembali sebagian sejarah hitamnya yang diwarnai tindak kekejaman aparat bersenjatanya sendiri pada
masa lampau.
Berikut ini bukan unsur kalimat inti di dalam kalimat tersebut kecuali ....
A. membuka kembali
B. dikuasai
C. diwarnai
D. rezim yang zalim
E. pada empat tahun terakhir ini

17. SPMB 2002


Jum’at lalu, Amerika mengakui kehilangan satu helikopter karena cuaca buruk dan semua krunya berhasil
diselamatkan dan dievaluasi dari Afghanistan oleh satu helikopter lain.
Inti kalimat di atas adalah ….
A. Amerika mengakui kehilangan
B. Ju’mat lalu Amerika kehilangan helikopter
C. Karena cuaca buruk helikopter Amerika jatuh berkeping-keping
D. Kru helikopter berhasil diselamatkan dan dievaluasi dari Afghanistan
E. Dalam kecelakaan tersebut, empat orang luka-luka dan telah diselamatkan dan dievaluasi oleh satu helikopter
lain.

Bidang Studi BAHASA INDONESIA 76


KUMPULAN SOAL – SOAL UJIAN MASUK PERGURUAN TINGGI NEGERI

18. SPMB 2002


Pelayan toko memberikan uang kembali.
Pola kalimat di atas sama dengan pola kalimat
A. Ketua RT ikut berperan dalam kebersihan lingkungan
B. Kakak belajar dengan tekun di perpustakaan
C. Adik menangis di pelukan Ibu.
D. Kereta api berhenti di setiap stasiun.
E. Bapak Kepala Sekolah menyerahkan buku Tabanas

19. SPMB 2002


1) perempuan Jerman itu berdecak
2) dengan menggenggam tiga buku tebal
3) sambil menggeleng-gelengkan kepala
4) di tangan kiri dan empat di tangan kanan
Urutan penggalan kalimat di atas yang membentuk kalimat yang baik adalah
A. 1, 2, 3, 4
B. 2, 4, 1, 3
C. 3, 1, 4, 2
D. 4, 3, 2, 1
E. 2, 3, 4, 1

20. SPMB 2002


Bengkalis sebuah kabupaten di Riau yang sebagian wilayahnya terletak di Riau Kepulauan, ternyata memiliki
potensi alam yang sangat besar.
Kalimat di atas merupakan perluasan dari kalimat ….
A. Bengkalis sebuah kabupaten di Riau
B. Sebagian wilayah Bengkalis terletak di Riau Daratan
C. Sebagian lainnya terletak di Riau Kepulauan
D. Bengkalis memiliki potensi alam.
E. Potensi alam yang sangat besar

21. SPMB 2002


Kata majemuk yang berkonstruksi A dan B terdapat pada kalimat :
A. “Bila ingin berhasil, patahkan dulu kaki tangan musuh!” kata komandan pasukan tempur
B. Rumah sakit wajib menyiapkan kamar rawat inap bagi pasien yang tidak mampu.
C. Kereta api itu datang tepat waktu
D. Orang itu sudah seharusnya memberikan contoh yang baik kepada anak-anak mereka
E. Karena mendung, matahari tidak menampakkan sinarnya

22. SPMB 2002


Imbuhan ber–an pada kata berhadapan di atas ring mempunyai makna yang sama dengan imbuhan ber-an di dalam
kalimat berikut kecuali ....
A. Rumah Adit berdekatan dengan rumah Sari.
B. Kami berkenalan sepuluh tahun yang lalu.
C. Kedua pasangan itu bertatapan muka dengan penuh rasa cinta.
D. Ayah saya berpandangan sebaliknya
E. Orang-orang berjalan berpegangan tangan.

23. SPMB 2002


Kata berawalan me– yang tidak menyatakan kerja terdapat dalam kalimat
A. Dia tidak mengakui perbuatannya yang salah.
B. Penduduk desa itu banyak yang merotan.
C. Para penerjun telah mendarat dengan selamat.
D. Kami mengontrak rumah di Pulomas.
E. Kami datang menjelang pesta dimulai.
24. SPMB 2002
Kalimat yang ditulis secara tepat adalah ….
A. Harga buku yang difotokopi ini, Rp 15.000,– per jilid.
B. Harga buku yang di-photocopi ini, Rp 15.000,– per jilid.
C. Harga buku yang difotokopi ini, Rp 15.000,00,– per jilid.
D. Harga buku yang diphotocopi ini, Rp 15.000,– per jilid.
E. Harga buku yang difotokopi ini, Rp 15.000,00 per jilid.

Bidang Studi BAHASA INDONESIA 77


KUMPULAN SOAL – SOAL UJIAN MASUK PERGURUAN TINGGI NEGERI

25. SPMB 2002


Kalimat yang menggunakan ejaan yang benar adalah
A. “Kerajaan Siak Sri Indrapura adalah sebuah kerajaan Melayu Islam yang terbesar di Riau”. Tegas Fauziah
Rahman, Salah satu keturunan Datuk Sri Amar Perkasa.
B. Minuman “Laksamana Mengamuk’ terbuat dari campuran santan kelapa yang telah dimasak dan kemudian
dicampur dengan gula pasir.
C. Pasar yang dibuka sejak pukul 15.00 WIB ini juga menyediakan makanan modern, seperti pizza, ayam saus,
hingga fried chicken.
D. Namun setiap harinya, para penjual yang terdiri atas mahasiswa dan mahasiswi dipungut bayaran sebesar Rp
1.500,00
E. Sambalnya pun mempunyai ciri-ciri khusus, ada sambal cencalo, sambal belacan tumis, sambal terong asam
dan sambal uleg mentah.

SOAL BAHASA INDONESIA TAHUN 2001

UMPTN

Kondisi sumber daya hutan sudah berada pada tahap yang sangat mengkhawatirkan, sementara lahan kritis terus
meluas. Dari 46,7 juta hektare hutan alam produksi yang ada, sekarang 14,2 juta hektare sudah teerwujud hutan rusak
dan tanah kosong.
Berdasarkan catatan terakhir hanya 13,6 juta hektare hutan bekas tebangan yangkondisinya agak baik,
sedangkan hutan alam primer yang tersisa tinggal 18,9 juta hektare. Bahkan, kawasan hutan konservasi dan hutan
lindung sudah dirambah dan dieksploitasi kayunya secara ilegal dengan kerusakan mencapai 5,9 juta hektare. Hal itu
diungkapkan oleh Menteri Pertanian dan Kehutanan pada acara Puncak Penghijauan dan Konservasi Alam Nasional
(PPKAN) ke –40 di Desa Jabiran, Kabupaten Kapuas, Kalimantan Tengah.
Kerusakan hutan antara lain merupakan akibat euforia reformasi yang disalhtafsirkan serta provokasi dari
sekelompok orang yang terdorong emosi dan nafsu serakah untuk memenuhi ambisinya. Masyarakat didorong untuk
melakukan tindakan desdruktif terhadap hutan.
Penyelenggaraan PPKAN tahun 2000 merupakan momentum yang tepat untuk menyadari pentingnya
pengelolaan hutan dengan prinsip manfaat bagi kesejahteraan rakyat secara menyeluruh, berkeadilan, dan berkelanjutan.
Upaya rehabilitasi dan konservasi alam harus melibatkan banyak pihak. Di samping itu, permasalahan tersebut tidaklah
homogen di wilayah-wilayah. Oleh karena itu, upaya rehabilitasi dan konservasi alam harus dirancang partisipatif
sehingga dapat dipraktekkan secara berkesinambungan.

1. UMPTN 2001
Judul yang tepat untuk bacaan di atas adalah sebagai berikut.
A. Penyelenggaraan PPKAN Tahun 2000
B. Kondisi sumber Daya Hutan Sangat Mengkhawatirkan
C. Upaya Rehabilitasi dan konservasi Melibatkan Banyak Pihak
D. Kerusakan Hutan
E. Rehabilitasi dan Konservasi Hutan

2. UMPTN 2001
Berdasarkan bacaan di atas, kerusakan hutan terkjadi akibat
A. masyarakat didorong untuk melakukan tindakan desdruktif terhadp hutan
B. penebangan kayu yang dilakukan secara tidak sah.
C. lahan kritis terus meluas
D. adanya hutan rusak dan tanah kosong yang mencapai 14,2 juta hektare
E. pengelolaan yang tidak mempertimbangkan kesejahteraan masyarakat dan tidak melibatkam masyarakat.

3. UMPTN 2001
Hal berikut diungkapkan pada bacaan di atas.
A. Acara Puncak Penghijauan dan Konservasi Alam (PPKAN) ke-40 di Desa Jabiran merupakan kegiatan yang
harus dilakukan.
B. Hutan bekas tebangan yang kondisinya agak baik hanya 13,6 juta hektare
C. Hutan lindung mengalami kerusakan sebanyak 6,7 %
D. Tanpa pelaksanaa PPKAN, masyarakat tidak dapat menyadari pentingnya.
E. Diperkirakan bahwa 14,2% hutan alam produksi telah mengalami kerusakan.

Bidang Studi BAHASA INDONESIA 78


KUMPULAN SOAL – SOAL UJIAN MASUK PERGURUAN TINGGI NEGERI

4. UMPTN 2001
Topik bacaan di atas adalah ....
A. Upaya rehabilitasi dan konservasi alam harus melibatkan banyak pilihan
B. Pelaksanaa PPKAN merupakan momentum yang sangat tepat.
C. Kerusakan hutan diakibatkan oleh euforia reformasi.
D. Kerusakan hutan mencapai tahap yang mengkhawatirkan.
E. Hutan Konservasi dan kutan lindung sudah diekploitasi kayunya secara ilegal

5. UMPTN 2001
Berdasarkan bacaan di atas, pernyataan berikut benar.
A. Setiap wilayah memiliki persoalan kehutanan yang berbeda
B. Penanganan rehabilitasi dan konservasi alam bukanlah persoalan yang bersifat heterogen
C. Kerusakan hutan diakibatkan belum dilaksanakannya PPKAN selama masa Orde Baru.
D. Kesadaran masyarakat untuk menjaga hutan muncul setelah PPKAN dilaksanakan.
E. Kerusakan hutan berada pada tahap yang mengkhawatirkan. Persoalan itu telah diungkapakn oleh Menteri
Pertanian dan Kehutanan.

6. UMPTN 2001
Penulisan kata serapan di bawah ini benar KECUALI ....
A. kualitas
B. kuitansi
C. kuesioner
D. jadual
E. jumat

7. UMPTN 2001
Semua bentuk kata berulang dalam kalimat-kalimat berikut ini menyatakan makna saling KECUALI ....
A. Kedua kelompok itu berlempar-lemparan batu
B. Sebagai bangsa yang santun kita harus hormat-menghormati.
C. Yanti dan Yanto tolong-menolong menyelesaikan tugas yang berat.
D. Kedua bus itu dahulu-mendahului di jalan bebas hambatan.
E. Soal masak-memasak saat ini menjadi urusan kaum bapak juga.

8. UMPTN 2001
Kedatangan tamu disambut dengan upacar adat.
Fungsi imbuhan ke-an pada kalimat di atas sama dengan fungsi –nya pada kalimat
A. Dia mengecat rumahnya dengan cat putih.
B. Hasil kebunnya cukup biaya hidup keluarga.
C. Buku itu berjudul Lahirnya Pancasila.
D. Sakitnya tidak tertahankan olehku.
E. Obat itu bukan main mahalnya.

9. UMPTN 2001
Ibu membacakan adik dongeng.
Fungsi imbuhan me-kan pada kata membacakan sama dengan imbuhan me-kan pada kata dalam kalimat
A. Pak Haji menjahitkan kain sarungnya yang baru
B. Ayah membelikan adik sepeda
C. Pemuda-pemuda mengucapkan sumpahnya dalm upacara tadi padi.
D. Pelayan menyediakan makan pagi.
E. Peristiwa itu sangat menyedihkan.

10. UMPTN 2001


Kata bercetak miring dalam kalimat berikut ini baku KECUALI
A. Pertemuan itu dihadiri para rohaniawan dari berbagai negara.
B. Heroisme di mana pun tidak pernah terlepas dari situasi sosial politik
C. Membuat masyarakat gemar berolahraga bukanlah pekerjaan yang mudah.
D. Untuk mempercayai seseorang, diperlukan kehati-hatian.
E. Untuk memcahkan masalah tersebut, diperlukan daya analisis yang tinggi.

Bidang Studi BAHASA INDONESIA 79


KUMPULAN SOAL – SOAL UJIAN MASUK PERGURUAN TINGGI NEGERI

11. UMPTN 2001


Kalimat yang benar pemakaian tanda bacanya adalah ....
A. Mereka bekerja terus, walaupun malam telah larut.
B. Lalu lintas padat sekali, karena itu kami datang terlambat.
C. Kami membutuhkan tangga, gergaju, dan palu.
D. Anak-anak, pemuda dan orang tua hadir dalam upacara itu.
E. Rupannya dia tidak tahu , bahwa besok libur.

12. UMPTN 2001


Penulisan kata yang tidak tepat terdapat pada kata-kata bercetak miring dalam kalimat
A. Warga kompleks membangun gedung pertemuan secara swadaya.
B. Pada saat pascapanen, harga gabah sering merosot.
C. Setiap bus antarprovinsi harus dalam kondisi laik jalan.
D. Para tunawisma di bawah jembatan layang akan ditertibkan.
E. Banyak pelajar melakukan kerjasosial untuk mengisi liburan.

13. UMPTN 2001


Imbuhan ber– yang mempunyai arti mempunyai terdapat dalam kalimat
A. Adik bersepeda ke sekolah
B. Murid berusaha menyelesaikan tugasnya
C. Saya bermaksud mengunjungi nenek.
D. Ibu berbelanja di pasar.
E. Petinju itu berkeringat selama bertanding.

14. UMPTN 2001


Tangkis–menangkis pukulan diantara kedua pesilat itu benar-benar memukau penonton.
Arti pengulangan tangkis-menangkis pada kalimat di atas sama dengan arti pengulangan pada kata bercetak miring
di bawah ini KECUALI ....
A. Budaya tolong-menolong diantara tetangga harus terus dipelihara
B. Kedua remaja itu tidak saling menyapa, tetapi lirik-melirik juga.
C. Caci-mencaci yang dilakukan mereka sangat merusak suasana.
D. Kami bekerja terus-menerus selama dua belas jam.
E. Awalnya dorong-mendorong, akhirnya mereka berkelahi.

15. UMPTN 2001


Pola kalimat inti yang sama dengan Pemandangan di desa itu sangat indah. adalah ....
A. Para mahasiswa harus belajar tekun.
B. Baju seragam kerja ayah sangat bagus.
C. Indahnya pemandangan di Puncak Pas.
D. Keberaniannya tidak diragukan lagi.
E. Kegigihan para mahasiswa pada masa reformasi membuahkan hasil.

16. UMPTN 2001


Nelayan menangkap ikan di laut.
Pola kalimat di atas sama dengan polakalimat berikut KECUALI
A. Siswa membaca buku di perpustakaan.
B. Kami bergembira pad pesta ulang tahunnya.
C. Kemarin polisi menangkap pencopet di pasar itu.
D. Kami harus mengembalikan buku-buku itu pada tempatnya semula.
E. Mereka menonton pesta rakyat di alun-alun.

17. UMPTN 2001


Pola kalimat Indonesia pernah juga berjaya sama dengan pola kalimat
A. Dagangan mereka kadang-kadang tidak laku.
B. Dia bersedia berkorban demi kepentingan negara.
C. Puisi itu di tulis untuk sahabatnya yang telah tiada.
D. Uangnya disimpan di dalam lemari besi
E. Gaji terasa kurang karean inflasi belum juga berakhir.

Bidang Studi BAHASA INDONESIA 80


KUMPULAN SOAL – SOAL UJIAN MASUK PERGURUAN TINGGI NEGERI

18. UMPTN 2001


Bu Karta memilih kain itu dan Pak Karta membayarnya.
A. Adik tidur saja di sini, atau Adik ikut pulang juga .
B. Hasan tidak mau belajar serius, melainkan bermain-main sepanjang hari.
C. Para mahasiswa aktif mengikuti diskusi, hanya Andi yang pasif.
D. Kaka makan di rumah, atau kakak makan di warung.
E. Miryati dan kepala regu penyiar pria mengetahu hal itu.

19. UMPTN 2001


Karena pengaruh globalisasi dan tuntutan kehidupan, masyarakat akan mengalami pergeseran apresiasi dan cara
pandang terhadap berbagai aspek kehidupan.
Frase verba dalam pernyataan di atas adalah ....
A. pengaruh globalisasi
B. akan mengalami
C. tuntutan kehidupan
D. pergeseran apresiasi
E. cara pandang

20. UMPTN 2001


Kesenimbangunan pembangunan hanya mungkin dilaksanakan jika ketahanan Nasional dan kualitas trilogi
pembangunan meningkat.
Kalimat di atas menggunakan frase nomina berikut KECUALI ....
A. kesinambungan pembangunan
B. ketahanan nasional
C. kualitas trilogi pembangunan
D. trilogi pembangunan
E. mungkin dilaksanakan

21. UMPTN 2001


Penghilangan unsur kalimat terdapat pada
A. Bagaimanakah keadaan ibumu ?
B. Saya menonton televisi kemarin
C. Kemarin saya menonton televisi
D. Saya menonton televisi dan adik juga.
E. Otong bunuh diri.

22. UMPTN 2001


Kami semua tengah menantikan angin baik untuk melancarkan aksi menentang mereka.
Makna angin dalam kalimat di atas adalah
A. cuaca
B. keadilan
C. suasana
D. kabar
E. kesempatan

23. UMPTN 2001


Hati-hati membawa barang ini. Jangan sampai jatuh, ya!
Kata jatuh yang dipakai dalam arti seperti pada kalimat di atas terdapat pada
A. Nilainya jatuh gara-gara tidak memperhatikan pertujuk soal ujian.
B. Nama Institusi kita tidak akan jatuh hanya karena kasus ini.
C. Selamat bertanding pembalap itu tidak memperdulikan berapa kali jatuh bangun di medan yang becek.
D. Sejak lama dia jatuh hati dengan kebudayaan Indonesia.
E. Sudah lama ia jatuh sakit.

24. UMPTN 2001


Aksi jual beli saham di Bursa Efek Jakarta kembali bergairah.
Pasangan kata di bawah ini yang setipe dengan jual beli tanah ialah
A. pantang mundur
B. terus terang
C. serah terima
D. cuci tangan
E. sepak terjang

Bidang Studi BAHASA INDONESIA 81


KUMPULAN SOAL – SOAL UJIAN MASUK PERGURUAN TINGGI NEGERI

25. UMPTN 2001


Di antara frase-frase berikut yang mempunyai tiga tafsiran makna adalah
A. anak guru yang pandai
B. penanam padi krawang
C. lima ribuan
D. surat nikah
E. lukisan Affandi

26. UMPTN 2001


Penangan stres sangat bersifat pribadi. Artinya, penanganan setiap penderita berbeda. Penanganan tersebut lebih
banyak menyangkut perawatan jiwa. Misalnya, mendekatkan diri kepada Tuhan mengungkapkan segala keluhan
kepada sahabat, menangis sepuas-puasnya, memaki-maki hewan, memukul-mukul kasur, atau mendatangi tempat
rekreasi. Memang, penanganan stres bisa juga dengan menggunakan obat-obatan. Akna tetapi, hali itu sering
mengakibatkan ketergantungan atau ketagihan.
Inti paragraf di atas adalah
A. pribadi stres
B. pendekatan stres
C. keistimewaan stres
D. penanganan stres
E. penderita stress

27. UMPTN 2001


Wawasan Nusantara tidak hanya bertujuan untuk mewujudkna kesejahteraan bagi bangsa Indonesia saja, tetapi
juga ikut serta dalam mewujudkan kebahagiaan bagi seluruh umat manusia.
Penggunaan kata pada kalimat di atas dapat dihemat dengan menghilangkan
A. untuk dan saja
B. untuk dan tetapi
C. untuk, saja, dan tetapi
D. untuk, saja dan bagi
E. untuk, tetapi, dan bagi.

28. UMPTN 2001


Nenek itu…masa lalunya yang indah di kapal pesiar.
Pengisis titik-titik dalam kalimat yang tepat adalah
A. bercerita
B. menceritakan tentang
C. bercerita tentang
D. bercerita dengan
E. menceritakan mengenai

29. UMPTN 2001


Pemerintah Indonesia melakukan berbagai daya dan upaya untuk merangsang kedatangan wisman ke Indonesia.
Kata yang tepat untuk menggambarkan daya dan upaya adalah
A. strategi.
B. strategis
C. strategik.
D. strategisasi
E. strata.

30. UMPTN 2001


Kalimat di bawah ini yang tepat digunakan sebagi penutup surat dinas ialah
A. Atas perhatian Saudara, kami ucapkan terima kasih.
B. Atas perhatian Saudara, kami menghaturkan ucapan terima kasih.
C. Atas perhatian Saudara, diperbanyak ucapan terima kasih.
D. Atas perhatian Saudara, beribu-ribu terima kasih disampaikan.
E. Atas perhatian Saudara, sebelum dan sesudahnya diucapakan terima kasih.

Bidang Studi BAHASA INDONESIA 82


KUMPULAN SOAL – SOAL UJIAN MASUK PERGURUAN TINGGI NEGERI

31. UMPTN 2001


Saya tidak menyetujui bahwa usul tersebut diajukan sekarang.
Yang diingkari dalam kalimat itu adalah
A. menyetujui usul tersebut
B. usul tersebut diajukan sekarang.
C. saya menyetujui usul tersebur dengan diajukan sekarang.
D. menyetujui usul tersebut diajukan sekarang.
E. diajukan sekarang.

32. UMPTN 2001


Premis Umum : Tugas TNI mengawal pertahanan dan keamanan nasional.
Premis khusus : Pak Tono Anggota TNI
Rumusan simpulan yang tepat adalah
A. Tugas utama Pak Tono menjaga stabilitas nasional
B. Pak Tono bertugas mengawal pertahanan dan keamanan nasional.
C. Karena Pak . Tono TNI, ia harus menjaga stabilitas nasional.
D. Pertahanan dan keamanan nasional harus dijaga Pak. Tono.
E. Pak Tono bertugas menjaga stabilitas nasional.

33. UMPTN 2001


Makin samar
mana mulia, mana hina
mana kemajuan, mana kemunduran.
Katakanlah,
adakah kemajuan kalau kita lebih banyak mendirikan.
bank dan ruang gudang dari candi atau mesjid.
Kalau kita lebih menimbang kasi orang dengan uang dari hati.
Perasaan dan sikap penyair terhadap dunianya waktu itu adalah
A. kecewa sehingga benci pada kemajuan
B. cemas mekihat kemajuan pembangunan yang terlalu pesat.
C. samar-samar melihat maju mundurnya pembangunan.
D. ragu atas kemajuan materi semata, apakah itu maju atau mundur.
E. frustasi karena kemajuan itu tidak jelas

34. UMPTN 2001


Klise adalah ungkapan yang sangat penting diulang penggunaannya. Berikut ini ada sebuah ungkapan yang bukan
klise, yaitu
A. Siang berebut dengan malam
B. Bulan bersembunyi di balik awan.
C. Matahari tenggelam di ufuk barat.
D. Nyiur melambai di tepi pantai.
E. Dinginnya tak tercatat di termometer.

35. UMPTN 2001


Ombak memecah di tepi pantai.
Angin berhembus lemah lembut.
Puncak kelapa melambai-lambai.
Di ruang angkasa awan bergelut.
Puisi di atas menggunakan gaya bahasa
A. hiperbola
B. personifikasi
C. repetisi
D. asosiasi
E. metafora

Bidang Studi BAHASA INDONESIA 83

Anda mungkin juga menyukai